Download as pdf or txt
Download as pdf or txt
You are on page 1of 102

9.

CIRCLE

1. INTRODUCTION
Definition: The locus of a point which moves in a plane such that its distance
from a fixed point in that plane always remains the same (i.e., constant) is P moving point
known as a circle.
The fixed point is called the centre of the circle and the distance between the .
fixed point
fixed point and moving point is called the radius of the circle.

2. DIFFERENT FORM OF EQUATION OF CIRCLE


Figure 9.1

2.1 General Form


The general equation of a circle is of the form x2 + y2 + 2gx + 2fy + c = 0 where g, f, and c are constants.
1 1
(a) Centre of the circle ≡ (–g, –f). i.e., (– coefficient of x, – coefficient of y).
2 2
(b) Radius of the circle = g2 + f 2 − c .

Nature of the circle:


(a) If g2 + f2 –c > 0, then the radius of the circle will be real. Hence, it is possible to draw a circle on a plane.
(b) If g 2+ f2 – c = 0, then the radius of the circle will be zero. Such a circle is known as point circle.
(c) If g2 + f2 – c < 0, then the radius g2 + f 2 − c of the circle will be an imaginary. Hence, it is not possible to
draw a circle.
The condition for the general second degree equation to represent a circle:
The general equation ax2 + 2hxy + by2 + 2gx +2fy + c=0 represents a circle iff
(a) a = b ≠ 0 i.e. the coefficient of x2 = the coefficient of y2 ≠ 0.
(b) h = 0 i.e. the coefficient of xy is 0.
a h g
(c) ∆ = h b f = abc + 2hgf – af2 – bg2 – ch2 ≠ 0, it implies that the general equation is non degenerate (i.e.
g f c
equation cannot be written into two linear factors)
(d) g2 + f2 – c ≥ 0
9 . 2 | Circle

PLANCESS CONCEPTS

• The general equation ax2 + 2hxy + by 2 + 2gx + 2fy + c =0 can be written in matrix form as

 a h g  x 
a h   x    
 x y      + 2gx + 2fy + c =0 and  x y 1 h b f   y  = 0
h b   y  g f c  1 
  
• Degeneracy condition depends on determinant of 3x3 matrix and the type of conic depends on
determinant of 2x2 matrix.
• Also the equation can be taken as intersection of z =ax2 + 2hxy + by 2 and the plane z =
− 2gx + 2fy + c ( )
Vaibhav Gupta (JEE 2009, AIR 54)

2.2 Standard Form


The equation of circle with center (0,0) and radius ‘a’ is x2 + y 2 =
a2 .

P(x,y)

X’ X
O (0,0)

2 2 2
X +Y =a

Y’

Figure 9.2: Standard Form

2.3 Central Form


The equation of the circle with centre (h, k) and radius ‘r’ is (x – y)2 + (y – k)2 = r2.
Y
P(x,y)
r

C(h,k)

2 2 2
(x-h) +(y-k) =r

X’ X
O

Y’
Figure 9.3: Central form
M a them a tics | 9.3

2.4 Diametric Form


Circle on a given diameter: The equation of the circle with (x1, y1) and (x2, y2) as (x,y)
the end points of the diameter is given by
(x – x1)(x – x2) + (y – y1) (y – y2) = 0
(x1,y1) (x2,y2)
 x + x 2 y1 + y 2 
Centre ≡  1 , 
 2 2 
2 2
 x2 − x1   y 2 − y1 
and, Radius =   +   . Figure 9.4: Diametric Form
 2   2 

2.5 Parametric Form


For x2 + y2 = r2, parametric co-ordinates of any point on the circle is given by (r cosθ, r sinθ), (0 ≤ θ < 2π).
(a) The parametric co-ordinates of a point on the circle (x – h)2 + (y – k)2 = r2 is given by (h + r cosθ, k + r sin θ),
(0 ≤ θ < 2π).
(b) The parametric co-ordinates of any point on the circle x2 + y2 + 2gx + 2fy + c = 0 are x = –g + r cos θ and
y = –f + r sinθ, (where r = g2 + f 2 − c , and 0 ≤ θ < 2π)

2.6 Equation of Circle under Special Conditions


(a) The equation of the circle through three points A(x1, y1), B(x2, y2), C(x3, y3) is

x2 + y 2 x y 1
x12 + y12 x1 y1 1
=0
x22 + y 22 x2 y2 1
x32 + y 32 x3 y3 1

The concept of family of circles can be used to derive this form.


Taking any two of the given three points as extremities of diameter, we get the equation of a circle S = 0 and the
equation of straight line passing through these two points L = 0. Then the equation of circle passing through the
intersection of circle and line is S + λL = 0, where λ is a parameter. The value of λ can be found by substituting the
third point in the equation as it also lies on the circle.
(b) Equation of circle circumscribing the triangle formed by the lines ar x + br y + cr =
0 where r = 1, 2, 3 is

a12 + b12
a1 b1
a1 x + b1 y + c1
a22 + b22
a2 b2 = 0
a2 x + b2 y + c2
a32 + b32
a3 b3
a3 x + b3 y + c3

PLANCESS CONCEPTS

Whenever the problem seems to be very complicated using formulas and geometrical approach, then
try to apply trigonometric approach as well. Like the given circle may be in circle or ex-circle of some
triangle. May be using properties of triangle we can solve it.
Vaibhav Gupta (JEE 2009, AIR 54)
9 . 4 | Circle

Illustration 1: Find the equation of the circle which passes through the point of intersection of the lines x – 4y – 1 = 0
and 4x + y – 21 = 0 and whose centre is (2, –3). (JEE MAIN)

Sol: By solving given equation of lines simultaneously we will get point of intersection of lines i.e. P. Therefore
using distance formula we will get radius of circle and using centre and radius we will get required equation.
Let P be the point of intersection of the lines
x – 4y – 1 = 0  …. (i)
and 4x + y – 21 = 0  …. (ii)
( ) ( )
From (i) and (ii), we get x = 5 , y = 1 . So, coordinates of P are 5,1 .Let C 2, −3 be the centre of the circle.
Since the circle passes though P, therefore

(5 − 2 ) + (1 + 3 )
2 2
CP= radius ⇒ = radius ⇒ radius = 5

Hence the equation of the required circle is (x – 2)2 + (y + 3)2 = 25.

Illustration 2: Find the equation of a circle of radius 10 whose centre lies on x-axis and passes through the point
(4, 6).  (JEE MAIN)

Sol: Consider coordinates of centre of circle as (a, 0). Now Y


by using distance formula, we can calculate the value
of ‘a’ and then by using central form, we get required
equation.
A(4,6)
Here centre C lies on x-axis, and the circle passes through
A (4,6).
10
Let C be (a,0)
X’ X
∴ CA = radius C2 O C1

(a − 4 ) + (0 − 6)
2 2
⇒ CA =10 ⇒ = 10
⇒ (a – 4)2 + 36 = 100 ⇒ (a – 4)2 = 64
⇒ a – 4 = ±8 ⇒ a = 12 or a = – 4.
Thus, the coordinates of the centre are (12, 0) or − 4,0 . ( )
Hence, the equations of the required circles are Y’
Figure 9.5
( x − 12) + ( y − 0 ) 102 and ( x + 4 ) + ( y − 0 ) =
2 2 2 2
= 102

Illustration 3: Find the equation of the circle concentric with the circle x2 + y 2 + 4x + 6y + 11 =
0 and passing
( )
through the point 5, 4 . (JEE MAIN)

Sol: Since both circle are concentric therefore their centre should be same. (5,4)
Hence equation of a required circle can be written as x2 + y 2 + 4x + 6y +
(constant term) = 0.
Let the equation of the concentric circle be
x2 + y2 + 4x + 6y + k = 0
(-2,-3)
Since the point (5,4) lies on this circle,
∴ (5)2 + (4)2 + 4(5) + 6(4) + k = 0
⇒ 25 + 16 + 20 + 24 + k = 0 ⇒ k = –85
Therefore, the equation of the required circle is
x2 + y2 + 4x + 6y – 85 = 0 Figure 9.6
M a them a tics | 9.5

Illustration 4: Find the equation of a circle passing through the origin and making intercepts 4 and 3 on the y and
x axis respectively. (JEE MAIN)

Sol: By observing the problem we conclude that given intercepts are end Y
(0,4)
points of diameter of this circle. Therefore by using diametric form we can
P
obtain the equation of circle.

Let the intercepts be OP = 4, OQ = 5


∴ The co-ordinates of P and Q are (0, 4) and (3, 0) respectively.
Since POQ = 90°, hence PQ is a diameter. X’ X
O Q(3,0)
∴ The required equation of the circle is
(x − 3)(x − 0) + (y − 0)(y − 4) =
0 ⇒ 0
x2 + y 2 − 3x − 4y = Y’
Figure 9.7

(
Illustration 5: Find the equation of the circle which is circumscribed about the triangle whose vertices −2,3 , 5,2 )( )
(
and 6, −1 . ) (JEE ADVANCED)

Sol: Consider (a, b) as the centre of circle and r as the radius. As circle passes from given vertices, therefore their
distance from the centre are same. Therefore by using distance formula, we will get the value of a, b and r.
( )( )
Since the circle passes through the points −2,3 , 5,2 and 6, −1 . ( )
∴ (–2 – a) + (3 – b) = r ⇒ a + 4a + 4 + b – 6b + 9 = r 
2 2 2 2 2 2
…. (i)
(5 – a)2 + (2 – b)2 = r2 ⇒ a2 – 10a + 25 + b2 – 4b + 4 = r2  …. (ii)
(6 – a)2 + (–1 – b)2 = r2 ⇒ a2 – 12a + 36 + b2 + 2b + 1 = r2 …. (iii)
Subtracting (ii) from (i), we have
14a – 21 – 2b + 5 = 0 i.e., 14a – 2b = 16 …. (iv)
Subtracting (iii) from (ii), we get
2a – 11 – 6b + 3 = 0 ⇒ 2a – 6b = 8  …. (v)
Solving (iv) and (v), we get
a = 1 and b = –1
Putting the values of a = 1 and b = −1 in (i), we get
1 + 4 + 4 + 1 + 6 + 9 = r2 ⇒ 25 = r2 ⇒r=5
Thus, the required equation of the circle is
(x – 1)2 + (y + 1)2 = 25 ⇒ x2 + 2x + 1 + y2 + 1 + 2y = 25 ⇒ x2 + y2 – 2x + 2y – 23 = 0
9 . 6 | Circle

3. EQUATION OF CIRCLE IN SOME SPECIAL CASES


Equation Centre/Radius Properties Figures
(a) x2 + y2 = a2 ( 0, 0 ) ; a When the centre of the circle Y
coincides with the origin
center = (0,0)

X’ a X
O

Y’

Figure 9.8
(b) ( x – h )2 + ( y ± a )2 = a2 ( h , ±a ) ; a Touches x axis only, y Y
coordinate of centre = ±a

C1 (h,a)

X’ O X
G

C2 (h,-a)

Y’

Figure 9.9
(c) ( x ± a )2 + ( y – k )2 = a2 ( ±a, k ) ; a Touches y–axis only, x Y
coordinate of centre = ±a

(-a,k) (a,k)
C1 C2

X’ X
O

Y’

Figure 9.10
M a them a tics | 9.7

Equation Centre/Radius Properties Figures


(d) (x ± a )2 + (y ± a)2 = a2 ( ±a, ±a ) ; a Touches both the axes Y
depending on the quadrant
center = ( ±a , ±a)

C2 C1
(-a,a) (a,a)

X’ X
O
(-a,-a) (a,-a)

C3 C4

Y’

Figure 9.11
(e) x2 + y2 – 2ax = 0 C (a, 0) ; a When the circle passes Y
through the origin and
centre lies on x axis

X’ X
O (a,0)

Figure 9.12
(f) x2 + y2 – 2ay = 0 C (0, a); a When the circle passes Y
through the origin and
centre lies on y axis.

(0,a)

X’ X
O

Y’

Figure 9.13
9 . 8 | Circle

Equation Centre/Radius Properties Figures


(g) x2 + y2 –αx –βy =0 α β Passes through (0, 0) and has Y’
 , ; intercepts α and β on x &
 2 2 y axes respectively.

1
α2 + β2 , 
2 2 2


X’ X
O

Y’

Figure 9.14
(h, h); h When circle touches both
( ) + ( y − h)
2 2
(h) x − h h2 or
=
the axes
x2 + y 2 − 2hx − 2hy + h2 =
0
(h,h)

Figure 9.15

Y
Illustration 6: Find the equation of the circle which passes through two points on
the x–axis which are at distances 12 from the origin and whose radius is 13.
C
Sol: There are two circles which passes through two points A and A’ on x–axis which
are at a distance 12 from the origin and whose radius is 13. The centre of these circles X’
A’ O A
X
lie on y–axis (perpendicular bisector of chord AA’)
In ∆AOC, AC2 = OA2 + OC2
⇒ 132 = 122 + OC2 ⇒ OC = 5
So the coordinates of the centre of the required circles are (0, 5) and C’ (0, –5). Hence Y’
the equations of the required circles are Figure 9.16
(x – 0 ) + (y ± 5 ) = 13 ⇒ x + y ± 10y – 144 = 0.
2 2 2 2 2

4. INTERCEPTS ON THE AXES Y

The lengths of intercepts made by the circle x2 + y2 +2gx + 2fy + c = 0 on X and Y axes
are 2 g2 − c and 2 f 2 − c respectively.
Therefore,
(a) The circle x2 + y2 +2gx + 2fy + c = 0 cuts the x–axis in real and distinct points, X’
O
X
touches or does not meet in real points according as g2 >c- Distinct points
Y’
g2 = c – Touches
Figure 9.17: Intercept made
g2 < c – Does not meet by circle on x-axis

(b) Similarly, the circle x2 + y2 +2gx + 2fy + c = 0 cuts the y–axis in real and distinct points, touches or does not
meet in real points according as f2 >, = or < c .
M a them a tics | 9.9

(-g,-f)

X’ X
O

Y’

Figure 9.18: Intercept made by circle on y-axis

Illustration 7: Find the equation to the circle which touches the positive axis of y at a distance 4 from the origin
and cuts off an intercept of 6 from the axis of x. (JEE MAIN)
Sol: As circle touches Y axis therefore Y coordinate of centre of circle is 4 so by Y
using formula of intercept we will get the value of X coordinate of centre of circle
and c.
Consider a circle x2 + y2 + 2gx + 2fy + c = 0. (5,4)
This meets the axis of y in points given by y2 + 2fy + c = 0 5
4
3
The roots of this equation must be each equal to 4, so that it must be equivalent X’
O
X
to (y – 4)2 = 0 ⇒ 2f = –8 & c = 16
∵ Intercept made on the x-axis = 6
Y’
Figure 9.19
⇒ 6 = 2 g2 − 16 ⇒ g = ±5.
Hence, the required equation is x2 + y2 ± 10x – 8y + 16 = 0.

5. POSITION OF A POINT W.R.T A CIRCLE


(a) If CP < radius, then the point P lies inside the circle. (Refer Fig. 9.20 (i))

r
C

(i)
Figure 9.20 (i)

(b) If CP = radius, then the point P lies on the circumference.

P
r
C

(ii)
Figure 9.20 (ii)
9 .10 | Circle

(c) CP > radius, then the point P lies outside the circle.
P

r
C

Figure 9.20 (iii)


(iii)

Hence, any point (x, y) lies outside, on or inside if

(x1 + g)2 + (y1 + f)2 > = < g2 + f 2 − c ⇒ (x1 + g2) + (y1 + f )2 >

= < (g2 + f2 – c )
⇒ x12 + y12 + 2gx1 + 2fy1 + c > = < 0
Or, S1 > = < 0 where S1 = x12 + y12 + 2gx1 + 2fy1 + c
Therefore, a point (x1, y1) lies outside, on or inside a circle
S ≡ x2 + y2 + 2gx + 2fy + c = 0 according as S1 ≡ x12 + y12 + 2gx1 + 2fy1+ c is positive, zero or negative.

5.1 Power of a Point w.r.t. a circle


Let P ( x1 , y1 ) be a point and a secant (a line which cuts the curve in two point) PAB is drawn.
T
B
A

P (x1,y1)

C
D
Figure 9.21

The power of P(x1, y1) w.r.t. S = x2 + y2 + 2gx + 2fy + c = 0 is equal to PA.PB which is S1, where S1 = x12 + y12 + 2gx1 + 2fy1 + c .
Power remains constant for the circle i.e. independent of A and B
PA×PB = PC×PD = PT2 = square of the length of a tangent

5.2 The Least and Greatest distance of a Point from a Circle


Let S = 0 be a circle and P ( x1 , y1 ) be a point. If the diameter of the circle through P
P
intersect the circle at Q and R, Q
then QP = PC – r= least distance; and C
PR = PC + r = greatest distance R

where ‘r’ is the radius and C is the centre of the circle.


(Refer Fig. 9.23)
Figure 9.22
M a them a ti c s | 9.11

Illustration 8: The coordinates of the point on the circle x2 + y2 – 2x – 4y – 11 = 0 farthest from the origin are
 (JEE MAIN)
 8 4   4 8   8 4 
(A)  2 + ,1+  (B)  1 + , 2+  (C)  1 + , 2+  (D) None of these
 5 5  5 5  5 5

Sol: (B) The required point lies on the normal to circle through the origin, i.e. on the line 2x = y. Therefore by
substituting y = 2x in above equation of circle we will get coordinates of required point.
4  4 
x2 + 4x2 – 2x – 8x – 11 = 0 ⇒ 5x2 – 10x – 11 = 0 ⇒ x = 1 ± and y = 2  1 ± 
5  5
 4 8 
and the required point farthest from the origin is  1 + , 2+ .
 5 5

Illustration 9: The point (1, 3) is inside the circle S whose equation is of the form x2 + y2 – 6x – 10y + k = 0, k
being an arbitrary constant. Find the possible values of k if the circle S neither touches the axes nor cuts them.
 (JEE ADVANCED)

Sol: As (1, 3) lies inside the circle S therefore S1 < 0 and it does not touches x and y axes. On the basis of this we
can solve the problem and will get range of k.
12 + 32 – 6×1 –10×3 + k < 0; ∴ k < 26  …(i)
Solving y = 0 and x + y – 6x – 10y + k = 0, we get x – 6x + k = 0
2 2 2

Since the circle S does not intersect with the x-axis,


⇒ discriminant < 0 i.e., 36 – 4k < 0 ⇒ k > 9  …(ii)
Solving x = 0 and x2 + y2 – 6x – 10y + k = 0, we get y2 – 10y + k = 0
Since the circle S does not intersect with the y-axis,
⇒ discriminant < 0 i.e., 100 – 4k < 0 ⇒ k > 25  …(iii)
From (i), (ii) and (iii), we get 25 < k < 26, i.e., k ∈ (25, 26).

6. LINE AND A CIRCLE


a2 (1 + m2 ) − c2
The length of the intercept cut off from the line y = mx + c by the circle x2 + y2 = a2 is 2 .
1 + m2
(a) If a2 (1 + m2 ) > c2, or, | c | < a 1 + m2

i.e., the line will intersects the circle at two real and different points.

(b) If a2 (1 + m2 ) = c2, | c | a 1 + m2
or, =

i.e., the line will touch the circle at only one point i.e. the line will be a tangent. (2,4)

(c) If a2 (1 + m2 ) < c2, or, | c | > a 1 + m2

i.e., the line will meet the circle at two imaginary points.

Figure 9.23

Illustration 10: Show that the line 3x – 4y – c = 0 will meet the circle having centre at (2, 4) and the radius 5 in real
and distinct points if – 40 < c < 20.  (JEE MAIN)

Sol: Since the line cuts the circle so length of perpendicular from centre of circle upon line is less than the radius
of circle.
9 .12 | Circle

3× 2 − 4 × 4 − c
<6 ⇒ 10 + c < 30
9 + 16
⇒ –30 < 10 +c < 30 ⇒ –40 < c < 20

Illustration 11: If 4l2 – 5m2 + 6l +1 = 0 then show that the line lx + my +1 = 0 touches a fixed circle. Find the radius
and centre of the circle. (JEE ADVANCED)

Sol: If line touches the circle then perpendicular distance from centre of circle to the line is equal to the radius of
circle so by using distance formula of point to line we will get one equation and other is given 4l2 – 5m2 + 6l +1 =
0 . hence by solving these two equation we will get required answer.
Let the circle be (x – α)2 + (y – β)2 = a2
The line lx + my + 1 = 0 touches the circle if
l α + mβ + 1
a= or a2(l 2 + m2) = (lα + mβ +1)2
2 2
l +m
or (a2 – α2) l2 + (a2 – β2) m2 –2 lβα m – 2 l α – 2mβ – 1 = 0 ... (i)
But 4l – 5 m + 6l + 1 = 0
2 2
... (ii)
It is possible to find α,β, a if (i) and (ii) are identical.
The condition is
a2 − α2 a2 − β2 −2α 2β −1
= = = =  ... (iii)
4 −5 6 0 1
a2 − α2
∴ β = 0, α =3 and = –1 which implies a2 – 32 = –4, i.e., a = 5 .Also α = 3, β = 0, a = 5 satisfies equation
4
( )
2
(iii) and hence the line touches the fixed circle (x – 3 )2 +(y – 0)2 = 5

or x2 + y2 – 6x + 4 = 0, whose centre = (α, β) = (3, 0) and radius = a = 5.

Illustration 12: Find equation of a line with slope gradient 1 and such that x2 + y2 = 4 and x2 + y2 – 10x – 14y + 65 = 0
intercept equal length on it ? (JEE ADVANCED)

Sol: As given slope of line is 1, therefore its equation will be y = x + c Hence by using perpendicular distance
formula we will get distance of line from centre of respective circle and then
by using Pythagoras we can obtain length of intercepts made by line to these
A’
circles and which are equal. Therefore we can obtain value of c and required
equation of circle.
C(5,7)
Let 2 be the length of the intercept made by the two circle.
For x2 + y2 = 4, centre ≡ (0,0) and radius = 2, and A
For x2 + y2 – 10x – 14y + 65 = 0, centre ≡ (5,7) and
O(0,0)
radius = 3.
c 5−7+c c−2
∴ OA = and CA’ = ⇒CA’ = Figure: 9.24
2 2 2

Now, from the diagram we get

4 – OA2 = 2  …(i)
M a them a ti c s | 9.13

and 9 – CA’2 = 2  …(ii)


2 2
c c−2
⇒4– =9–   [from (i) and (ii)]
2  2 
3
⇒c=–
2
The equation of line is 2x - 2y = 3

Illustration 13: Find the values of α for which the point (2α, α +1) is an interior point of the larger segment of the
circle x2 + y2 – 2x –2y – 8 = 0 made by the chord whose equation is x – y + 1 = 0. (JEE ADVANCED)

Sol: As point (2α, α +1) lies inside the circle S, therefore S1 < 0. Hence by substituting
the point in the equation, we will get the range of α and as it lies in larger segment =0
+1
made by line x – y + 1 = 0. The centre of circle i.e. (1, 1) and (2α, α +1) will have the x-y
same sign.
∴ (2α)2 + (α +1)2 – 2×2α – 2(α + 1) – 8 < 0
Centre
⇒ 5a2 – 4α – 9 < 0 or (5α – 9)(α + 1) < 0
(2,+1)
⇒ –1 < α <9/5  ... (i)
Also, as the point lies in the larger segment, the centre (1, 1) and the point (2α, α + 1) Figure 9.25
must be on the same side of the line x – y + 1 = 0.
Clearly, 1 – 1 + 1 > 0; So, 2α – (α + 1) + 1 > 0;
∴ α > 0 … (ii)
 9
∴ The set of values of α satisfying (i) and (ii) is  0,  .
 5

7. TANGENTS

7.1. Point Form


(a) The equation of tangent at (x1 ,y1) to circle x2 + y2 = a2 is xx1 + yy1 - a2 = 0.

2 2 2
x +y=a
P(x1,y1)

Figure 9.26

(b) The equation of tangent at (x1, y1) to circle x2 + y2 + 2gx + 2fy + c = 0 is xx1 + yy1 + g(x + x1) + f(y + y1) + c = 0.

7.2. Parametric Form


Since parametric co-ordinates of a point on the circle x2 + y2 = a2 is (a cos θ, a sin θ), then equation of tangent at (a
cos θ, a sin θ) is x a cos θ + y a sin θ =a2 or, x cos θ + y sin θ = a.
9 .14 | Circle

7.3. Condition for Tangency


A line L = 0 touches the circle S = 0. If length of perpendicular (p) drawn from the centre of the circle to the line
is equal to radius of the circle i.e. p = r. This is the condition of tangency for the line L = 0. Circle x2 + y2 = a2 will
touches the line y = mx + c if c = ±a 1 + m2

Illustration 14: For what value of c will the line y = 2x + c be a tangent to the circle x2 + y2 = 5.  (JEE MAIN)

Sol: The equation of the tangent to the circle x2 + y2 = a2 in slope form is y = mx + a 1 + m2 .


On comparison, we get a = 5 and m = 2.

∴c=± 5 × 1 + 22 ⇒ c = ± 5

The required equation is =


y 2x ± 5

7.4. Slope Form


The straight line y = mx + c touches the circle x2 + y2 = a2 if c2 = a2 (1 + m2). Therefore, the equation of the tangent
 ma ±a 
in the slope form is y = mx ± a 1 + m2 and the point of contact is  ,  .
 2 
 1+m 1 + m2 
7.5 Length of Tangent

The length of the tangent drawn from a point P(x1, y1 ) to the circle S =x2 + y2 + 2gx +2fy + c = 0 is PT1 = PT2 =

x12 + y12 + 2gx1 + 2fy1 + c


∴ Length of Tangent = S1

Note:
(i) P
 T2 is called the power of the point with respect to a given circle, where PT is the tangent from a point P to a
given circle.
(ii) Area of quadrilateral PT1C T2 = 2 × (Area of triangle PT1C), and

 r 
 
(iii) The angle between tangents PT1 and PT2 is equal to 2tan–1 
P(x1,y1)
S .
 1

Illustration 15: If OA and OB are tangents from the origin O, to the circle x2 + y2 +2gx +2fy
 
+ c= 0, c > 0 and C is the centre of the circle, then area of the quadrilateral OACB is 
 (JEE MAIN)

(A)
1
2
( )
c g2 + f 2 − c (B) ( )
c g2 + f 2 − c

g2 + f 2 − c
(C) c g2 + f 2 − c (D)
c
T1
Sol: (B) As we know quadrilateral OACB is formed by two right angle triangle OAC and r
C
T3
triangle OBC. Line OA and OB are tangent to the circle from common point O. Therefore
OA = OB and (AC = CB) radius of circle, hence both triangle are equal. Therefore Area of
the quadrilateral OACB = 2 Area of the triangle OAC. 2 2
x +y -2gx+2fc c=0
OA = OB = S1 = c (Length of the tangent from the origin) Figure 9.27
M a them a ti c s | 9.15

and, CA = CB = g2 + f 2 − c (Radius of the circle)

∴ Area of the quadrilateral OACB = 2 Area of the triangle OAC C


1 B
= 2 ×   OA × CA = c g2 + f 2 − c A
2

Illustration 16: The locus of a point which moves such that the tangents from it to the
two circles x2 + y2 – 5x – 3 = 0 and 3x2 + 3y2 + 2x + 4y – 6 =0 are equal, is (JEE MAIN)
(A) 7x + 4y – 3 = 0 (B) 17x + 4y + 3 = 0 (C) 3x – 4y + 9 = 0 (D) 13x – 4y + 15 =0

Sol: (B) Use the formula for length of tangent. Let P(h, k) be any point on the locus.

The length of the tangent from P to the first circle is h2 + k 2 − 5h − 3 ,  …(i)

2 4 6
Similarly, the length of the tangent to the other circle is h2 + k 2 + h + k − . …(ii) O (0,0)
3 3 3
On equating (i) and (ii), we get 17h + 4k + 3 = 0, Figure 9.28

Therefore, the required locus is 17x + 4y + 3 = 0.

7.6 Pair of Tangents


From a given point P(x1, y1) two tangents PA and PB can be drawn to the circle S = x2 + y2 +2gx +2fy + c = 0.
The combined equation of the pair of tangents is A
SS1 = T2, where
S = 0 is the equation of circle,
T = 0 is the equation of tangent at (x1, y1), and
S1 = x12 + y12 +2gx1 +2fy1 + c
(S1 is obtained by replacing x by x1 and y by y1 in S) P(x1,y1) B

Pair of tangents from point (0, 0) to the circle are at right angles if g2 + Figure 9.29
f2 = 2c.

Illustration 17: Find the equation of the pair of the tangents drawn to the circle x2 + y2 – 2x + 4y = 0 from the
point (0, 1). (JEE MAIN)

Sol: Here (x1, y1) = (0, 1). So by using formula SS1 = T2 we can get required equation, where S = x2 + y 2 − 2x + 4y =
0,
S1 ≡ x12 + y12 +2gx1 +2fy1 + c and T = xx1 + yy1 – (x+x1) +2(y+x1).
0
Given circle is S ≡ x2 + y 2 − 2x + 4y = …(i)
Let P be the point (0, 1).
∴ S1 ≡ x12 + y12 +2gx1 +2fy1 + c ⇒ S1 ≡ 02 + 12 – 2.0 + 4.1 = 5
And, T ≡ xx1 + yy1 – (x+x1) +2(y+x1) ⇒ T ≡ x(0) + y(1) – (x + 0) + 2(y + 1)
i.e., T ≡ – x + 3 y + 2.
Hence, the equation of pair of tangents from P (0, 1) to the given circle is SS1 = T2

( )
i.e. 5 x2 + y 2 − 2x + 4y = ( −x + 3y + 2 )
2

⇒ 5x2 + 5y2 – 10x + 20y = x2 + 9y2 + 4 – 6xy – 4x + 12y


9 .16 | Circle

⇒ 4x2 – 4y2 – 6x + 8y + 6xy – 4 = 0


⇒ 2x2 – 2y2 + 3xy – 3x + 4y – 2 = 0  …(ii)
Note: From (ii), we have 2x2 + 3(y – 1) x – (2y2 – 4y + 2) = 0.
This is a quadratic equation in x, hence by using quadratic formula we get

3(y − 1) ± 9(y − 1)2 + 8(2y 2 − 4y + 2)


x= or, 4x – 3y + 3 = ± 25y 2 − 50y + 25
4
or, 4x – 3y + 3 = ±5(y – 1).
∴ Separate equations of tangents are x – 2y + 2 = 0 and 2x + y - 1 = 0.

Illustration 18: From a point on the line 4x – 3y = 6 tangents are drawn to the circle x2 + y2 – 6x –4y + 4 = 0 which
24
make an angle of tan–1 between them. Find the coordinates of all such points and the equations of tangents.
 7 (JEE ADVANCED)

Sol: Consider a point P on the line 4x – 3y = 6 and use the formula.


A
Let P (x1 , y1) be a point on the line 4x – 3y = 6.
24
If θ is the angle between the tangents, then tan θ = . P(x1,y1) 
7
C
For the given circle, Centre C = (3, 2) and
Radius = CA = 32 + 22 − 4 = 3 for tan θ
B
∴ The length of tangent, PA = S1 = x12 + y12 − 6x1 − 4y1 + 4
Figure 9.30
θ AC 3
∴ tan = =
2 PA S1
θ 9
1 − tan2 2 2 1−
S1 − 9 7
⇒ tan2
θ
=
9
or, 2 = x1 + y1 − 6x1 − 4y1 + 4 − 9 or −S1
= = ⇒ S1 = 16
2 S1 2 θ x 2
+ y 2
− 6x − 4y + 4 + 9 9 S1 + 9 25
1 + tan 1 1 1 1 1+
2 S1

7 x12 + y12 − 6x1 − 4y1 − 5  24 


∴ = 2  tan θ = 
25 x1 + y12 − 6x1 − 4y1 + 13  7 

or, x12 + y12 – 6x1 – 4y1 – 12 = 0  … (i)


As (x1, y1) is on the line 4x – 3y = 6, we get 4x1 – 3y1 = 6 … (ii)
Solving (i) and (ii), we get
2
 4x − 6   4x − 6 
x12
+  1  – 6x1 – 4  1  –12 = 0
 3   3 
⇒ 9x12 + (4x1 – 6)2 – 54x1 – 12(4x1– 6) – 108 = 0
⇒ 25x12 – 150x1 = 0 ⇒ x1 (x1 – 6) = 0
4x1 − 6 6 18
⇒ x1 = 0, 6 and, y1 = =– , = –2, 6.
3 3 3
∴(x1, y1) ≡ (0, –2) and (6, 6).
The equation of the pair of tangents is given by SS1 =T2
where S ≡ x2 + y2 – 6x – 4y + 4,
S1 = x12 + y12 – 6x1 – 4y1 + 4, and
M a them a ti c s | 9.17

T = xx1 + yy1 – 3(x + x1) – 2(y + y1) + 4


∴ The pair of tangents from (0, –2) is
(x2 + y2 – 6x – 4y + 4)⋅(0 + 4 – 0 + 8 + 4) = (0 + y(–2)– 3(x) –2(y – 2) + 4)2
⇒ 16(x2 + y2 – 6x –4y + 4) = (–3x –4y + 8)2
⇒ 16(x2 + y2 – 6x –4y + 4) = 9x2 + 16y2 + 64 +24xy – 48x – 64y
⇒ 7x2 – 24xy –48x = 0 ⇒ x(7x – 24y – 48) = 0
∴ The tangents from (0, –2) are x = 0, and 7x –24y –48 =0.
Similarly, the equation of the pair of tangents from (6, 6) is
(x2 + y2 – 6x – 4y + 4) ⋅ (36 + 36 – 6 ⋅ 6 – 4 ⋅ 6 + 4) = {x ⋅ 6 + y ⋅ 6 – 3(x + 6) – 2(y + 6) + 4}2
⇒ 16(x2 + y2 – 6x – 4y + 4) = (3x + 4y – 26)2 = 9x2 + 16y2 + 676 + 24xy – 156x – 208y
⇒ 7x2 – 24xy + 60x + 144y – 612 = 0
⇒ (7x – 24y + 102) (x – 6) = 0
∴ The tangents from (6, 6)are x – 6 = 0, and 7x – 24y + 102 =0.

Illustration 19: Obtain the locus of the point of intersection of the tangents to the circle x2 + y2 = a2 which include
an angle α. (JEE ADVANCED)

Sol: Consider (x1, y1) as the point of intersection of tangents to the given circle (x1,y1)

α a
and then use tan = to get the desired result.
2 S 1
O
Let (x1, y1) be the point of intersection of a pair of tangents to the given circle.
If the pair of straight lines includes an angle α, then
a
2 2
x +y =a
2 2

α a S1
⇒ tan = ⇒ tan α = Figure 9.31
2 S1 a2
1−
S1
2a x12 + y12 − a2
⇒ tan α =
y12 + x12 − 2a2

(
⇒ x12 + y12 − 2a2 )
2
( 2 2
tan2 α = 4a2 x1 + y1 − a
2
)
Hence, the required locus is (x2 + y2 – 2a2)2 tan2 α =4a2 (x2 + y2 – a2).

7.7 Director Circle


The locus of the point of intersection of two perpendicular tangents to a circle is called
the Director circle.
For the circle x2 + y2 = a2, the equation of the director circle is x2 + y2 = 2a2. P
Hence, the centre of the director circle is same as the centre of the given circle, and the
radius is 2 times the radius of the given circle.
General Form: For the general form of the circle x2 + y2 + 2gx + 2fy + c = 0, the
equation of the director circle is given by x2 + y2 + 2gx + 2fy + 2c – g2 – f2 = 0. Figure 9.32
9 .18 | Circle

Illustration 20: Find the equation of the director circle of the circle (x – 2)2 + (y + 1)2 = 2.  (JEE MAIN)

Sol: As we know, for the circle x2 + y2 = a2, the equation of the director circle is x2 + y2 = 2a2 .
For the given circle, Centre ≡(2, –1) & Radius = 2.
∴ The centre of the director circle ≡ (2, – 1), and the radius of the director circle = 2 × 2 = 2.
∴ The required equation is (x – 2)2 + (y + 1)2 = 4.

8. NORMALS
The normal of a circle at any point is a straight line, perpendicular to
the tangent and passing through the centre of the circle.
(a) Equation of normal: The equation of normal to the general l
form of the circle x2 + y2 + 2gx + 2fy + c = 0 at any point rma C
No
(x1, y1) on the circle is P
y1 + f x − x1 y − y1
(y – y1) = (x – x1) or, =
x1 + g x1 + g y1 + f
Tangent
The equation of normal to the circle x2 + y2 = a2 at any point (x1 ,y1) is
x y Figure 9.33
xy1 – xy1 = 0 or, = .
x1 y1
x y
(b) Parametric Form: Equation of normal at (a cos θ, a sin θ) to the circle x2 + y2 = a2 is =
acos θ asinθ
x y
or, = or, y = x tan θ or, y = mx (where m = tan θ).
cos θ sin θ

Illustration 21: Find the equation of the circle having the pair of lines x2 + 2xy + 3x + 6y = 0 as its normal and
having the size just sufficient to contain the circle x(x – 4) + y(y – 3) = 0. (JEE ADVANCED)

Sol: By solving equation x2 + 2xy + 3x + 6y = 0 we will get point of intersection of normals i.e. centre of required
circle. As given circle x(x – 4) + y(y – 3) = 0 lies inside the required circle hence distance between centres will be
equal to the difference between their radius, therefore we can find out radius of required circle by using distance
formula.
Given the equation of pair of normal is x(x + 3) + 2y(x + 3) = 0
⇒ (x + 3)(x + 2y) = 0
∴ Either (x+3)=0 …(i) or (x+2y)=0  …(ii)
3
On solving (i) and (ii), we get x = −3 and y =
2 (-3, 3/2) (2, 3/2)
 3
∴ The centre ≡  −3,  (The point of intersection of the normals).
 2
For the circle x2 + y2 – 4x – 3y = 0 …(iii)
2
 3  −3  5
( −2)
2 Figure 9.34
centre =  2,  and radius, r = +   − 0 = .
 2  2  2
If the circle x2 + y2 – 4x – 3y = 0 lies inside another circle of radius ‘a’, then
 3  3
a – r = distance between the centres  −3,  and  2, 
 2   2
2
5 5 15
( −3 − 2) +  32 − 32 
2  
⇒a– = ⇒a=5+ ∴a= .
2   2 2
M a them a ti c s | 9.19

2 2
 3  15 
Hence, the equation of the circle is (x + 3) +  y −  =   or, x2 + y2 + 6x – 3y = 45.
2
 2  2 

9. CHORD OF CONTACT (x’,y’)Q

Consider a point P(x1, y1) lying outside the circle. Tangents are drawn to touch the
given circle at Q and R respectively (as shown in the diagram). The chord joining P
Chord of
the points of contact of the two tangents to a circle (or any conic) from the point (x1, y1)
contact
P, outside it, is known as the chord of contact.
(x”, y”)R
9.1 Equation of Chord of contact Figure 9.35

The equation of the chord of contact of tangents drawn from a point (x1 , y1) to
the circle x2 + y2 = a2 is xx1 + yy1 = a2. Equation of chord of contact at (x1, y1) to the circle x2 + y2 + 2gx + 2fy + c = 0
is xx1 + yy1 + g(x + x1) + f(y + y1) + c =0 .
Clearly, the equation of the chord of contact coincides with the equation of the tangent.

Length of chord of contact


Consider a circle of radius ‘r’ and the length of perpendicular from the centre to the chord of contact be ‘p’, then
the length of the chord, QR = 2 r 2 − p2 .

PLANCESS CONCEPTS

( )
3/2
1 a x12 + y12 − a2 RL3
• Area of ∆PQR = × PM × QR = =
2 x12 + y12 R 2 + L2

where, the length of the tangent, L = x12 + y12 − a2 and,
radius of circle, R = a.

 quation of circle circumscribing the triangle PQR is


• E
(x – x1) (x + g) + (y – y1) (y + f) = 0.
Note: Circumscribing Circle also passes through centre Figure 9.36
of original Circle
Vaibhav Krishnan (JEE 2009, AIR 22)

Illustration 22: Find the equation of the chord of contact of the tangents drawn from (1, 2) to the circle x2 + y2 – 2x
+ 4y + 7 = 0. (JEE MAIN)

Sol: Equation of chord of contact is T = 0


Given circle is S ≡ x2 + y2 – 2x + 4y + 7 = 0 …(i)
For point P ≡ (1, 2),
S1 > 0, ⇒ the point P lies outside the circle. and, T ≡ x(1) + y(2) – (x + 1) + 2(y + 2) + 7 i.e. T ≡ 4y + 10
∴ The equation of the chord of contact is T = 0 i.e. 2y + 5 = 0.
9 .20 | Circle

Illustration 23: The locus of the point of intersection of the tangents at the extremities of a chord of the circle x2
+ y2 = a2 which touches the circle x2 + y2 – 2ax = 0 passes through the point (JEE ADVANCED)
a   a
(A)  , 0  (B)  0,  (C) (0, a) (D) (a, 0)
2   2

Sol: (A) and (C) Apply the condition of tangency to the equation of chord of contact.
Let P (h, k) be the point of intersection of the tangents at the extremities of the chord AB of the circle x2 + y2 = a2.
∴ The equation of the chord of contact AB w.r.t. the point P is hx + ky = a2.
h(a) + k(0) − a2
The line hx + ky = a2 touches the circle x2 + y2 – 2ax = 0 if =a
h2 + k 2
⇒ (h – a)2 = h2 + k2
Therefore, the locus of (h, k) is (x – a)2 = x2 + y2 or, y2 = a(a – 2x).
Clearly, points (A) and (C) satisfy the above equation.

9.2 Chord Bisected at a given Point B


P(x1,y1)
The equation of the chord of the circle A

S ≡ x + y + 2gx + 2fy + c = 0 bisected at the


2 2
C
point (x1, y1) is given by T = S1.
i.e., xx1 + yy1 + g ( x + x1 ) + f ( y + y1 ) + c =x12 + y12 + 2gx1 + 2fy1 + c .

Figure 9.37: Chord bisected by point P

PLANCESS CONCEPTS

The smallest chord of a circle passing through a point 'M' at a maximum distance from the centre is the
one whose middle point is M.
Shrikant Nagori (JEE 2009, AIR 30)

Illustration 24: Find the equation of the chord of the circle x2 + y2 + 6x + 8y – 11 = 0, whose middle point is (1, –1).
 (JEE MAIN)
Sol: Use T = S1
Given, S ≡ x2 + y2 + 6x + 8y – 11 = 0
For point L (1, 1), S1 = 12 + (–1)2 + 6.1 + 8(–1) – 11 = –11 and
T = x.1 + y.(–1) + 3(x + 1) + 4(y – 1) – 11 i.e. T = 4x + 3y – 12
Now equation of the chord having middle point, L(1, –1) is
∴ 4x + 3y – 12 = –11 ⇒ 4x + 3y – 1 = 0
Second method:
Let C be the centre of the given circle, C ≡ (–3, –4)
−4 + 1 3
∴ Slope of CL = =
−3 − 1 4
∴ Equation of chord whose middle point is L, is
M a them a ti c s | 9.21

4
∴ y + 1 = – (x – 1) [ chord is perpendicular to CL]
3
Or, 4x + 3y – 1 = 0

Illustration 25: Find the locus of the middle points of chords of given circle x2 + y2 = a2 which subtends a right
angle at the fixed point (p, q). (JEE ADVANCED)

Sol: As M(h, k)be the midpoint of the chord AB which subtends an angle of 900 at the point N (p , q) therefore a
circle can be drawn with AB as the diameter and passing through the point N, hence AM = MN.
∴ AM = MN ⇒ AM2 = MN2 ⇒a2 – (h2 + k2) = (h - p)2 + (k - q)2 A
⇒ a2 – h2 – k 2 =
h2 + p2 – 2hp + k 2 + q2 − 2kq
⇒ 2h2 + 2k 2 – 2ph − 2qk + p2 + q2 − a2 =
0 N(p,q) M(h,k)
1 2
⇒ h2 + k 2 – ph − qk +
2
(
p + q2 − a2 = )0

1
( )
B
Hence, the required locus is x2 + y 2 – px − qy + p2 + q2 − a2 =
0. Figure 9.38
2

10. COMMON CHORD OF TWO CIRCLES


Definition: The chord joining the points of intersection of two given circles is called their common chord.

Equation of common chord: The equation of the common chord of two circles
S1 ≡ x2 + y2 + 2g1x + 2f1y + c1 = 0 …. (i)
and S2 ≡ x2 + y2 + 2g2x + 2f2y + c2 = 0 …. (ii)
is given by S1 – S2 = 0 i.e., 2x(g1 – g2) + 2y (f1 – f2) + c1 – c2 = 0.

C1 M C2(2,-4)
=

Figure 9.39: Common Chord

Length of the common chord: AB = 2(AM) = 2 C1 A2 − C1M2

Where, C1A = radius of the circle S1 = 0, and


C1M = length of the perpendicular from the centre C1 to the common chord AB.

Note: If the two circles touch each other, then the length of common chord is zero and the common chord is the
common tangent to the two circles at the point of contact.
9 .22 | Circle

PLANCESS CONCEPTS

The length of the common chord AB is maximum when it is the diameter of the smallest circle.
Nitish Jhawar (JEE 2009, AIR 7)

Illustration 26: Find the equation and the length of the common chord of two circles.
2x2 + 2y2 + 7x – 5y + 2 = 0 and x2 + y2 – 4x + 8y – 18 = 0 (JEE MAIN)

Sol: Use the formula for equation of common chord and length of common chord. Equation of common chord of
circle is S1 – S2 = 0 i.e., 2x(g1 – g2) + 2y (f1 – f2) + c1 – c2 = 0 and length of common chord is 2 C2 A2 − C2M2
.
7 5
Given S1 = x + y + x – y + 1 =0 
2 2
…. (i)
2 2
S2 = x2 + y2 – 4x + 8y – 18 = 0 …. (ii)
Therefore, the equation of the common chord AB is S1 – S2 = 0
15 21
i.e. x– y + 19 = 0 ⇒ 15x – 21y + 38 = 0 .… (iii)
2 2
30 + 84 + 38 152
The length of the perpendicular from the centre C2(2, –4) to the common chord AB is C2M = =
Radius of the circle S2 = 0 is, C2A = 38 152 + 212 666

∴ The length of the common chord = AB = 2AM


2
 152  1102
= 2 C2 A2 − C2M2 ; = 2 38 −   =2
 666  333

Illustration 27: Tangents are drawn to the circle x2 + y2 = 12 at the points where it is met by the circle x2 + y2 – 5x
+ 3y – 2 = 0; find the point of intersection of these tangents.  (JEE ADVANCED)

Sol: As we know that, if (x1, y1) is a point of intersection of tangents of circle x2 + y2 = a2 then equation of chord of
contact is xx1 + yy1=a2 and the equation of common chord of two circles are S1 – S2 = 0 i.e., 2x(g1 – g2) + 2y (f1 – f2)
+ c1 – c2 = 0. By using these two formulae we can solve the problem.
Given circles are S1 ≡ x2 + y2 – 12 = 0 … (i)
and S2 ≡ x2 + y2 – 5x + 3y – 2 = 0 … (ii)
The equation of common chord is S1 – S2 = 0 i.e. 5x – 3y – 10 = 0  … (iii)
Let this line meet circle (i) at A and B, and P(α, β) be the point of intersection of the tangents at A and B. Therefore,
the equation of the chord of contact AB is xα + yβ – 12 = 0 … (iv)
As (iii) and (iv) represent the same line, therefore on comparison, we get
α β 6 18  18 
= = ∴ α = 6 and β = – . Hence, P ≡  6, − .
5 −3 5 5  5 

O
11. DIAMETER OF CIRCLE P(h,k)

The locus of the middle points of a system of parallel chords of a circle Diameter
x+my=0
is known as the diameter of the circle.
Let the equation of parallel chords be y=mx+c
Figure 9.40
M a them a ti c s | 9.23

y = mx + c (where, c is a parameter).
∴The equation of the diameter bisecting parallel chords
of the circle x2 + y2 = a2 is given by x + my = 0.

12. POLE AND POLAR


Let P (x1,y1) be any point inside or outside the circle. Passing through the point P chords AB and A’ B’ are drawn. If
the tangents at point A and point B intersect at Q (h, k) , then the locus of Q is a straight line and is called the polar
of point P with respect to circle and P is called the pole. Similarly, if the tangents to the circle at A’ and B’ meet at
Q’, then the locus of Q’ is the polar with P as its pole.
Q’ Q(h,k1) A
B’
A’

Polar
A’ Polar P(x1,y1)
B
Pole B
P(x1,y1) A
Q’ B’

Q(h,k1)

Figure 9.41(a): Polar of a point P outside the circle Figure 9.41(b): Polar of a point P inside the circle

Equation of polar of the circle x2 + y2 + 2gx + 2fy + c = 0 w.r.t. point P (x1, y1) is xx1 + yy1 + g(x + x1) + f(y + y1) + c = 0
i.e. T = 0.
If the circle is x2 + y2 = a2, then its polar w.r.t. (x1, y1) is xx1 + yy1 – a2 = 0 i.e. T = 0.
Pole of a line w.r.t. the circle x2 + y2 = a2
Consider a line lx + my + n = 0 and let (x1, y1) be the pole of the line w.r.t. the circle x2 + y2 = a2.
For the point (x1, y1),
The equation of polar w.r.t. the circle x2 + y2 = a2 is xx1 + yy1 – a2 = 0.
Since lx + my + n = 0 and xx1 + yy1 – a2 = 0 represent the same line.
x1 y1 −a2 a2l a2m
∴ = = ⇒ x1 = – and y1 = – .
l m n n n
 a2l a2m 
Hence, the pole of the line lx + my +n = 0 is  − ,− 
 n n 

Pole of a line w.r.t. the circle x2 + y2 + 2gx + 2fy + c = 0
Consider a line lx + my + n = 0.
If (x1, y1) is the pole, then the equation of polar is xx1 + yy1 + g(x + x1) + f(y + y1) + c = 0.
Now, since lx + my + n = 0 and xx1 + yy1 + g(x + x1) + f(y + y1) + c = 0 represent the same line,
x1 + g y1 + f gx1 + fy1 + c
∴ = =
l m n
x1 + g y1 + f g2 + f 2 − c
On simplification, we get = =
l m l g+ fm − n

x1 + g y1 + f r2
⇒ = = , where r is radius of the circle.
l m l g+ mf − n
9 .24 | Circle

12.1 Conjugate Points and Conjugate Lines


(a) If the polar of point P (x1 , y1) w.r.t. a circle x2 + y2 = a2, passes through Q(x2 , y2), then the polar of Q will pass
through P. Such points are called conjugate points and they satisfy the relation x1x2 + y1y2 = a2
(b) If the pole of the line l1x + m1y + n1 = 0 w.r.t. a circle lies on another line l2x + m2y + n2 = 0, then the pole of
the second line will lie on the first and such lines are said to be conjugate lines.
Consider the circle x2 + y2 = a2,
 a2l a2l 
The pole P of the line l1x + m1y + n1 = 0 w.r.t. the circle is given by  − 1 , − 1  .
 n1 n1 
 
 a2l   a2m 
⇒ l2  − 1  + m2  − 1 
+n = 0 ∴ a (l1l2 + m1m2 ) =
2
n1n2
 n1   n1  2
   

PLANCESS CONCEPTS

Points P (x1, y1) and Q (x2, y2) are conjugate points w.r.t. the circle x2 + y2 + 2gx + 2fy + c = 0 if x1x2 + y1y2
+ g (x1 + x2) + f(y1 + y2) + c = 0.
If P and Q are conjugate points w.r.t. a circle with centre at O and radius ‘a’ then PQ2 = OP2 + OQ2 – 2a2.
Shivam Agarwal (JEE 2009, AIR 27)

Illustration 28: Find the pole of the line 3x + 5y + 17 = 0 with respect to the circle x2 + y2 + 4x + 6y + 9 = 0. 
 (JEE MAIN)

Sol: If P(α, β) be the pole of line with respect to the given circle. Then the equation of polar of point P(α, β) w.r.t.
the circle is xα + yβ + 2(x + α) + 3(y + β) + 9 = 0. And this equation represent same line which is represented by
equation 3x + 5y + 17 = 0. By solving these two equation simultaneously we will get required pole.
Given circle is x2 + y2 + 4x + 6y + 9 = 0 ... (i)
and, given line is 3x + 5y + 17 = 0  … (ii
⇒ (α + 2) x + (β + 3) y + 2α + 3β + 9 = 0  ... (iii)
Since equation (ii) and (iii) represent the same line,
α+2 β + 3 2α + 3β + 9
∴ = = ⇒ 5α + 10 = 3β + 9
3 5 17
⇒ 5α – 3β = –1 ... (iv)
and, 17α + 34 = 6α + 9β + 27 ⇒ 11α – 9β = –7 ... (v)
From (iv) and (v), we get α = 1, β = 2
Hence, the pole of the line 3x + 5y + 17 = 0 w.r.t. the circle x2 + y2 + 4x + 6y + 9 = 0 is (1, 2).

Illustration 29: A variable circle is drawn to touch the axis of x at origin. Find locus of pole of straight line lx + my
+ n = 0 w.r.t. circle. (JEE ADVANCED)

Sol: As circle touches x-axis at origin therefore let (0, λ) be its centre then equation of circle will be x2 + (y – λ)2 =
λ2. Hence by considering P(x1, y1) be the pole and using polar equation we will get required result.
Let the centre of the circle be (0, λ).
Then the equation of the circle is x2 + (y – λ)2 = λ2
M a them a ti c s | 9.25

⇒ x2 + y2 – 2λy = 0.
Let P(x1, y1) be the pole of the line lx + my + n = 0 w.r.t. the circle,
then, the equation of the polar is xx1 + yy1 – λ(y + y1) = 0 (0,)
xx1 + y (–λ + y1) – λy1 = 0
x −λ + y1 −λy1
∴ On comparison, we =
get 1 = .
l m n
Figure 9.42
Hence, the locus of the pole is ly2 = mxy – xn.

Illustration 30: Prove that if two lines at right angles are conjugate w.r.t. circle then one of them passes through
centre.  (JEE ADVANCED)

Sol: Let two perpendicular lines which are conjugate to each other be
ax + by + c = 0 ... (i)
bx – ay + λ = 0  ... (ii)
∴ The equation of the polar of a point (x1, y1) is xx1 + yy1 – r2=0 … (iii)
x1 y1 −r 2
On comparing (i) and (iii), we get = = .
a b c
From the definition of conjugate lines, we know that the point (x1, y1) should satisfy the equation bx – ay + λ = 0,
−br 2a ar 2b
hence + + λ = 0 ⇒ λ = 0.
c c
Therefore, bx - ay + λ = 0 passes through (0, 0).

13. COMMON TANGENTS TO TWO CIRCLES


Different cases of intersection of two circles:
Let the two circles be (x – x1)2 + (y – y1)2 = r12 … (i) and, (x – x1)2 + (y – y2)2 = r22 … (ii)

Then following cases may arise:
Case I: When the distance between the centres is greater than the sum of radii. C1 C2 > r1 + r2

Direct common
tangent

r1
r2
P C1
C2 T

Transverse common
tangents
Figure 9.43: Common tangents for non-intersecting and non-overlapping circles

In this case four common tangents can be drawn, in which two are direct common tangents and the other two are
transverse common tangents.
9 .26 | Circle

The points P and T , the point of intersection of direct common tangents and transverse common tangents
respectively, always lie on the line joining the centres of the two circles. The point P and T divide the join of C1 and
C2 externally and internally respectively in the ratio r1 : r2.
C1P r1 C1 T r1
i.e. = (externally) and = (internally)
C2P r2 C2 T r2
r x −r x r y −r y   r1 x2 + r2 x1 r1 y 2 + r2 y1 
∴ P ≡  1 2 2 1 , 1 2 2 1  and T ≡  , .
 r1 − r2 r1 − r2   r1 + r2 r1 + r2 

Steps to find Equations of Common Tangents


Let the equation of tangent of any circle in the slope form be (y+f) = m(x+g) + a 1 + m2 where, a is radius of circle
and m is the slope of tangent.
The value of ‘m’ can be obtained by substituting the co-ordinates of the point P and T in the above equation.

Note: Length of an external (or direct) common tangent, Lext = d2 − (r1 − r2 )2 , and Length of an internal (or

transverse) common tangent, Lint = d2 − (r1 + r2 )2 . where, d is the distance between the centres of the two circles,

and r1, r2 are the radii of the two circles. Therefore, the length of internal common tangent is always less than the
length of the external common tangent.
Case-II: When the distance between the centres is equal to the sum of radii (Circles touching externally)

C1 C2= r1 + r2
Direct common
tangent

r1
r2
P C1
C2 T

Transverse common
tangents

Figure 9.44: Common tangents of circles touching externally

In this case three common tangents can be drawn, two direct common tangents and one transverse common
tangent.
Case III: When the distance between the centres is less than the sum of radii. (Intersecting circles)

r1 − r2 < C1 C2 < r1 + r2
Direct common
Tangent

P C1
C2

Figure 9.45: Common tangents for intersecting circles


M a them a ti c s | 9.27

In this case two direct common tangents can be drawn as shown in the diagram.
Case IV: When the distance between the centres is equal to the difference of the radii. (Circles touching each other
internally), i.e.C1C2 = r1 –r2.

Tangent at
the point of
contact
r2
P C1
C2
r1

Figure 9.46: Common tangents for circles touching each other internally

In this case the total number of common tangents is one.


Case V: When the distance between the centres is less than the difference of the radii. (Circles neither touch each
other nor intersect), i.e. C1C2<r1 – r2.

r2
C1
C2
r1

Figure 9.47

In this case, the number of common tangents is zero.

Illustration 31: Examine if the two circles x2 + y2 – 2x – 4y = 0 and x2 + y2 – 8y – 4 = 0 touch each other externally
or internally. (JEE MAIN)

Sol: When distance between centre of circle is equal to the sum of their radius then they touches eachother
externally and when it is equal to the difference of their radius then circle touches eachother internally.
Let C1 and C2 be the centres and r1 and r2 the radii of S1 ≡x2 + y2 – 2x – 4y = 0 and S1 ≡x2 + y2 – 8y – 4 = 0 respectively.
∴ C1 ≡ (1, 2), C2 ≡ (0, 4), r1 = 5 , r2 = 2 5

(1 − 0 ) + ( 2 − 4 )
2 2
Now, C1C2 = = 5 and

r1 +r2 = 3 5 ,r1 –r2 = 5


Thus, C1C2 = r1 –r2, hence the two circles touch each other internally,

Illustration 32: Prove that the two circles x2 + y2 + 2ax + c = 0 and x2 + y2 +2by + c = 0 touch each other,
1 1 1
if + =  (JEE ADVANCED)
2
a b 2 c

Sol: Two circles touch each other if distance between centres of these two circles are equal to the sum or difference
of their radius.
Let centres of given circles be C1 and C2 and their radii be r1 and r2 respectively.
9 .28 | Circle

∴ C1 ≡ (–a, 0); r1 = a2 − c and C2 ≡ (0, – b); r2 = b2 − c

Two circles touch each other, if C1C2 = r1 ± r2


⇒ a2 + b2= a2 − c ± b2 − c ⇒ a2 + b2 = a2 – c + b2 – c ±2 (a − c )(b
2 2
−c )
⇒ c =± a b − a c − b c + c
2 2 2 2 2
⇒ c = a b – a c – b c + c
2 2 2 2 2 2

1 1 1
⇒ a2c+ b2c = a2b2 ∴ + =.
a 2 2
b c

Illustration 33: An equation of a common tangent to the circles x2 + y2 + 14x – 4y + 28 = 0


and x2 + y2 – 14x + 4y – 28 = 0 is  (JEE ADVANCED)
(A) x – 7 = 0 (B) y + 7 = 0 (C) 28x + 45y + 371 = 0 (D) None of these

Sol: (C) Calculate the distance between the centres and use the different cases of two circles.
Let S1 ≡x2 + y2 + 14x – 4y + 28 = 0 …(i) A2
A1
⇒ C1 = (-7 , 2) and r1 = 5
9
5
and, S2 ≡x + y - 14x + 4y - 28 = 0
2 2
…(ii) P
⇒ C2 = (7 , -2) and r2 = 9

( 7 + 7 ) + ( −2 − 2)
2 2
∴ C1C=
2 > r1 + r2.
Figure 9.48
Hence, four common tangents are possible.
For x – 7 = 0, … (iii)
Clearly, C2 lies on the (iii).
For y + 7 = 0 … (iv)
Length of perpendicular from C1 = 9 > r1.
For 28x + 45y + 371 = 0 … (v)
180o-
28( −7) + 45(2) + 371 265
Length of perpendicular from=
C1 = = r1 . r1 r2
282 + 452 53 C1
d C2

28(7) + 45( −2) + 371 477


Length of perpendicular from C2 = = = r2 .
282 + 452 53
Figure 9.49

14. ANGLE OF INTERSECTION OF TWO CIRCLES


The angle of intersection between two circles S = 0 and S’ = 0 is defined as the angle between their tangents at
their point of intersection.
If S ≡ x2 + y2 + 2g1x + 2f1y + c1 = 0;
P
S’ ≡ x2 + y2 + 2g2x + 2f2y + c2 = 0
90°
are two circles with radii r1, r2 and d be the distance between their centres then
r 2 + r 2 − d2
C1 C2
the angle of intersection θ between them is given by cos (180 − θ ) = 1 2 (-g1, -f1) (-g2, -f2)
2r1r2
2 ( g1g2 + f1 f2 ) − ( c1 + c2 )
or, cos (180 − θ ) = .
2 g12 + f12 − c1 g22 + f22 − c1 Figure 9.50: Angle of intersection
M a them a ti c s | 9.29

Condition of Orthogonality: Two circles are said to be orthogonal to each other if the angle of intersection of the
two circles is 90°.
⇒ 2(g1g2 + f1f2) = c1 + c2 .

PLANCESS CONCEPTS

If two circles are orthogonal, then the polar of a point ‘P’ on first circle w.r.t. the second circle passes
though the point Q which is the other end of the diameter through P. Hence locus of a point which
moves such that its polar w.r.t. the circles S1 = 0, S2 =0 & S3 = 0 are concurrent in a circle which is
orthogonal all the three circles.
Ravi Vooda (JEE 2009, AIR 71)

Illustration 34: If a circle passes through the point (3, 4) and cuts the circle x2 + y2 = a2 orthogonally, the equation
of the locus of its centre is (JEE MAIN)
(A) 3x + 4y – a2 = 0 (B) 6x + 8y = a2 + 25
(C) 6x + 8y + a2 + 25 = 0 (D) 3x + 4y =a2 + 25

Sol : (B)
As we know Two circle are said to be orthogonal if 2(g1g2 + f1f2) = c1+c2. So by considering required equation of
circle as x2 + y2+ 2gx + 2fy + c = 0 and As point (3, 4) satisfies this equation so by solving these two equation we
will get required equation of the locus of its centre.
Let the equation of the circle be x2 + y2+ 2gx + 2fy + c = 0 ... (i)
As the point (3, 4) lies on (i), we have 9 + 16 + 6g +8f + c = 0
⇒ 6g + 8f + c = – 25 ... (ii)
⇒ 2g×0 + 2f×0 = c – a2 ⇒ c = a2 .
∴ From equation (ii), we have 6g + 8f + a2 + 25 = 0.
Hence locus of the centre (–g, –f) is 6x + 8y – (a2 + 25) = 0.

Illustration 35: Obtain the equation of the circle orthogonal to both the circles x2 + y2 +3x – 5y + 56 = 0 and
4x2 + 4y2 – 28x + 29 = 0 and whose centre lies on the line 3x + 4x + 1 = 0. (JEE ADVANCED)

Sol: By considering the required circle to be S ≡ x2 + y2 + 2gx + 2fy + c = 0 and using orthogonality formula
2(g1g2 + f1f2) = c1+c2 we will get a relation between g and f. Also as the centre lies on the line 3x + 4x + 1 = 0 , by
solving these equation we will get required result.
Let the required circle be S ≡ x2 + y2 + 2gx + 2fy + c = 0 … (i)
Given S1 ≡ x + y + 3x – 5y + 56 = 0
2 2
… (ii)
29
and, S2 ≡ x2 + y2 – 7x + = 0. ... (iii)
4
Since (i) is orthogonal to (ii) and (iii)
3  5
∴ 2g   + 2f  −  =c + 6 + 2f ⇒ 3g – 5f = c + 6 ... (iv)
2
   2
9 .30 | Circle

 7 29 29
and 2g  −  + 2f.0 ⇒ c+ ⇒ –7g = c +  ...(v)
 2 4 4

From (iv) and (v), we get 40g –20f = –5 ... (vi)
Given line is 3x + 4y = –1  ... (vii)
(–g, –f) also lies on the line (vii). ⇒ –3g – 4f = –1 ... (viii)
1 29
∴ g =0, f = and c =– [From (vi) and (viii)]
4 4
1 29
∴ The equation of the circle is x2 + y2 + y– = 0 or, 4(x2 + y2) + 2y –29 = 0
2 4

15. FAMILY OF CIRCLES


(a) The equation of the family of circles passing through the point of intersection of two given circle S = 0 and
S’ = 0 is given by S + λS’ = 0, (where λ is a parameter, λ≠–1)

S’=0

S=0

Figure 9.51

(b) The equation of the family of circles passing through the point of intersection of circle S = 0 and a line L = 0
is given by S + λL = 0, (where λ is a parameter)

S+L=0 S=0
L=0

Figure 9.52

(c) The equation of the family of circles touching the circle S =0 and the line L = 0 at their point of contact P is
S + λL = 0, (where λ is a parameter)

S+L=0 S=0

L=0
Figure 9.53
M a them a ti c s | 9.31

(d) The equation of a family of circles passing through two given points P(x1, x1) and Q(x2, x2) can be written in the
form
x y 1
(x –x1) (x – x2) + (y –y1)(y – y2) + λ x1 y1 1 = 0, (where λ is a parameter)
x2 y 2 1

Figure 9.54

In this equation, (x –x1) (x – x2) + (y –y1)(y – y2) = 0 is the equation of the circle with P and Q as the end points
x y 1
of the diameter and x1 y1 1 =0 is the equation of the line through P and Q.
x2 y2 1

(e) The equation of the family of circles touching the circle S ≡ x2 + y2 + 2gx +2fy +c = 0 at point P(x1, y1) is x2 +
y2 + 2gx + 2fy + c + λ {xx1 + yy1 + g(x +x1) + f(y + y1) +c} = 0 or, S + λL = 0, where, L = 0 is the equation of
the tangent to the circle at P(x1, y1) and λ ∈ R.

p(x1,y1)

Figure 9.55

(f) The equation of family of circle, which touch y – y1 = m(x –x1) at (x1, y1) for any finite m is
(x – x1)2 + (y – y1)2 + λ {(y – y1) –m(x – x1)} = 0. And if m is infinite, the family of circle is
(x – x1)2 + (y – y1)2 + λ(x – x1) = 0, (where λ is a parameter)

p(x1,y1)

Figure 9.56
9 .32 | Circle

Note that (x – x1)2 + (y – y1)2 = 0 represents the equation of a point circle with centre at (x1, y1)
(g) Equation of the circles given in diagram is
(x – x1)(x – x2) + (y – y1) (y – y2) ±cot θ {(x – x1)(y – y2) – (x – x2) (y – y1)} = 0

(h) Family of circles circumscribing a triangle whose side are given by L1 = 0; L2 = 0 and L3 = 0 is given by L1 L2 + λL2
L3 + µL3 L1 = 0 provided coefficient of xy = 0 and co-coefficient of x2 = co-efficient of y2.

(x1,y1)

 

(x2,y2)

Figure 9.57

(i) Equation of circle circumscribing a quadrilateral whose sides in order are represented by the lines L1 = 0, L2 =
0, L3 = 0 & L4 = 0 are L1 L3 + λL2 L4 = 0 where value of λ can be found out by using condition that co-efficient
of x2 = y2 and co-efficient of xy = 0.

Illustration 36: Find the equation of circle through the points A(1, 1) & B(2, 2) and whose radius is 1.(JEE MAIN)

Sol: As we know that, equation of family of circle passing through (x1, y1) and
(x2, y2) is given by (x − x1 )(x − x2 ) + (y − y1 )(y − y 2 ) + λ(x − y) = 0
Equation of AB is x – y = 0
∴ Equation of the family of circle passing through A and B is
(x – 1)(x – 2) + (y – 1) (y – 2) + λ (x – y) = 0 or x2 + y2 + (λ – 3)x – (λ + 3) y + 4 = 0

( λ − 3) ( λ + 3)
2 2

∴ Radius = + −4 .
4 4

( λ − 3) ( λ + 3)
2 2

According to the question, + −4 =1


4 4
or (λ – 3)2 + (λ +3)2 – 16 = 4 or 2λ2 = 2 or λ = ±1
∴ Equation of circle is x2 + y2 – 2x – 4y + 4 = 0 and x2 + y2 – 4x –2y + 4 = 0

Illustration 37: Find the equations of circles which touches 2x – y + 3 = 0 and pass through the points of intersection
of the line x + 2y – 1 = 0 and the circle x2 + y2 –2x + 1 = 0. (JEE MAIN)

Sol: Here in this problem the equation of family of circle will be S + λL = 0 by solving this equation we will get centre
and radius of required circle in the form of λ and as this circle touches the line 2x – y + 3 = 0 hence perpendicular
distance from centre of circle to the line is equal to the radius of circle.
Let the equation of the family of circles be S + λL = 0.
∴ x2+ y2 – 2x + 1 + λ (x + 2y – 1) = 0 or, x2 + y2 – x(2 – λ) + 2λy + (1 – λ) = 0  ... (i)
⇒ Centre (–g, –f) is ({2 – λ}/2, – λ)

(2 − λ )
2
1 λ
⇒r= g2 + f 2 −=
c + λ 2 − (1 − =
λ) λ2
5= 5.
4 2 2
M a them a ti c s | 9.33

Since the circle touches the line 2x – y + 3 = 0,

2. ( 2 − λ ) / 2 − ( −λ ) + 3 λ λ
∴ = 5 or, 5 = ± 5 ⇒ λ = ±2  … (ii)
± 5 2 2

Hence, the required circles are x2 + y2+ 4y – 1 = 0 and x2 + y2 – 4x – 4y +3 = 0.

Illustration 38: If P and Q are the points of intersection of the circles x2 + y2 + 3x +7y +2p – 5 = 0
and x2 + y2 +2x + 2y + p2 = 0, then there is a circle passing through P, Q and (1, 1) for (JEE MAIN)
(A) All except two values of p (B) Exactly one value of p
(C) All values of p (D) All except one value of p.

Sol: (D) Here in this problem the equation of family of circle will be S + λL = 0 . and as the circle passes through
(1, 1), we can find the values of P such that λ is any real no. except – 1.
Equation of a circle passing through P and Q is
x2 + y2 + 3x +7y +2p – 5 + λ (x2 + y2 + 2x + 2y – p2) = 0 … (i)
Since (i) also passes through (1 , 1), we get (7 + 2p) – λ (p2 – 6) = 0
7 + 2p
⇒λ= ≠ –1 ⇒ p ≠ –1.
p2 − 6

Illustration 39: C1 and C2 are circles of unit radius with centres at (0, 0) and (1, 0) respectively. C3 is a circle of unit
radius, passes through the centres of the circles C1 and C2 and have its centre above x-axis. Equation of the common
tangent to C1 and C3 which does not pass through C2 is  (JEE ADVANCED)
(A) x – 3 y + 2 = 0 (B) 3x–y+2=0

(C) 3 x – y – 2= 0 (D) x + 3 y + 2 = 0

Sol: (B) Equation of any circle passing through any two point (x1, y1)
Y
and (x2, y2) is given by
x y 1
( x – x1 ) ( x – x2 ) + ( y – y1 ) ( y – y 2 ) + λ 0 0 1 = 0
1 0 1

Equation of any circle passing through the centre of C1 and C2 is


x y 1
(2 2 )
C3 1 , 3

(x – 0) (x – 1) + (y – 0) (y – 0) + λ 0 0 1 = 0
1 0 1
⇒ x + y – x + λy = 0.
2 2
… (i) X’
C1 C2
X
(0,0) (1,0)
If (i) represents C3, its radius = 1
⇒ 1 = (1/4) + (λ2/4) ⇒ λ = - 3 (as λ cannot be +ve)
Y’
Hence, the equation of C3 is x2 + y2 – x – 3y = 0.
Figure 9.58
Since the radius of C1 and C3 are equal, their common tangents will
be parallel to the line joining their centres (0, 0)
1 3 
and  , 
2 2 
 
9 .34 | Circle

So, let the equation of a common tangent be y = 3 x + k. … (ii)


k
From the condition of tangency on C1, we get = 1 ⇒ k = ±2
3+1

Since the tangent does not pass through C2, the equation of the required common tangent is 3 x – y + 2 = 0.

Illustration 40: Find the equation of circle circumscribing the triangle whose sides are 3x – y – 9 = 0, 5x – 3y– 23 = 0
& x + y – 3 = 0. (JEE ADVANCED)
Sol: Given L1 ≡ 3x – y – 9 = 0 L2 ≡ 5x – 3y – 23 = 0 L3 ≡ x + y – 3 = 0
Family of circles circumscribing a triangle whose side are L1 = 0; L2 = 0 and L3 = 0 is
L1L2 + λL2 L3 + µL3 L1 = 0 provided coefficient of xy = 0 & co-coefficient of x2 = co-efficient of y2 .
∴ L1L2 + λL2L3 + µL1L3 = 0
⇒ (3x – y – 9)(5x – 3y – 23) + λ(5x – 3y – 23)(x + y – 3) + µ(3x – y – 9) (x + y – 3) = 0
⇒ (15x2 + 3y2 – 14xy – 114x + 50y + 207) + λ(5x2 – 3y2 + 2xy – 38x – 14y + 69)
+ µ(3x2 – y2 + 2xy –18x – 6y + 27) = 0
⇒ (5λ + 3µ + 15) x2 + (3 – 3λ – µ) y2 + xy(2λ + 2µ – 14) – x(114 + 38λ + 18µ)
+ y(50 – 14λ – 6µ) + (207 + 69λ +27µ) = 0 ... (i)
The equation (i) represents a circle if
coefficient of x2 = coefficient of y2
⇒ 5λ + 3µ + 15 = 3 – 3λ – µ ⇒ 8λ + 4µ + 12 = 0 ; 2λ + µ + 3 = 0 ... (ii)
and, coefficient of xy = 0
⇒ 2λ + 2µ – 14 = 0 ⇒λ + µ – 7 = 0  ... (iii)
From equation (ii) and (iii), we have λ = –10, µ = 17
Putting these values of λ & µ in equation (i), we get 2x2 + 2y2 – 5x + 11y – 3 = 0

16. RADICAL AXIS AND RADICAL CENTRE


P(x1,y1)

16.1 Radical Axis


The radical axis of two circles is defined as the locus of a point which c2
moves such that the lengths of the tangents drawn from it to the two c1
s2 =0
circles are equal. The radical axis of two circles is a straight line.
S1 = S2 s1 =0

⇒ S1 = S2 ⇒ S1 − S2 = 0
Figure 9.59
Consider two circles given by S1 = 0 and S2 = 0. Then the equation of
the radical axis of the two circle is S1 – S2 = 0
i.e. 2x (g1 – g2) + 2y(f1 – f2) + c1 – c2 = 0,
which is a straight line.
S1
Properties of the radical axis S1-S2=o
S2
(a) For two intersecting circles the radical axis and common chord are identical. Also, the
radical axis and the common tangent are same for two circles touching each other. Figure 9.60
M a them a ti c s | 9.35

(b) The radical axis is perpendicular to the line joining the centres of the two circles.
(c) If two circles cut a third circle orthogonally, the radical axis of the two circles will pass through the centre of
the third circle.
(d) Radical axis does not exist if circles are concentric.
(e) Radical axis does not always pass through the mid-point of the line joining the centre of the two circles.
(f) The radical axis of two circles bisects all common tangents of the two circles.

L
16.2 Radical Centre
s1=0
The point of intersection of the radical axis of three circles, taken in pairs,
is known as their radical centre. s3=0
Let the three circles be
S1 = 0 …..(i), S2 = 0 ...(ii), S3 = 0 ...(iii)
O
Refer to the diagrams shown alongside.
N
Let the straight line OL be the radical axis of the circles S1 = 0 & S3 = 0
and the straight line OM be the radical axis of the circles S1 = 0 & S2 = 0.
The equation of any straight line passing through O is given by (S1 – S2) M
s2=0
+ λ(S3 – S1) = 0 , where λ is any constant.
Figure 9.61
For λ= 1 , this equation become S2 – S3 = 0 , which is, equation of ON.
Clearly, the third radical axis also passes through the point where the
straight lines OL and OM meet. Hence, the point of intersection of the three radical axis, O is the radical centre.

Properties of radical center


(a) Co-ordinates of radical centre can be found by solving the equation S1 = S2 = S3.
(b) The radical centre does not exist if the centre of three circles are collinear.
(c) The circles with centre at radical centre and radius is equal to the length of tangents from radical centre to any
of the circle will cut the three circle orthogonally.
(d) If circles are drawn on three sides of a triangle as diameter then radical centre of the three circles is the
orthocenter of the triangle. Hence, in case of a right angled triangle, the radical centre of the three circles with
the sides as diameter is the vertex with the right angle.

PLANCESS CONCEPTS

Alternate approach to find the equation of the tangent of a circle passing through a point lying on a
given circle.
Consider a point (x1 , y1) on the given circle S1 = 0. Then the equation of a point circle with (x1 , y1) as the
centre is S2 ≡ (x – x1)2 + (y – y1)2= 0. Now we have two circles - one given circle and another point circle.
We now have to find the radical axis of those two circles, which is S1 – S2 = 0.
E.g.: Given a circle x2 + y2 = 8 and the point on circle is (2, 2), we need to find equation of a tangent to
the circle at point (2, 2).
Point circle: (x – 2)2 + (y – 2)2= 0 ⇒ x2 + y2 – 4x – 4y +8= 0
Hence, the radical axis is S1 – S2 = 0. ⇒ x + y = 4, which is also the tangent to the given circle at the
point (2, 2).
Akshat Kharaya (JEE 2009, AIR 235)
9 .36 | Circle

Illustration 41: Find the co-ordinates of the point from which the lengths of the tangents to the following three
circles be equal. 3x2 + 3y2 + 4x – 6y – 1= 0, 2x2 + 2y2 – 3x – 2y – 4 = 0, 2x2 + 2y2 – x + y – 1 = 0. (JEE MAIN)

Sol: Here by using formula S1 – S2 = 0, S2 – S3 = 0, and S3 – S1 = 0 we will get equations of radical axis and solving
these equations we will get required co-ordinate.
Reducing the equation of the circles to the standard form,
4 1
S1 ≡x2 + y2 + x – 2y – = 0
3 3
3
S2 ≡x2 + y2 - x–y–2=0
2
1 1 1
S3 ≡x2 + y2 - x + y – = 0
2 2 2
Hence, the equations of the three radical axis is given by
17 5
L1 ≡ x – y + = 0 … (i)
6 3
3 3
L2 ≡–x – y – = 0,  … (ii)
2 2
11 5 1
and, L3 ≡– x+ y+ = 0. … (iii)
6 2 6
 16 31 
Solving (i) and (ii), we get the point  − ,  , which also satisfies the equation (iii).
 21 63 
This point is called the radical centre and by definition the length of the tangents from it to the three circles are
equal.

Illustration 42: Find the equation of the circle orthogonal to the three circles x2 + y2 – 2x + 3y – 7 = 0, x2 + y2 +
5x – 5y + 9 = 0 and x2 + y2 + 7x – 9y + 29 = 0 (JEE ADVANCED)

Sol: By using formula of radical axis we will get co- ordinate of radical centre which is also equal to the centre of
required circle.
The given circles are
S1 ≡x2 + y2 – 2x + 3y – 7 = 0 … (i)
S2 ≡x2 + y2 + 5x – 5y + 9 = 0 … (ii)
and S3 ≡x + y + 7x – 9y + 29 = 0
2 2
... (iii)
The radical axis of S1 = 0 and S2 = 0 is 7x – 8y + 16 = 0 ... (iv)
The radical axis of S2 = 0 and S3 = 0 is x – 2y + 10 = 0  ...(v)
∴ The radical centre is (8, 9).
Therefore, the length of the tangent from (8, 9) to each of the given circles is 149 .
∴ The required equation is (x – 8)2 + (y – 9)2 = 149 or x2 + y2 – 16x – 18y – 4 = 0.

Illustration 43: If two circles intersect a third circle orthogonally. Prove that their radical axis passes through the
centre of the third circle. (JEE ADVANCED)

Sol: By considering equation of these circles as Sr = x2 + y2 + 2grx + 2fry + cr = 0


(r = 1, 2, 3) and using radical axis formula we will prove given problem.
Let the given circles be Sr = x2 + y2 + 2grx + 2fry + cr = 0 (r = 1, 2, 3)
M a them a ti c s | 9.37

Let S1 and S2 cut each other orthogonally, then we have


2g1g2 + 2f1f2 = c1 + c2  ... (i)
Similarly, let S2 and S3 cut each other orthogonally, then we have
2g2g3+ 2f2f3 = c2 + c3 ... (ii)
Subtracting (ii) from (i), we get 2(g1 – g3)g2 + 2(f1 – f3)f2 = c1 – c3  ... (iii)
Now the radical axis of S1 and S3 is 2(g1 – g3)x +2(f1 – f3)y + c1 –c3 = 0 ... (iv)
From (iii) and (iv), the point (–g2, –f2) lies on the line (iv). Hence, proved.

Illustration 44: Prove that the square of the length of tangent that can be drawn from any point on one circle to
another circle is equal to twice the product of the perpendicular distance of the point from the radical axis of the
two circles, and the distance between their centres. (JEE ADVANCED)

Sol: Consider two circle as S1 ≡ x2 + y2 = a2 and S2 ≡ (x – h)2 + y2 = b2 and then by using radical axis formula and
perpendicular distance formula we will prove given problem.
We have to prove that PQ2 = 2 ×PN×C1 C2
Let the equation of the two circles be
S1 ≡ x2 + y2 = a2, and … (i)
S2 ≡ (x – h)2 + y2 = b2 … (ii)
Let P ≡ (a cos θ, a sin θ) be a point on the circle S1 =0 ∴ PQ = 2 2 2
a − b + h − 2ahcos θ
and, Radical axis is {x2 + y2 - a2} – {(x – h)2 + y2 - b2 } = 0
h2 + a2 − b2
⇒ –2 hx + h2 + a2 - b2 = 0 or x=
2h
h2 + a2 − b2 h2 + a2 − b2 − 2ahcos θ
⇒ PN = − acos θ =
2h 2h

h2 + a2 − b2 − 2ahcos θ PQ2
⇒ PN×C1C2 = ×h ⇒ PN × C1C2 =
2h 2
∴ PQ2 = 2 PN×C1C2

17. CO-AXIAL SYSTEM OF CIRCLES


A system (or a family) of circles, every pair of which have the same radical axis, are called co-axial circles.
(1) The equation of a system of co-axial circles, when the equation of the radical axis is P ≡ lx + my + n = 0 and,
one circle of the system is S ≡ x2 + y2 + 2gx + 2fy + c = 0 respectively, is S + λP = 0 (λ is an arbitrary constant).

S+P=0 S+P=0

S+P=0
S=0
P=0
Figure 9.62

(2) The equation of a co-axial system of circles, when the equation of any two circles of the system are S1 ≡ x2 + y2
+ 2g1x + 2f1y + c1 = 0 and S2 ≡ x2 + y2 + 2g2x + 2f2y + c2 = 0 respectively, is S1 + λ(S1 – S2) = 0
9 .38 | Circle

S2 + (S1 - S2) = 0
S2 + (S1 - S2) = 0
S1 + (S1 - S2) = 0
S1+S1= 0 S2 = 0

S1 = 0
S1 = 0 S2 = 0 S1 - S2 = 0
Figure 9.63

or S2 + λ1(S1 – S2) = 0
Other form
S1 + λS2 = 0, (λ ≠ –1)

Properties of co-axial System of Circles


(a) Centres of all circles of a coaxial system lie on a straight line which is perpendicular to the common radical axis
as the line joining the centres of two circles is perpendicular to their radical axis.
(b) Circles passing through two fixed points P and Q form a coaxial system, because every pair of circles has the
same common chord PQ and therefore, the same radical axis which is perpendicular bisector of PQ.

PLANCESS CONCEPTS

The equation of a system of co-axial circles in the simplest form is x2 + y2 + 2gx + c = 0 , where g is a
variable and c is a constant. This is the system with center on x-axis and y-axis as common radical axis.
Anvit Tawar (JEE 2009, AIR 9)

Illustration 45: Find the equation of the system of coaxial circles that are tangent at ( )
2, 4 to the locus of point
of intersection of mutually perpendicular tangents to the circle x2 + y2 = 9. (JEE ADVANCED)

Sol: The locus of point of intersection of mutually perpendicular tangents is known as the Director circle. Hence by
using formula of director circle and co-axial system of circle we will get required result.
∴ The equation of the locus of point of intersection of perpendicular tangents is
x2 + y2 = 18 ... (i)
Since, ( )
2, 4 satisfies the equation x2 + y2= 18,

∴ The tangent at ( )
2, 4 to the circle x2 + y2 = 18 is x⋅ 2 + y⋅4 = 18 ... (ii)

The equation of the family of circles touching (i) at ( )


2, 4 is

x2 + y2 –18 +λ ( 2x + 4y – 18) = 0 or x2 + y2 + 2 λx + 4λy – 18(λ + 1) = 0 ... (iii)

Also any two circles of (iii) have the same radical axis 2x + 4y – 18 = 0

∴ The required equation of coaxial circles is (iii).


M a them a ti c s | 9.39

Illustration 46: Find equation of circle co-axial with S1=x2+y2+4x+2y+1=0 and S2 = 2x2 + 2y2 – 2x – 4y – 3 = 0 and
centre of circle lies on radical axis of these 2 circles.  (JEE MAIN)

Sol: By using S1 – S2 = 0 and S1 + λL = 0 we will get equation of radical axis and equation of co-axial system of
circle respectively.
5
S1 – S2 = 0 ⇒ 5x + + 4y = 0 ⇒ 10x +8y + 5 = 0
2
∴ The equation of the radical axis is 10x + 8y + 5 = 0
The equation of the coaxial system of circles is x2 + y2 + 4x + 2y + 1 +λ(10x + 5 + 8y) = 0
23
⇒ Centre ≡ [ −(2 + 5λ ), − (1 + 4λ )] which lies on radical axis, after substituting we get ⇒ λ = −
82

Illustration 47: For what values of l and m the circles 5(x2 + y2) + ly – m = 0 belongs to the coaxial system
determined by the circles x2 + y2 + 2x + 4y – 6 = 0 and 2(x2 + y2) – x = 0 ? (JEE ADVANCED)

Sol: By using radical axis formula i.e. S1 – S2 = 0 we will get equations of radical axis and by solving them
simultaneously we will get required value of l and m.
Let the circles be S1 ≡ x2 + y2 + 2x + 4y – 6 = 0;
1
S2 ≡ x2 + y2 – x = 0;
2
l m
S3 ≡ x2 + y2 + y– = 0.
5 5
The equation of the radical axis of circles S1 = 0 and S2 = 0 is S1 – S2 = 0,
 1 
i.e., x2 + y2 + 2x + 4y – 6 –  x2 + y 2 − x  = 0 or, 5x + 8y – 12 = 0 ... (i)
 2 
The equation of the radical axis of circles S2 = 0 and S3 = 0 is S2 – S3 = 0,
1  l m
i.e., x2 + y2 – x –  x2 + y 2 + y −  = 0 or, 5x + 2ly – 2m = 0 ... (ii)
2  5 5
5 8 −12 4 6
On comparing (i) and (ii), = = ⇒ 1= = ∴ l = 4, m = 6.
5 2l −2m l m

18. LIMITING POINTS


Limiting point of system of co-axial circles are the centres of the point circles belonging to the family
Let the circle be x2 + y2 + 2gx + c = 0 where g is a variable and c is a constant.
∴ Centre ≡ (–g, 0) and Radius = g2 − c .
A circle is said to be a point circle, if the radius is equal to 0, i.e. g2 − c = 0 ⇒ g = ± c
Thus, we get the two limiting points of the given co-axial system as ( c , 0) and (– c , 0).
Depending on the sign of c, either the limiting points are real and distinct, real and coincident or imaginary.

18.1 System of Co-axial Circles when Limiting Points are given


Let (a, b) and (α , β) be two limiting points of a coaxial system of circles. Then, the equation of the corresponding
point circles are S1 ≡ (x – a)2 + (y – b)2 = 0 and S2 ≡ (x – α)2 + (y – β)2 = 0.
∴ The coaxial system of circles is given by S1 + λS2 = 0, λ ≠ –1.
or, (x – a)2 + (y – b)2 + λ{(x – α)2 + (y – β)2} = 0, λ ≠ –1.
9 .40 | Circle

PLANCESS CONCEPTS

If origin is a limiting point of the coaxial system containing the circles x2 + y2 + 2gx + 2fy + c = 0
 −gc −fc 
then the other limiting point is  , .
 g2 + f 2 g2 + f 2 
 
π
A common tangent drawn to any two circles of a coaxial system subtends an angle of at the limiting
points. 2

Chinmay S Purandare (JEE 2012, AIR 698)

Illustration 48: Equation of a circles through the origin and belonging to the co-axial system, of which the limiting
points are (1, 2), (4, 3) is (JEE ADVANCED)
(A) x2 + y2 – 2x + 4y = 0 (B) x2 + y2 – 8x – 6y = 0
(C) 2x2 + 2y2 – x – 7y = 0 (D) x2 + y2 – 6x – 10y = 0

Sol: (C) As we know, if (a, b) and (α , β) be two limiting points of a coaxial system of circles. Then, the equation of
the corresponding point circles are S1 ≡ (x – a)2 + (y – b)2 = 0 and S2 ≡ (x – α)2 + (y – β)2 = 0 so by using the formula
of co-axial system i.e. S1 + λS2 = 0 we will get required result.
Equations of the point circles having (1, 2) and (4,3) as centres is
S1 ≡ (x – 1)2 + (y – 2)2 = 0 ⇒ x2 + y2 – 2x – 4y + 5 = 0
and, S2 ≡ (x – 4)2 + (y – 3)2 = 0 ⇒ x2 + y2 – 8x – 6y + 25 = 0
∴ The co-axial system of circles is S1 + λ S2 = 0.
i.e. x2 + y2 – 2x –4y + 5 + λ(x2 + y2 – 8x – 6y + 25) = 0 … (i)
If (0, 0) lies on the circle given by equation (i), then
02 + 02 – 2(0) – 4(0) + 5 + λ(02 + 02 – 8(0) – 6(0) + 25) = 0
1
⇒ 5 + 25λ = 0 or, λ = –   .
5
∴ The equation of the required circle is 5(x2 + y2 – 2x – 4y + 5) – (x2 + y2 – 8x – 6y + 25) = 0
⇒ 4x2 + 4y2 – 2x – 14y = 0 ⇒ 2x2 + 2y2 – x – 7y = 0.

19. IMAGE OF THE CIRCLE BY LINE MIRROR


Here, let us consider a general equation of a circle x2 + y2 + 2gx + 2fy + c = 0 and a line lx + my + n = 0. If we take
the image of the circle in the given line, then the radius of image circle remains unchanged and the centre lies on
the opposite side of the line at an equal distance.
Let the centre of image circle be (x1, y1).
∴ Slope of C1C2 × Slope of (lx + my +n = 0) = –1 ..... (i)
And the mid-point of C1 and C2 lies on the line lx + my + n = 0
 x1 − g   y1 − f 
l   + m 
  + n =0  ….. (ii)
 2   2 
M a them a ti c s | 9.41

From (i) and (ii), we get the centre of the image circle and the radius is
hence the equation of the image.
(g
2
)
+ f 2 − c (same as the given circle), and

C1 Given
circle
r

Given line

C2 Image of the circle

Figure 9.64: Image of a circle

PROBLEM-SOLVING TACTICS

(a) Let S = 0, S’ = 0 be two circles with centers C1, C2 and radii R1, R2 respectively.
(i) If C1C2 > r1 + r2 then each circle lies completely outside the other circle.
(ii) If C1C2 = r1 + r2 then the two circles touch each other externally. (Trick) the point of contact divides C1C2
in the ratio r1 : r2 internally.
(iii) If |r1 – r2| < C1C2 < r1 + r2 then the two circles intersect at two points P and Q.
(iv) If C1C2 = |r1 – r2| then the two circles touch each other internally. (Trick) The point of contact divides C1C2
in the ratio r1 : r2 externally.
(v) If C1C2 < |r1 – r2| then one circle lies completely inside the other circle.

(b) Two intersecting circles are said to cut each other orthogonally if the angle between the circles is a right angle.
Let the circles be S = x2 + y2 + 2gx + 2fy + c = 0, S’ = x2 + y2 + 2g’x + 2f’y + c’ = 0.
And let d be the distance between the centers of two intersecting circles with radii r1, r2. The two cir-
cles will intersect orthogonally if and only if

(i) D2 = and
(ii) 2g g’ + 2f f’ = c + c’.
9 .42 | Circle

FORMULAE SHEET

 1. General equation of a circle: x2 + y2 + 2gx +2fy + c = 0


(i) Centre of the circle = (–g, –f).
1 1
g= coefficient of x, and f = coefficient of y.
2 2

(ii) r = g2 + f 2 − c

2 ax2 + 2hxy + by2 + 2gx + 2fy + c = 0 represents a circle if

(i) a = b ≠ 0 (ii) h = 0 (iii) ∆ = abc + 2hgf – af2 – bg2 – ch2 ≠ 0 (iv) g2 + f2– c ≥ 0

 3. if centre of circle is (h, k) and radius ‘r’ then equation of circle is: (x – h)2 + (y – k)2 = r2

 4. The equation of the circle drawn on the straight line joining two given points (x1 , y1) and (x2 , y2) as diameter
is : (x – x1) (x – x2) + (y – y1) (y – y2) = 0

2 2
 x + x 2 y1 + y 2   x2 − x1   y 2 − y1 
Centre :  1 .r=  + 
 2 , 2   
   2   2 

 5. (i) In parametric form:

x=–g+ (g 2
)
+ f 2 − c cos θ and y = –f + (g 2
)
+ f 2 − c sin θ , (0 ≤ θ < 2π)

 6. (i) Circle passing through three non-collinear points


x2 + y 2 x y 1
x12 + y12 x1 y1 1
A(x1 , y1), B(x2, y2), C(x3 ,y3) is represented by =0
x22 + y 22 x2 y2 1
x32 + y 32 x3 y3 1

 7. Circle circumscribing the triangle formed by the lines


a12 + b12
a1 b1
a1 x + b1 y + c1
a22 + b22
ai x + bi y + ci = 0 ( i= 1,2,3) : a2 b2 = 0
a2 x + b2 y + c2
a32 + b32
a3 b3
a3 x + b3 y + c3

 8. Intercepts length made by the circle On X and Y axes are 2 g2 − c and 2 f 2 − c respectively.

 9. Position of point (x1, y1) lies outside, on or inside a circle S ≡ x2 + y2 + 2gx + 2fy + c = 0.
When S1 ≡ x12 + y12 + 2gx + 2fy + c > = < 0 respectively.

 10. The power of P(x1, y1) w.r.t. S = x2 + y2 + 2gx + 2fy + c = 0 is equal to PA. PB which is S1 = x12 + y12 + 2gx1 + 2fy2 + c.
PA. PB = PC.PD = PT2 = square of the length of a tangent
M a them a ti c s | 9.43

 11. Intercept length cut off from the line y =mx + c by the circle x + y = a is 2 2 2 2
( )
a2 1 + m2 − c2
2
1+m
 12. The equation of tangent at (x1, y1) to circle x2 + y2 + 2gx + 2fy + c = 0 is xx1 + yy1 + g(x +x1) + f(y + y1) + c = 0.

 13. The equation of tangent at (a cos θ , a sin θ) is x cos θ + y sin θ = a

 14. Condition for tangency:


line y = mx + c is tangent of the circle x2 + y2 = a2 if c2 = a2(1 + m2)

  ma ±a 
and the point of contact of tangent y = mx ± a 1 + m2 is  , 
 2 
 1+m 1 + m2 

 15. The length of the tangent from a point P(x1,y1) to the circle S = x2 + y2 +2gx + 2fy + c = 0 is equal to

x12 + y12 + 2gx1 + 2fy1 + c

 16. Pair of tangent from point (0, 0) to the circle are at right angles if g2 + f2 = 2c.

 17. Equation of director circle of the circle x2 + y2 = a2 is equal to x2 + y2 = 2a2 .

 18. Equation of Director circle of circle x2 + y2 + 2gx + 2fy + c = 0 is


x2 + y2 + 2gx + 2fy + 2c – g2 – f2 = 0.
x y
 19. The equation of normal at any point (x1, y1) to the circle x2 + y2 = a2 is xy1 – x1y = 0 or = .
x1 y1
 20. Equation of normal at (a cosθ , a sinθ) is y = x tan θ or y = mx .

 21. The equation of the chord of contact of tangents drawn from a point (x1, y1)
to the circle x2 + y2 = a2 is xx1 + yy2 = a2. And
to the circle x2+ y2 + 2gx + 2fy + c = 0 is xx1 + yy1 + g(x + x1) + f(y + y1) + c = 0.
3

 22. Area of ∆ APQ is given by


(
a x12 + y12 − a2 ) 2
.=
RL3
. Where L & R are length of tangent and radius of circle.
x12 + y12 R 2 + L2
P
L

A R

 23. The equation of the chord of the circle x2 + y2 + 2gx + 2fy + c = 0. Bisected at the point (x1, y1) is given T = S1.
i.e., xx1 + yy1 + g(x + x1) + f(y + y1) + c = x12 + y12 + 2gx1 + 2fy1 + c.

 24. The equation of the common chord of two circles x2 + y2 + 2g1x + 2f1y + c1 = 0 and x2 + y2 + 2g2x + 2f2y + c2 = 0
is equal to 2x(g1 – g2) + 2y(f1 – f2) + c1 – c2 = 0 i.e., S1 – S2 = 0.

2 2
 25. Length of the common chord : PQ = 2(PM) = 2 C1P − C1M . Where,
C1P = radius of the circle S1 = 0
C1M =perpendicular length from the centre C1 to the common chord PQ.
9 .44 | Circle

 26. Equation of polar of the circle x2 + y2 + 2gx + 2fy + c = 0 and x2 + y2 = a2


w.r.t. (x1, y1) is xx1 + yy1+ g(x + x1) + f(y + y1) + c = 0 and xx1 + yy1 – a2 = 0. Respectively.

 a2l a2m 
 27. The pole of the line lx + my + n = 0 with respect to the circle x2 + y2 = a2:  − ,− 
 n n 

 28. P (x1, y1) and Q(x2, y2) are conjugate points of the circle x2 + y 2 + 2gx + 2fy + c =0
When x1 x2 + y1 y 2 + g(x1 + x2 ) + f(y1 + y 2 ) + c =.
0
If P and Q are conjugate points w.r.t. a circle with centre at O and radius r then PQ2 = OP2 + OQ – 2r2.

 29. The points P and T are a intersection point of direct common tangents and transverse. Common tangents
respectively, and it divide line joining the centres of the circles externally and internally respectively in the
ratio of their radii.
C1P r1
= (externally)
C2P r2
C1 T r1
= (internally)
C2 T r2
Hence, the ordinates of P and T are.

r x −r x r y −r y   r1 x2 + r2 x1 r1 y 2 + r2 y1 
P ≡  1 2 2 1 , 1 2 2 1  and T ≡  , 
 r1 − r2 r1 − r2   r1 + r2 r1 + r2 

 30. If two circles S ≡ x2+ y2 + 2g1x + 2f1y + c1 = 0 and S’ ≡ x2+ y2 + 2g2x + 2f2y + c2 = 0 of r1 , r2 and d be the distance
between their centres then the angle of intersection θ between them is given by

r12 + r22 − d2 2 ( g1g2 + f1 f2 ) − ( c1 + c2 )


(
cos 180 − θ = ) or cos = (180 − θ ) = .
2r1r2 2 g12 + f12 − c1 g12 + f12 − c2

 31. Condition for orthogonality: 2g1g2 + 2f1f2 = c1 + c2

 32. S1 – S2 = 0 the equation of the radical axis of the two circle. i.e. 2x (g1 – g2) + 2y (f1 – f2) + c1 – c2 = 0 which is
a straight line.

 33. The two limiting points of the given co-axial system are ( c , 0) and ( − c , 0).

 34. If two limiting points of a coaxial system of circles is (a, b) and (α, β).
then S1 + λS2 = 0, λ ≠ –1. or, {(x – a)2 + (y – b)2} + λ{(x – α)2 + (y – β)2} = 0, λ ≠ –1 is the
Coaxial system of circle.

 35. If origin is a limiting point of the coaxial system containing the circle x2 + y2 +2gx + 2fy + c = 0 then the other
 −gc −fc 
limiting point is  , .
 g2 + f 2 g2 + f 2 
 
M a them a ti c s | 9.45

Solved Examples

JEE Main/Boards Required circle is x2 + y2 – 6x – 6y + 9 = 0

Example 3: Lines 5x + 12y –10 = 0 and 5x – 12y – 40 =


Example 1: The given curves ax2 + 2hxy + by2 + 2gx + 0 touch a circle C1 (of diameter 6). If centre of C1 lies in
2fy + c = 0 and Ax2 + 2Hxy + By2 + 2Gx + 2Fy + c = 0 the first quadrant, find concentric circle C2 which cuts
intersect each other at four concyclic points then prove intercepts of length 8 units on each given line.
a−b A −B
that = .
h H Sol: Consider centre of required circle is (h, k) and by
using perpendicular distance formula from centre to
Sol: Equation of second degree curve passing through
given tangent we will get value of h and k.
the intersections of the given curves is S1 + λS2 = 0
Let centre of circle C1 be O (h, k), where h > 0 and k > 0
ax2 + 2hxy + by2 + 2gx + 2fy + c + λ(Ax2 + 2Hxy + By2
+ 2Gx + Fy + C) = 0  ….. (i) WP = 4 and OP = 3 … (given)
intersection points of of the two curves are concyclic, In OWP,
(i) must be a circle for some λ .
∴ Coefficient of x2 = coefficient of y2 and coefficient of OW = OP2 + WP2 = 42 + 32 = 5
xy = 0.
OW = 5 = radius of C2.
∴ a + λA = b + λB 5h + 12k − 10 5h − 12k − 40
⇒ = =3
and 2h + λ ⋅ 2H = 0 or a – b = λ(B – A) 13 13
and h = –λH
a − b λ (B − A ) a−b A −B Z
∴ = ; ∴ = . Q
h −λH h H
Y

Example 2: Find the equation of a circle which cuts the C1 C2


W O(h,k)
circle x2 + y2 – 6x +4y – 3 = 0 orthogonally and which
passes though (3, 0) and touches the y-axis. P

Sol: When two circle intersects each other orthogonally


X
then 2(g1g2 + f1f2) = c1 + c2. Hence by considering
centre as (h, k) and using given condition we can solve
problem. 5h + 12k − 10  5h − 12k − 40 
= ± 
13  13 
Let C(h, k) be the centre of required circle
⇒ Either 5h + 12 k – 10 = 5h – 12 k – 40
(h − 3 )
2
radius of circle = + k 2 = |h| ⇒ 24 k = - 30
∴ (h – 3) + k = h
2 2 2
−30
⇒k= (Not possible)
or k – 6h + 9 = 0 
2
…..(i) 40
Required circle is (x – h)2 + (y – k)2 = h2 Or 5h + 12 k – 10 = - 5h + 12 k + 40

or x2 + y2 – 2hx – 2ky + k2 = 0 ⇒ 10 h = 50

It is intersected by x2 + y2 – 6x + 4y – 3 = 0 , orthogonally; ⇒ H=5

∴ 2(–3)(–h) + 2(2)(–k) = k2 – 3 Substituting h = 5 in

or 6h – 4k + 3 = k2  …..(ii) 5h + 12k − 10
=3
13
Solve (i) and (ii) : h = 3, k = 3
9 .46 | Circle

⇒ k = 2 (as k > 0) Example 5: From a point P tangents are drawn to


circles x2 + y2 + x – 3 = 0,
Hence, equation of required circle is
(x – 5)2 + (y – 2)2 = 25 5
x2 + y2 –   x + y = 0 and 4x2 + 4y2 + 8x + 7y + 9 = 0,
3
Example 4: Find the locus of the middle points of the and they are of equal lengths. Find equation of a circle
chords of the circle x2 + y2 = a2 which pass through a passing through P and touching the line x + y = 5 at
given point (x1, y1). A(6, –1).

Sol: As line joining centre of given circle to the mid


Sol: By reading the problem we get that P is a radical
point of chord is perpendicular to the chord and
centre of these circles. Hence by radical axis formula
hence product of their slope will be – 1. Therefore by
we can obtain co-ordinate of point P, as required circle
considering mid point of chord as (α, β) and by finding
is passing from these points so we can obtain required
their slope we will get required equation.
equation.
Let M (α, β) be the middle point of any chord PQ
Write third circle as
through the given point (x1, y1). The centre of the circle
is O (0, 0). Clearly MO is perpendicular to PQ. 7 9
x2 + y2 + 2x +   y +   = 0
4
  4
By definition, P is radical centre of three circles. Equation
of two of the radical axis are
O(0,0) 8 7  21 
  x – y – 3 = 0 and x +   y +   = 0
3 4
   4 
which intersect at P(0, –3). Let required circle be
M(,) Q
P(x1,y1) x2 + y2 + 2gx + 2fy + c = 0

β − y1 with centre Q(–g, –f )


Now, slope of PQ =
α − x1 P(0, –3) lies on it
β−0 β ⇒ –6f + c + 9 = 0  … (i)
slope of OM = =
α−0 α A(6, –1) lies on it
β − y1 β
∴ · = –1 ⇒ 12g – 2f + c + 37 = 0 … (ii)
α − x1 α
Since, PA is perpendicular to x + y = 5
or α(α – x1) + β(β – y1) = 0
 −f + 1 
∴ the equation of the locus of M (α, β) is ∴   ( −1 ) =−1  … (iii)
 −g − 6 
x(x – x1) + y(y – y1) = 0
⇒ f–g=7
Alternative Solving (i), (ii) and (iii) for f, g and c, we have
The equation of chord when mid-point is known is 7 7
f = , g = – and c = 12.
T = S1 2 2
Hence equation of required circle is
Let the mid-point be ( α, β) x2 + y2 – 7x + 7y + 12 = 0.
∴ x α + y β − a2 = α2 + β2 − a2
∵ It passes through (x1, y1) we get Example 6: Find the equation of a circle which touches
the line x + y = 5 at the point P(–2, 7) and cut the circle
x, α + y , β = α2 + β2
x2 + y2 + 4x –6y + 9 = 0 orthogonally.
⇒ α ( α − x1 ) + β ( β − y1 ) = 0

∴ Required locus is Sol: Using the concept of family of circle and the
condition for two circles to be orthogonal, we can find
x ( x − x1 ) + y ( y − y1 ) =
0 the equation of the required circle.
As the circle is touching the line x + y = 5. It (-2, 7).
M a them a ti c s | 9.47

Consider the equation of circle as Sol: As polar of point (x1, y1) with respect to the circle x2
x y
( x + 2) + ( y + 7 ) + λ ( x + y − 5) =0
2 2
+ y2 = c2 is same as line + = 1.
a b
⇒ x2 + y 2 + x ( 4 + λ ) + y ( λ − 14 ) + 53 − 5λ = 0 ….(i) On comparing the two equations, we can prove the
 given statement.
∴ As the circle given equation (i) is orthogonal to
Let the pole be (x1, y1). Then the polar of (x1,y1) with
x2 + y 2 + 4x − 6y + 9 =0, respect to the circle x2 + y2 = c2 is
We have xx1 + yy1 = c2 … (i)
( 4 + λ ) .2 + ( λ − 14 )( −3=) 53 − 5λ + 9 x y
Now, the line (i) and + = 1 must be the same line.
⇒ 8 + 2λ − 3λ + 42= 62 − 5λ a b
⇒ 4λ =12 x1 y1 c2
∴ comparing coefficients, = =
1/a 1/b 1
⇒λ =3
or ax
= 1 by
= 1 c2 ,
∴ Equation of the circle is x + y + 7x − 11y + 38 =
2 2
0.
∴ ax1 = by1

Example 7: Find the equation of the circle described on ∴ (x1 , y1) always lies on the line ax = by which is a fixed
the common chord of the circles x2 + y2 - 4x – 5 = 0 and line.

x2 + y2 + 8y + 7 = 0 as diameter.
Example 9: Inside the circle x2 + y2 = a2 is inscribed
Sol: Use Geometry to find the centre and the radius of an equilateral triangle with the vertex at (a , 0). The
the required circle. equation of the side opposite to this vertex is

For x2 + y 2 − 4x − 5 = (A) 2x – a = 0 (B) x + a = 0


0
(C) 2x + a = 0 (D) 3x – 2a = 0
Centre ≡ ( +2, 0 )
Radius = 3 Sol: (C) As P (a , 0) be the vertex of the equilateral
triangles PQR inscribed in the circle x2 + y2 = a2 Let
For x2 + y 2 + 8y + 7 =0 M be the middle point of the side QR, then MOP is
Centre ≡ ( 0, − 4 ) C perpendicular to QR and O being the centroid of the
triangle OP = 2 (OM).
Radius = 3
(Circumcentre and Centroid of an equilateral triangle
A M B are same)
(2,0) (0,-4)
So if (h, k) be the coordinates of M, then
Y

The mid point of AB is the centre of the required circle


i.e. M ≡ (1, − 2 )
( 2a , 23 a) Q

and Radius
= AC2 − AM2
= 9 −5

=2
1
X
M O P(a,o)

Equation of circle is ( x − 1 ) + ( y − 2 ) =
2 2
4.

Example 8: Prove that, for all c ∈ R, the pole of the line


R
x y ( -a2 , - 23 a)
+ = 1 with respect to the circle x2 + y2 = c2 lies on
a b
a fixed line. 1
Y
9 .48 | Circle

2h + a 2k + 0 Sol: As we know Equation of any tangent to x2 + y2 = 1,


= 0 and =0
3 3 is y = mx ± 1 + m2 and perpendicular distance from
a centre to tangent is equal to its radius. By using this
⇒ h = −   and k = 0
2 condition we can solve above problem.

and hence the equation of BC is As they are drawn from A(–2, 0), conditions are
−2m ± 1 + m2
0=
a
x = – or 2x + a = 0. 1
2 ⇒m=±
3
Example 10 : Find the radical centre of the three circles Equations of tangents become
x2 + y2 = a2, (x – c)2 + y2 = a2 and x2 + (y – b)2 = a2.
T1 : 3y= x + 2
Sol: Here by using the formula
T2 : 3y =−x − 2
S 1 – S2 = 0 , S 2 − S 3 =
0 and S3 − S1 =
0
Circles touching C and having T1 and T2 as tangents
we will get equation of radical axis and by solving them must have their center on x-axis (the angle bisector of
we can obtain requird radical centre. T1 and T2).
Radical axis of first & second circle is given by Let C1 and C2 be the 2 circles and M (h1, 0) & L(h2, 0) be
(x + y ) – (x + y – 2cx + c ) = 0
2 2 2 2 2 their respective centers where

c h1 > 0 and h2 < 0


or x=
2 By tangency of T1, perpendicular distance from centre
Also the radical axis of first and third circle is given M is equal to radius r1 of the circle C1
by h1 + 2
∴ r1 =
(x + y ) – (x + y – 2by + b ) = 0
2 2 2 2 2 2
b As C1 and C touch each other r1 = h1 – 1
or y =
2
c b Y
⇒ The radical centre =  ,  .
2 2
T1
C2
JEE Advanced/Boards
X’ C1 (0,0)
X
(-2,0)
Example 1: Two distinct chords drawn from the point
P(a, b) to the circle x2 + y2 – ax – by = 0, (ab ≠ 0), are
bisected by the x-axis. Show that a2 > 8b2. T2

Sol: As Circle passes through (0, 0) and P(a, b) .Consider


the chord PQ intersect x-axis at A; then, Q is (α, –b). Y’
Hence by substituting this point to given equation of h1 + 2
or + 1 = h1
circle we can solve above problem. 2
∴ α2 + b2 – aα + b2 = 0 or α2 – aα + 2b2 = 0 or h1 = 4

Hence, Discriminant > 0 ∴ For circle C1 : centre is M(4, 0) and radius = 3.

⇒ a2 > 8b2 h2 + 2
Similarly for circle C2, –h2 – 1 =
2
Example 2: Let T1, T2 be two tangents drawn from (–2 , 0) ⇒ –2h2 – 2 = h2 + 2
to the circle C : x2 + y2 = 1. Determine circles touching C
and having T1, T2 as their pair of tangents. Further find (∴ h2 > –2; see figure)
the equation of all possible common tangents to these ⇒ –3h2 = 4
circles, when taken two at time.
4 1
or h2 = – and radius = .
3 3
M a them a ti c s | 9.49

Equations of two circles are (x – 4)2 + y2 = 9 and 2


 2r  r2
 4
2
1 or x12 +  y1 +  =
2  3  9
x +  + y = 
 3 9
C1 & C have x = 1 as transverse common tangent and ∴ Locus of (x1, y1) is a circle.
C2 & C have x = –1 as transverse common tangent.
Example 4: Derive the equation of the circle passing
Example 3 : Let AB be a chord of the circle x2 + y2 = r2 through the point P(2, 8) and touches the lines 4x – 3y
subtending a right angle at the centre O. Show that the – 24 = 0 and 4x + 3y – 42 = 0 and coordinates of the
centroid of the triangle PAB as P moves on the circle is centre less than or equal to 8.
a circle.
Sol: Here using Equations of bisectors of angle between
Sol: By considering point P(r cos θ , r sinθ) and centroid the lines we will get co-ordinate of centre of circle i.e
as point (x1, y1) we can obtain required result. O. and as OA = OP we can obtain required equation of
circle. consider O is the center of circle.
∆OAB is isosceles with
Let L1 ≡ 4x – 3y – 24 = 0
OA = OB = x(say)
L2 ≡ 4x + 3y – 42 = 0
We may assume AB is parallel to and below x-axis
and Let A and B denote the respective points of contact
r
∴ x2 + x2 = r2 ⇒ x =
2 Equations of bisectors of angle between the lines are;
 r r   r r 
∴ B is  ,−  and A is  − ,− 
 2 2  2 2 -24=
0
A 4x-3y
Let P be (r cos θ , r sinθ) and centroid of ∆PAB be G (x1, y1) P(2,8)
Y
P
C O

A
r
B
X’ X 4x+3y
O - 42=0
4x − 3y − 24 4x + 3y − 42

5 5
33
B i.e., y = 3 & x=
4
Y’ Since O lies on one of these bisectors and x-coordinate
of O is less then or equal to 8,
r r
r cos θ + − ∴ O lies on y = 3.
2 2,
∴ x1 = Let O be (a,3). Then, OA = CP
3
2
r r  4a − 33 
 =( a − 2 ) + 25
2
r sin θ − − or 
y1 = 2 2  5 
3 or 16a2 – 264a + (33)2 = 25{a2 – 4a + 29}
3x1 = r cos θ ; 3y1 = r(sin θ – 2) or 9a2 + 164a – 364 = 0
Eliminating θ, we get or (a – 2)(9a + 182) = 0
2
 3x1   3y1 
2 182
∴ a =2 or a = −
∴  r  +  r + 2  =
1 9
   
and radius = OP.
9 .50 | Circle

Example 5: Coordinates of a diagonal of a rectangle Sol: The condition for one circle to be within the other is
are (0, 0) and (4, 3). Find the equations of the tangents
C1C2 < r1 − r2
to the circumcircle of the rectangle which are parallel to
this diagonal. Without the loss of generality,

Sol: Here centre of circle is the mid-point of line OP Let λ > µ


hence by using slope point form we can get required
λ−µ λ2 µ2
equation of tangents. ∴ C1 C2 < r1 − r2 ⇒  < −c − −c
 2  4 4
Two extremities are O (0, 0) and P (4, 3). Middle point of λ 2 µ2 λ µ λ2 µ2
⇒ + −2× × < −c+ −c
 3 4 4 2 2 4 4
the diagonal OP is M  2,  which is the centre of the
 2  λ2   µ2 
9 5 −2  − c − c
circumscribed circle and radius is OM = 4 + =  4  4 
4 2   

 λ2   µ2  λµ
⇒2  − c − c  < 2. − 2c
 4  4  4
  

λ 2 µ2  λ 2 µ2  2 λ 2 µ2
− c + +c < + c2 −
16  4 4  16
P(4,3)  
λµ
2×c×
4
 λ 2 µ2 λ µ
c + − 2× ×  > 0
 4 4 2 2 
O(0,0) 
2
λ µ
c −  > 0
2 2
⇒C>0
Also ∴ λ > µ
3  −µ 
A line parallel to OP is y = x+c  , 0  will be inside
4  2 
It is a tangent to the circumscribed circle.
x 2 + y 2 + λx + c = 0
Therefore length of perpendicular from
µ2 µλ
3 ⇒ +0− +c < 0
 3 5 4
( 2) − 23 + C 5 4 2
M  2,  to it = ⇒ = µ2
 +c>0
 2 2 9 2 4
1+
16 λ µ µ2
5 5 25 ∴ > +c
or C =± ⋅ =± 2 4
2 4 8
λµ
3 25 ∴ >0
Hence tangents are y = x± 2
4 8
⇒ λµ>0
25
or 3x – 4y ± = 0. Hence, proved.
2

Example 6: The equations two circles are Example 7: A circle touches the line y = x at a point P
x + y + λx + c = 0 and x + y + µx + c = 0. Prove that
2 2 2 2 such that OP = 4 2 where O is the origin. The circle
one of the circles will be within the other if λµ > 0 and contains the point (–10, 2) in its interior and the length
c > 0. of its chord on the line x + y = 0 is 6 2 . Find the
equation of the circle.
M a them a ti c s | 9.51

Sol: In this question, the concept of rotation of axes Example 8: Derive the equation of the circle passing
would be useful. through the centres of the three given circles x2 + y2 –
4y – 5 = 0,
Let the new co-ordinate axis be rotated by an angle of
45° in the clockwise direction. Then x2 + y2 + 12x + 4y +31 = 0 and
=X x cos ( θ) + y sin ( θ ) x2 + y2 + 8x + 10y + 32 = 0.
Y =−x sin ( θ ) + y cos ( θ )
Sol: Find the relation between the centres of the circle
Where θ= 45° and there use the appropriate form of circle.
x−y Let P, Q and R denote the centres of the given circle
∴ X =
2 P ≡ ( 0, 2 ) , Q =( −6, − 2 ) and
x+y
Y =
2
R≡ ( −4, − 5)
The image after rotation would be −2 − 2 −4 2
∴ mPQ = = =
−6 − 0 −6 3
−5 + 2 −3
mQR
= =
−4 + 6 2
2 −3
∴ mPQ . mQR =× −1
=
C1 (0, 4 2) 3 2
⇒ PQ is perpendicular to QR
A
B
∴ Using diameter form, we get
3 2 (x - 0) (x + 4) + (y - 2) (y + 5) =0
C2 (0, -4 2)

Example 9: Area of Quadrilateral PQRS is 18, side PQ


|| RS and PQ = 2RS and PS ⊥ PQ and RS. then radius a
circle drawn inside the quadrilateral PQRS touching all
the sides is,
In ∆ ABC, AC, =
4 2 3
(A) 3 (B) 2 (C) (D) 1
AB = 3 2 2
Sol: (B) Let r be the radius of the circle, then PS = 2r.
( ) ( )
2 2
∴ Radius = 4 2 + 3 2 b 2
= Let P be the origin and PQ and PS as x-axis and y-axis
respectively.
∴ Equation of the circle is
∴ The coordinates of P, Q, R, S are (0, 0), (2a, 0), (a, 2r)
(X + 5 2 ) + (Y  4 2 ) =
(5 2 )
2 2 2
and (0, 2r) respectively.
1
2 2 ∴ Area (PQRS) =   (a + 2a)(2r) = 18
x−y
Or, 
 2
 x+y
+5 2 + 
  2

 4 2 =

5 2 ( ) 2
⇒ ar = 6.
or, ( x − y + 10 ) + ( x + y ± 8 ) =
2 2
100 S(0,2r) R(a,2r)

But, since (-10, 2) lies inside the circle.


r
The equation of the circle is
(r,r)
( x − y + 10 ) + ( x + y + 8 )
2 2
=100
Or, x2 + y 2 + 100 − 2xy − 20y + 20x
P(0,0) Q(2a,0)
+ x2 + y 2 + 64 + 2xy + 16y + 16x =
100
∴ Equation of QR is
Or, 2x2 + 2y 2 + 36x − 4y + 64 =
0
 y 2 − y1 
Or, x2 + y 2 + 18x − 2y + 32 =
0 (=
y − y1 )   ( x − x1 )
 x2 − x1 
9 .52 | Circle

 0 − 2r  Similarly, equation of BQ is
⇒ (=
y − 2r )   ( x − a)
 2a − a  asin β − 0
(y − v)
=
a cos β − a
( x − a)
−2r
⇒ ( y − 2r
= ) ( x − a)
a β β
2 a.sin
.cos
⇒ ay − 2ar =
−2rx + 2ar 2 2 x−a
=⇒y
β
( )
⇒ 2rx + ay − 4ar =
0 −a × 2 sin
2
∴ QR is a tangent to the circle β
⇒y=− cot   ( x − a)  …(ii)
2r 2 + ar − 4ar 2
∴ r
=
Now, we eliminate α , β using (i) and (ii)
4r 2 + a2
 α − β =2r
r ( 2r − 3a)
⇒ r
= α β
2 2 ⇒ − = r
4r + a 2 2
α β
⇒ ( 2r − 3a) = 4r 2 + a2
2
⇒ tan  −  =tan r
 2 2
⇒ 4r 2 + 9 a2 − 12 ar = 4r 2 + a2
α β
2
tan − tan
⇒8a =
12 ar ⇒ 2 2 = tan r
α β
⇒ 2a2 =
3 ar 1 + tan . tan
2 2
y a−x
⇒ 2a2 =×
3 6 −
x+a y
⇒a =3 ⇒ tan γ
=
y a−x
1+ ×
∴ r 2 (=
=  ar 6 ) a+ x y

y 2 − a2 + x2
Example 10: A circle having centre at (0, 0) and radius ⇒ tan γ
=
ay + xy + ay − xy
equal to 'a' meets the x - axis at P and Q. A(α) and B(β)
are points on this circle such that α – β = 2γ, where γ ⇒ x2 + y 2 − 2ay tan γ − a2 =0
is a constant. Then locus of the point of intersection of
PA and QB is a a-r
(A) x – y – 2ay tan γ = a
2 2 2 

(B) x2 + y2 – 2ay tan γ = a2 r

(C) x2 + y2 + 2ay tan γ = a2


(D) x2 – y2 + 2ay tan γ = a2 r

Sol: (B) Let the equation of the circle be x2 + y 2 =
a2 

∴ P ≡ ( −a, 0 ) and Q = ( a, 0 )
∴ Equation of PA is
a sin α − 0
( y − 0)
=
acos α + a
( x + a)
a sin α
=⇒y
a ( cos α + 1 )
( x + a)
α α
.cos a.2 sin
2 2 x+a
=⇒y ( )
2 α
a.2 cos
2
α
⇒ y tan
=
2
x+a  ( ) …(i)
M a them a ti c s | 9.53

JEE Main/Boards

Exercise 1 Q.15 Show that the line x + y = 2 touches the circles


x2 + y2 = 2 and x2 + y2 + 3x + 3y – 8 = 0 at the point
Q.1 Find the equation of the circle whose centre lies on where the two circles touch each other.
the line 2x – y – 3 = 0 and which passes through the
Q.16 One of the diameters of the circle circumscribing
points (3, –2) and (–2, 0).
the rectangle ABCD is 4y = x + 7. If A and B are the
Q.2 Show that four points (0, 0), (1, 1), (5, –5) and (6, –4) points (–3, 4) and (5, 4) respectively, find the area of the
are concylic. rectangle.

Q.3 Find the centre, the radius and the equation of the Q.17 A circle of radius 2 lies in the first quadrant
circle drawn on the line joining A(–1, 2) and B(3, –4) as and touches both the axes of co-ordinates, Find the
diameter. equation of the circle with centre at (6, 5) and touching
the above circle externally.
Q.4 Find the equation of the tangent and the normal to
the circle x2 + y2 = 25 at the point P(–3, –4).  1 
Q.18 If  mi ,  ; i = 1, 2, 3, 4 are four distinct point on
 mi 
Q.5 Show that the tangent to x2 + y2 = 5 at (1, –2) also
a circle, show that m1m2m3m4 = 1.
touches the circle x2 + y2 – 8x + 6y + 20 = 0

Q.6 Find the equation of the tangents to the circle x2 + Q.19 Show that the circle on the chord
y2 – 2x + 8y = 23 drawn from an external point (8, –3). xcosα + ysinα – p = 0 of the circle x2 + y2 = a2 as
diameter is x2 + y2 – a2 – 2p (xcosα + y sin α – p) = 0.
Q.7 Find the equation of the circle whose centre is
(–4, 2) and having the line x – y = 3 as a tangent Q.20 Find the length of the chord of the circle x2 + y2 = 16
which bisects the line joining the points (2, 3) and (1, 2)
Q.8 Find the equation of the circle through the points perpendicularly.
of intersections of two given circles
Q.21 Find the angle that the chord of circle x2 + y2 – 4y=0
x2 + y2 – 8x – 2y + 7 = 0 and along the line x + y = 1 subtends at the circumference
x2 + y2 – 4x + 10y + 8 = 0 and passing through (3, –3). of the larger segment.

Q.9 Find the equation of chord of the circle x2 + y2 – 4x Q.22 Prove that the equation x2 + y2 – 2x – 2λ y – 8=0,
= 0 which is bisected at the point (1, 1). where λ is a parameter, represents a family of circles
passing through two fixed points A and B on the x-axis.
Q.10 Find the equation of chord of contact of the circle Also find the equation of that circle of the family,
x2 + y2 – 4x = 0 with respect to the point (6, 0). the tangents to which at A and B meet on the line
x + 2y + 5 = 0.
Q.11 Find the length of the tangent drawn from the
point (3, 2) to the circle 4x2 + 4y2 + 4x + 16y + 13 = 0. Q.23 Find the area of the quadrilateral formed by
a pair of tangents from the point (4, 5) to the circle
Q.12 Obtain the equations of common tangents of the x2 + y2 – 4x – 2y – 11 = 0 and a pair of its radii.
circles x2 + y2 = 9 and x2 + y2 – 12x + 27 = 0.
Q.24 If the lines a1x + b1y + c1=0 and a2x + b2y + c2=0
Q.13 The centres of the circle passing through the cut the co-ordinate axes in concyclic points, prove that
points (0, 0), (1, 0) and touching the circle x2 + y2 = 9 a1a2 = b1b2.
1 
are  , ± 2  . Q.25 Show that the length of the tangent from any
2 
point on the circle
Q.14 The abscissae of two points A and B are the roots
of the equation x2+ 2ax – b2 = 0 and their ordinates x2 + y2 + 2gx + 2fy + c = 0 to the circle
are the roots of the equation x2 + 2px – q2 = 0. Find x2 + y2 + 2gx + 2fy + c1 = 0 is c1 − c .
the equation and the radius of the circle with AB as
diameter.
9 .54 | Circle

Q.26 Find the point from which the tangents to the (A) Are the vertices of a right triangle
three circles x2 + y2 – 4x + 7 = 0,
(B) The vertices of an isosceles triangle which is not
2x + 2y – 3x + 5y + 9 = 0
2 2
regular
and x2 + y2 + y = 0 are equal in length. Find also this (C) Vertices of a regular triangle
length.
(D) Are collinear
Q.27 The chord of contact of tangents from a point on
the circle x2 + y2 = a2 to the circle x2 + y2 = b2 touches Q.2 2x2 + 2y2 + 2λ x + λ2 = 0 represents a circle for :
the circle x2 + y2 = c2. Show that a, b, c are in G.P. (A) Each real value of λ
Q.28 Obtain the equation of the circle orthogonal to (B) No real value of λ
both the circles (C) Positive λ
x2 + y2 + 3x – 5y + 6 = 0 and (D) Negative λ
4x + 4y – 28x + 29 = 0 and whose centre lies on the
2 2

line 3x + 4y + 1 = 0. Q.3 The area of an equilateral triangle inscribed in the


circle x2 + y2 – 2x = 0 is
Q.29 From the point A (0, 3) on the circle x2 + 4x +
(y – 3)2 = 0, a chord AB is drawn and extended to a 3 3 3 3
(A) (B)
point M such that AM = 2AB. Find the equation of the 4 2
locus of M.
3 3
(C) (D) None of these
Q.30 From the origin, chords are drawn to the circle 8
(x – 1)2 + y2 = 1. Find the equation to the locus of the
middle points of these chords. Q.4 A circle of radius 5 has its centre on the negative
x-axis and passes through the point (2, 3). The intercept
Q.31 Tangent at any point on the circle x2 + y2=a2 meets made by the circle on the y-axis is
the circle x2 + y2 = b2 at P and Q. Find the condition on a
(A)10 (B) 2 21
and b such that tangents at P and Q meet at right angles.
(C) 2 11 (D) imaginary y-intercept
Q.32 The tangent from a point to the circle x2 + y2 = 1
is perpendicular to the tangent from the same point to
Q.5 The radii of the circle x2 + y2 = 1, x2 + y2 – 2x – 6y = 6
the circle x2 + y2 = 3. Show that the locus of the point
and x2 + y2 – 4x – 12y = 9 are in
is a circle .
(A) A.P. (B) G.P.
Q.33 A variable circle passes through the point A (a, b)
(C) H.P. (D) None of these
and touches the x-axis. Show that the locus of the other
end of the diameter through A is (x – a)2 = 4 by.
Q.6 If the equation x2 + y2 + 2λx + 4 = 0 and x2 + y2 –
Q.34 AB is a diameter of a circle. CD is a chord parallel 4λy + 8 = 0 represent real circles then the value of λ
to AB and 2CD = AB. The tangent at B meets the line AC can be
(produced) at E. Prove that AE = 2AB. (A) 5 (B) 2
(C) 3 (D) All of these

Exercise 2
Q.7 The equation of the image of the circle x2 + y2 +16x
Single Correct Choice Type – 24y + 183 = 0 by the line mirror 4x + 7y + 13 = 0 is;
(A) x2 + y2 + 32x – 4y + 235 = 0
Q.1 Centres of the three circles
(B) x2 + y2 + 32x + 4y – 235 = 0
x2 + y2 – 4x – 6y – 14 = 0
(C) x2 + y2 + 32x – 4y – 235 = 0
x2 + y2 + 2x + 4y – 5 = 0
(D) x2 + y2 + 32x + 4y + 235 = 0
and x2 + y2 – 10x – 16y + 7 = 0
M a them a ti c s | 9.55

Q.8 The circle described on the line joining the points y= 4 and the x-axis is
(0, 1), (a, b) as diameter cuts the x-axis in points whose y
abscissae are roots of the equation : y=4
(A) x + ax + b = 0
2
(B) x – ax + b = 0
2
x/2
y=
(C) x2 + ax – b = 0 (C) x2 – ax – b = 0 x
O

Q.9 A straight line l1 with equation x – 2y + 10 = 0 8 5


meets the circle with equation x2 + y2 = 100 at B in the (A) 4 + 2 5 (B) 4 +
5
first quadrant. A line through B, perpendicular to l1 cuts
6 5
the y-axis at P (0, t). The value of ‘t’ is (C) 2 + (D) 8 + 2 5
5
(A) 12 (B) 15 (C) 20 (D) 25

Q.15 From the point A (0, 3) on the circle x2 + 4x +
 1  1  1  1 (y – 3)2 = 0 a chord AB is drawn and extended to a point
Q.10 If  a,  ,  b,  ,  c,  and  d,  are four distinct M such that AM = 2AB. The equation of the locus of M is,
 a   b   c   d
point on a circle of radius 4 units then, abcd is equal to (A) x2 + 8x + y2 = 0
1 (B) x2 + 8x + (y – 3)2 = 0
(A) 4 (B) (C) 1 (D) 16
4 (C) (x –3)2 + 8x + y2 = 0
Q.11 The radius of the circle passing through the (D) x2 + 8x + 8y2 = 0
vertices of the triangle ABC, is
A Q.16 If L1 and L2 are the length of the tangent from (0, 5)
to the circles x2 + y2 + 2x – 4 = 0 and x2 + y2 – y + 1 = 0
12 12 then
(A) L1 = 2L2 (B) L2 = 2L1 (C) L1 = L2 (D) L21 = L2
B 6 C
Q.17 The line 2x – y + 1 = 0 is tangent to the circle
at the point (2, 5) and the centre of the circles lies on
8 15 3 15 x–2y = 4. The radius of the circle is
(A) (B) (C) 3 15 (D) 3 2
5 5
(A) 3 5 (B) 5 3 (C) 2 5 (D) 5 2
Q.12 The points A(a, 0), B(0, b), C(c, 0) and D(0, d) are
such that ac = bd and a, b, c, d are all non-zero. Then Q.18 Coordinates of the centre of the circle which
the points bisects the circumferences of the circles x2 + y2 = 1 ; x2
(A) Form a parallelogram (B) Do not lie on a circle + y2 + 2x – 3 = 0 and x2 + y2 + 2y – 3 = 0 is

(C) Form a trapezium (D) Are concyclic (A) (–1, –1) (B) (3, 3) (C) (2, 2) (D) (–2, –2)

Q.19 The anglebetween the two tangents from the


Q.13 Four unit circles pass through the origin and origin to the circle (x –7)2 + (y + 1)2 = 25 equals
have their centres on the coordinate axes. The area π π π π
of the quadrilateral whose vertices are the points of (A) (B) (C) (D)
6 3 2 4
intersection (in pairs) of the circle, is
(A) 1 sq. unit Q.20 In a right triangle ABC, right angled at A, on
the leg AC as diameter, a semicircle is described. The
(B) 2 2 sq. units chord joining A with the point of intersection of the
(C) 4sq. units hypotenuse and the semicircle, then the length AC
equals to
(D) Cannot be uniquely determined, insufficient data
AB ⋅ AD AB ⋅ AD
(A) (B)
2
AB + AD 2 AB +AD
Q.14 The x-coordinate of the center of the circle in the
1 AB ⋅ AD
(C) AB ⋅ AD (D)
first quadrant (see figure) tangent to the lines y = x,
2 AB2 − AD2
9 .56 | Circle

Q.21 Locus of all point P (x, y) satisfying x3 + y3 + 3xy = Q.7 The triangle PQR is inscribed in the circle x2 + y2
1 consists of union of = 25. If Q and R have coordinates (3, 4) and (–4, 3)
respectively, then ∠ QPR is equal to  (2002)
(A) A line and an isolated point
π π π π
(B) A line pair and an isolated point (A) (B) (C) (D)
2 3 4 6
(C) A line and a circle
Q.8 The number of common tangents to the circles
(D) A circle and an isolated point.
x2 + y2 = 4 and x2 + y2 – 6x – 8y = 24 is  (1998)

(A) 0 (B) 1 (C) 3 (D) 4


Previous Years’ Questions
Q.9 Tangents are drawn from the point (17, 7) to the
Q.1 The circle passing through the point(–1, 0) and circle x2 + y2 = 169.  (2007)
touching the y-axis at (0, 2) also passes through the
point  (2011) (A) Statement-I is true, statement-II is true and
statement-II is correct explanation for statement-I.
 3   5   3 5
(A)  − ,0  (B)  − ,2  (C)  − ,  (D) (–1, –4) (B) Statement-I is true, statement-II is true and
 2   2   2 2
statement-II is NOT the correct explanation for statement-I.
Q.2 Consider the two curves C1 : y2 = 4x (C) Statement-I is true, statement-II is false.

C2 : x2 + y2 – 6x + 1 = 0, then  (2008) (D) Statement-I is false, statement-II is true.

(A) C1 and C2 touch each other only at one point Statement-I: The tangents are mutually perpendicular.
(B) C1 and C2 touch each other exactly at two points
Statement-II: The locus of the points from which a
(C) C1 and C2 intersect (but do not touch) at exactly two mutually perpendicular tangents can be drawn to the
points given circle is x2 + y2 = 338.
(D) C1 and C2 neither intersect nor touch each other
Q.10 Find the equation of circle touching the line
Q.3 If one of the diameters of the circle x + y – 2x – 6y
2 2 2x + 3y + 1 = 0 at the point (1, –1) and is orthogonal to
+ 6 = 0 is a chord to the circle with centre (2, 1), then the circle which has the line segment having end points
the radius of the circle is  (2004) (0, –1) and (–2, 3) as the diameter.  (2004)

(A) 3 (B) 2 (C) 3 (D) 2


Q.11 Let C1 and C2 be two circles with C2 lying inside
C1. A circle C lying inside C1 touches C1 internally and C2
Q.4 The centre of circle inscribed in square formed by externally. Identify the locus of the centre of C (2001)
the lines x2 – 8x + 12 = 0 and y2 – 14y + 45 = 0, is
 (2003)
Q.12 Consider the family of circles x2 + y2 = r2, 2< r < 5.
(A) (4, 7) (B) (7, 4) (C) (9, 4) (D) (4, 9) If in the first quadrant, the common tangent to a circle
of this family and the ellipse 4x2 + 25y2 = 100 meets the
Q.5 If the tangent at the point P on the circle x2 + y2 + coordinate axis at A and B, then find the equation of the
6x + 6y = 2 meets the straight line 5x – 2y + 6 = 0 at a locus of the mid points of AB. (1999)
point Q on the y-axis, then the length of PQ is  (2002)
(A) 4 (B) 2 5 (C) 5 (D) 3 5 Q.13 C1 and C2 are two concentric circle the radius of C2
being twice that of C1. From a point P on C2, tangents
PA and PB are drawn to C1. Prove that the centroid of
Q.6 If the circle x2 + y2 + 2x + 2ky + 6 = 0 and x2 + y2 + the triangle PAB lies on C1. (1998)
2ky + k = 0 intersect orthogonally, then k is (2000)
3 3 Q.14 The length of the diameter of the circle which
(A) 2 or – (B) –2 or –
2 2 louches the x-axis at the point (1,0 ) and passes
3 3 through the point ( 2,3) is  (2012)
(C) 2 or (D) –2 or
2 2
10 3 6 5
(A) (B) (C) (D)
3 5 5 3
M a them a ti c s | 9.57

Q.15 The circle through (1, ﹘2) and touching the axis of
x at (3,0) also passes through the point  (2013)
3 3 1 1
(A) (B) (C) (D)
(A) ( 2, −5 ) (B) (5, −2 ) (C) ( −2,5 ) (D) ( −5,2 ) 2 2 2 4

Q.18 The number of common tangents to circles


Q.16 The equation of the circle passing through the x2 + y 2 − 4x − 6y − 12 =
0 and
x2 y 2 x2 + y 2 + 6x − 18y + 26 =
0 , is  (2015)
foci of the ellipse + =1 , and having centre at
16 9
( ) is 
0,3
(2013) (A) 2 (B) 3 (C) 4 (D) 1

2 2 2 2
(A) x + y − 6y + 7 =0 (B) x + y − 6y − 5 =0 Q.19 The centres of those circles which touch the
2 2
(C) x + y − 6y + 5 =0 2 2
(D) x + y − 6y − 7 =0 circle, x2 + y 2 − 8x − 8y − 4 =
0 , externally and also
touch the x-axis, lie on: (2016)
Q.17 let C be the circle with centre at (1,1) and radius (A) An ellipse which is not a circle
=1. If T is the circle centred at ( 0, y ) , passing through
origin and touching the circle C externally, then radius (B) A hyperbola
of T is equal to  (2014)
(C) A parabola

(D) A circle

JEE Advanced/Boards

Exercise 1 2x2 + 2y2 + 3x + 8y + 2c = 0 touches the circle x2 + y2 +


3
Q.1 Let S : x2 + y2 – 8x – 6y + 24 = 0 be a circle and O is 2x – 2y + 1 = 0. Show that either g = or f = 2
4
the origin. Let OAB is the line intersecting the circle at
Q.6 Consider a family of circles passing through two
A and B. On the chord AB a point P is taken. The locus
fixed points A (3, 7) & B (6, 5). The chords in which the
of the point P in each of the following cases.
circle x2 + y2 – 4x – 6y + 3 = 0 cuts the members of the
(i) OP is the arithmetic mean of OA and OB family are concurrent at a point. Find the coordinates
(ii) OP is the geometric mean of OA and OB of this point.
(iii) OP is the harmonic mean between OA and OB Q.7 Find the equation of circle passing through (1, 1)
Q.2 A circle x2 + y2 + 4x – 2 2y + c = 0 is the director belonging to the system of co-axial circles that are
circle of circle S1 and S1 is the director circle of circle S2 tangent at (2, 2) to the locus of the point of intersection
and so on. If the sum of radii of all these circles is 2, of mutually perpendicular tangent to the circle x2 + y2 = 4.
then the value of c is equal to n where n ∈ N. Find
Q.8 The circle C : x2 + y2 + kx + (1 + k)y – (k + 1) = 0
the value of n.
passes through two fixed points for every real number
Q.3 If the circle x2 + y2 + 4x + 22y + a = 0 bisects the k. Find
circumference of the circle x2 + y2 – 2x + 8y – b = 0 (i) the coordinates of these points.
(where a, b > 0), then find the maximum value of (ab).
(ii) the minimum value of the radius of a circle C.
Q.4 Real number x, y satisfies x2 + y2 =1. If the maximum
4−y Q.9 Find the equation of a circle which is co-axial with
and minimum value of the expression z = are M
7−x circles 2x2 + 2y2 – 2x + 6y – 3 = 0 and
and m respectively, then find the value (2M + 6m). x2 + y2 + 4x + 2y + 1 = 0. It is given that the centre of
Q.5 The radical axis of the circle the circle to be determined lies on the radical axis of
these two circles.
x2 + y2 + 2gx + 2fy + c = 0 and
9 .58 | Circle

Q.10 Find the equation of the circle passing through the column-I, the ratio of b/a is
points of intersection of circles x2 + y2 – 4x – 6y – 12 = 0 and
x2 + y2 + 6x + 4y – 12 = 0 and cutting the circle x2 + y2 Column I Column II
– 2x – 4 = 0 orthogonally.
(A) C1 and C2 touch each other
(p) 2 + 2
Q.11 The centre of the circles S = 0 lie on line 2x – 2y
(B) C1 and C2 are orthogonal (q) 3
+ 9 = 0 & S = 0 cuts orthogonally the circle x2 + y2 = 4.
Show that circle S = 0 passes through two fixed points (C) C1 and C2 intersect so that
(r) 2 + 3
& find their coordinates. the common chord is longest
(D) C2 passes through the
Q.12 Find the equation of a circle passing through the (s) 3 + 2 2
centre of C1
origin if the line pair, xy – 3x + 2y – 6 = 0 is orthogonal
to it. If this circle is orthogonal to the circle x2 + y2 – kx (t) 3 – 2 2
+ 2ky – 8 = 0 then find the value of k.
Q.20 A circle with centre in the first quadrant is tangent
Q.13 Find the equation of the circle which cuts the
to y = x + 10, y = x – 6, and the y-axis. Let (h, k) be the
circle x2 + y2 – 14x – 8y + 64 = 0 and the coordinate
axes orthogonally. centre of the circle. If the value of (h + k) = a + b a
where a is a surd, find the value of a + b.
Q.14 Show that the locus of the centres of a circle
which cuts two given circles orthogonally is a straight Q.21 Circles C1 and C2 are externally tangent and they
line & hence deduce the locus of the centres of the are both internally tangent to the circle C3. The radii
circles which cut the circles x2 + y2 + 4x – 6y + 9 = 0 & of C1 and C2 are 4 and 10, respectively and the centres
x2 + y2 – 5x + 4y + 2 = 0 orthogonally. Intercept the locus. of the three circles are collinear. A chord of C3 is also
a common internal tangent of C1 and C2. Given that
Q.15 Find the equation of a circle which touches the m n
line x + y = 5 at the point (–2, 7) and cuts the circle the length of the chord is where m, n and p are
p
x2 + y2 + 4x – 6y + 9 = 0 orthogonally. positive integers, m and p are relatively prime and n is
not divisible by the square of any prime, find the value
Q.16 Find the equation of the circle passing through
of (m + n + p).
the point (–6, 0) if the power of the point (1, 1) w.r.t. the
circle is 5 and it cuts the circle x2 + y2 – 4x – 6y – 3 = 0 Q.22 Find the equation of the circle passing through
orthogonally. the three points (4, 7), (5, 6) and (1, 8). Also find the
coordinates of the point of intersection of the tangents
Q.17 As shown in the figure, the five circles are tangent
to the circle at the points where it is cut by the straight
to one another consecutively and to the lines L1 and
line 5x + y + 17 = 0.
L2. If the radius of the largest circle is 18 and that of
the smaller one is 8, then find the radius of the middle Q.23 The line 2x – 3y + 1 = 0 is tangent to a circle
circle. S = 0 at (1, 1). If the radius of the circle is 13 . Find the
equation of the circle S.
L1
Q.24 Find the equation of the circle which passes
through the point (1, 1) & which touches the circle
x2 + y2 + 4x – 6y – 3 = 0 at the point (2, 3) on it.
L2

Q.18 Find the equation of a circle which touches the line


7x2 – 18xy + 7y2 = 0 and the circle x2 + y2 – 8x – 8y = 0
and is contained in the given circle.

Q.19 Consider two circle C1 of radius ‘a’ and C2 of radius


‘b’ (b > a) both lying in the first quadrant and touching
the coordinate axes. In each of the conditions listed in
M a them a ti c s | 9.59

Exercise 2 Q.8 A rhombus is inscribed in the region common to


the two circles x2 + y2 – 4x – 12 = 0 and x2 + y2 + 4x –
Single Correct Choice Type 12 = 0 with two of its vertices on the line joining the
centres of the circles. The area of the rhombus is
Q.1 B and C are fixed points having co-ordinates
(3, 0) and (–3, 0) respectively. If the vertical angle BAC (A) 8 3 sq. units (B) 4 3 sq. units
is 90°, then the locus of the centroid of the ∆ABC has (C) 16 3 sq. units (D) None of these
the equation :
(A) x2 + y2 = 1 (B) x2 + y2 = 2 Q.9 From (3, 4) chords are drawn to the circle x2 + y2
– 4x = 0. The locus of the mid points of the chords is:
(C) 9(x2 + y2) = 1 (D) 9(x2 + y2) = 4
(A) x2 + y2 – 5x – 4y + 6 = 0
Q.2 Number of points in which the graphs of |y| = x + 1 (B) x2 + y2 + 5x – 4y + 6 = 0
and (x – 1)2 + y2 = 4 intersect, is
(C) x2 + y2 – 5x + 4y + 6 = 0
(A) 1 (B) 2 (C) 3 (D) 4
(D) x2 + y2 – 5x – 4y – 6 = 0

Q.3 y - 1 = m1(x – 3) and y – 3 = m2(x – 1) are two family


of straight lines, at right angles to each other. The locus Q.10 The line joining (5, 5) to (10 cos θ, 10 sin θ) is
of their point of intersection is divided internally in the ratio 2 : 3 at P. If θ varies then
the locus of P is :
(A) x2 + y2 – 2x – 6y + 10 = 0
(A) A pair of straight lines
(B) x2 + y2 – 4x – 4y + 6 = 0
(B) A circle
(C) x2 + y2 – 2x – 6y + 6 = 0
(C) A straight line
(D) x2 + y2 – 4x – 4y – 6 = 0
(D) A second degree curve which is not a circle
Q.4 The points (x1, y1), (x2, y2), (x1, y2) and (x2, y1) are always:
Q.11 The normal at the point (3, 4) on a circle cuts the circle
(A) Collinear (B) Concyclic
at the point (–1, –2). Then the equation of the circle is:
(C) Vertices of a square (D) Vertices of a rhombus
(A) x2 + y2 + 2x – 2y – 13 = 0

Q.5 Consider 3 non-collinear points A, B, C with (B) x2 + y2 – 2x – 2y – 11 = 0


coordinates (0, 6), (5, 5) and (–1, 1) respectively. (C) x2 + y2 – 2x + 2y + 12 = 0
Equation of a line tangent to the circle circumscribing (D) x2 + y2 – 2x – 2y + 14 = 0
the triangle ABC and passing through the origin is
(A) 2x – 3y = 0 (B) 3x + 2y = 0 Q.12 The shortest distance from the line 3x + 4y = 25
(C) 3x – 2y = 0 (D) 2x + 3y = 0 to the circle x2 + y2 = 6x – 8y is equal to
7 9 11 32
(A) (B) (C) (D)
Q.6 A (1, 0) and B(0, 1) and two fixed points on the circle 5 5 5 5
x2 + y2 = 1. C is a variable point on this circle. As C moves, π
the locus of the orthocenter of the triangle ABC is Q.13 The equation of a line inclined at an angle to
4
(A) x2 + y2 – 2x – 2y + 1 = 0 the axis X, such that the two circles x2 + y2 = 4 and x2 +
(B) x2 + y2 – x – y = 0 y2 – 10x – 14y + 65 = 0 intercept equal lengths on it, is
(C) x2 + y2 = 4 (A) 2x – 2y – 3 = 0 (B) 2x – 2y + 3 = 0
(D) x2 + y2 + 2x – 2y + 1 = 0 (C) x – y + 6 = 0 (D) x – y – 6 = 0

Q.7 A straight line with slope 2 and y - intercept 5 Q.14 The locus of the midpoint of a line segment that
touches the circle, x2 + y2 + 16x +12y + c =0 at a point is drawn from a given external point P to a given circle
Q. Then the coordinates of Q are with centre O (where O is origin) and radius r, is
(A) (–6, 11) (B) (–9, –13) (A) A straight line perpendicular to PO
(C) (–10, –15) (D) (–6, –7) (B) A circle with centre P and radius r
9 .60 | Circle

(C) A circle with centre P and radius 2r Q.19 The circles x2 + y2 + 2x + 4y – 20 = 0 and x2 + y2
r + 6x – 8y + 10 = 0
(D) A circle with centre at the midpoint PO and radius
2 (A) Are such that the number of common tangents on
Multiple Correct Choice Type them is 2
(B) Are not orthogonal
Q.15 Locus of the intersection of the two straight lines
passing through (1, 0) and (–1, 0) respectively and (C) Are such that the length of their common tangent
1
including an angle of 45° can be a circle with  12  4
is 5  
(A) Centre (1, 0) and radius 2 .  5 
(B) Centre (1, 0) and radius 2. (D) Are such that the length of their common chord is
(C) Centre (0, 1) and radius 2. 3
5 .
2
(D) Centre (0, –1) and radius 2.
Q.20 Three distinct lines are drawn in a plane. Suppose
Q.16 Consider the circles there exist exactly n circles in the plane tangent to all
the three lines, then the possible values of n is/are
S1 : x2 + y2 + 2x + 4y + 1 = 0
(A) 0 (B) 1 (C) 2 (D) 4
S2 : x2 + y2 – 4x + 3 = 0
S3 : x2 + y2 + 6y + 5 = 0 Q.21 The equation of a circle C1 is x2+y2+14x–4y + 28=0.
Which of this following statement are correct? The locus of the point of intersection of orthogonal
tangents to C1 is the curve C2 and the locus of the point
(A) Radical centre of S1, S2 and S3 lies in 1st quadrant.
of intersection of perpendicular tangents to C2 is the
(B) Radical centre of S1, S2 and S3 lies in 4st quadrant. curve C3 then the statement (s) which hold good?
(C) Radical centre of S1, S2 and S3 orthogonally is 1. (A) C3 is a circle
(D) Circle orthogonal to S1, S2 and S3 has its x and y (B) Area enclosed by C3 is 100π sq. unit
intercept equal to zero.
(C) Area of C2 is 2 times the area of C1.
(D) C2 and C3 are concentric circles.
Q.17 Consider the circles
C1 : x2 + y2 – 4x + 6y + 8 = 0
Q.22 The circles x2 + y2 – 2x – 4y + 1 = 0 and x2 + y2 +
C2 : x + y – 10x – 6y + 14 = 0
2 2 4x + 4y – 1 = 0
Which of the following statement (s) hold good in (A) Touch internally
respect of C1 and C2?
(B) Touch externally
(A) C1 and C2 are orthogonal.
(C) Have 3x + 4y – 1 = 0 as the common tangent at the
(B) C1 and C2 touch each other. point of contact.
(C) Radical axis between C1 and C2 is also one of their (D) have 3x + 4y + 1 = 0 as the common tangent at the
common tangent. point of contact.
(D) Middle point of the line joining the centres of C1
and C2 lies on their radical axis. Q.23 Which of the following is/are True? The circles x2
+ y2 – 6x – 6y + 9 = 0 and x2 + y2 + 6x + 6y + 9 = 0 are
such that
Q.18 A circle passes through the points (–1 , 1), (0, 6)
and (5, 5). The point (s) on this circle, the tangent (s) (A) They do not intersect.
at which is/are parallel to the straight line joining the
(B) They touch each other.
origin to its centre is/are:
(C) Their direct common tangents are parallel.
(A) (1, –5) (B) (5, 1) (C) (–5, –1) (D) (–1, 5)
(D) Their trannsverse common tangents are
perpendicular.
M a them a ti c s | 9.61

Q.24 Two circles x2 + y2 + px + py – 7 = 0 and x2 + y2 Q.30 Let A(x1, y1), B(x2, y2) and C(x3, x3) are the vertices
– 10x + 2py + 1 = 0 intersect each other orthogonally of a triangle ABC.
then the value of p is
Statement-I : If angel C is obtuse then the quantity
(A) 1 (B) 2 (C) 3 (D) 5 (x3 – x1)(x3 – x2) + (y3 – y1) (y3 –y2) is negative.
Statement-II: Diameter of a circle subtends obtuse
Q.25 Which of the following statements is/are incorrect? angle at any point lying inside the semicircle.
(A) Two circles always have a unique common normal.
(B) Radical axis is always perpendicular bisector to the Q.31 Let C be a circle with centre ‘O’ and HK is the
line joining the centres of two circles. chord of contact of pair of the tangents from point
A. OA intersects the circle C at P and Q and B is the
(C) Radical axis is nearer to the centre of circle of smaller midpoint of HK, then
radius.
Statement-I: AB is the harmonic mean of AP and AQ.
(D) Two circles always have a radical axis.
Statement-II: AK is the Geometric mean of AB and AO
and OA is the arithmetic mean of AP and AQ.
Assertion Reasoning Type
(A) Statement-I is true, statement-II is true and Comprehension Type
statement-II is correct explanation for statement-I.
(B) Statement-I is true, statement-II is true and Paragraph for questions 32 to 34
statement-II is NOT the correct explanation for statement-I. Let A, B, C be three sets of real numbers (x, y) defined as
(C) Statement-I is true, statement-II is false. A : {(x, y): y ≥ 1}
(D) Statement-I is false, statement-II is true. B : {(x, y): x2 + y2 – 4x – 2y – 4 = 0}

Q.26 Consider the lines L : (k + 7)x – (k – 1)y – 4(k – 5)=0 C : {(x, y): x + y = 2}
where k is a parameter and the circle
C : x2 + y2 + 4x + 12y – 60 = 0 Q.32 Number of elements in the A ∩ B∩ C is

Statement-I: Every member of L intersects the circle ‘C’ (A) 0 (B) 1 (C) 2 (D) infinite
at an angle of 90°
Q.33 (x + 1)2 + (y – 1)2 + (x – 5)2 + (y – 1)2 has the value
Statement-II: Every member of L tangent to the circle C.
equal to

Q.27 Statement-I: Angle between the tangents drawn (A) 16 (B) 25 (C) 36 (D) 49
from the point P(13, 6) to the circle S : x2 + y2 – 6x +
8y – 75 = 0 is 90° . Q.34 If the locus of the point of intersection of the pair
of perpendicular tangents to the cirlc B is the curve S
Statement-II: Point P lies on the director circle of S.
then the area enclosed between B and S is

Q.28 Statement-I: From the point (1, 5) as its centre, (A) 6π (B) 8π (C) 9π (D) 18π
only one circle can be drawn touching the circle
x2 + y2 – 2x = 7. Paragraph for questions 35 to 36
Statement-II: Point (1, 5) lies outside the circle Consider a circle x2 + y2 = 4 and a point P(4, 2). θ denotes
x2 + y2 – 2x = 7. the angle enclosed by the tangents from P on the circle
and A, B are the points of contact of the tangents from
Q.29 Statement-I: Let C1 (0, 0) and C2(2, 2) be centres P on the circle.
of two circle and L : x + y –2 = 0 is their common chord.
Q.35 The value of θ lies in the interval
If length of common chord is equal to 2 , then both
circles intersect orthogonally. (A) (0, 15º) (B) (15º, 30º)
Statement-II: Two circles will be orthogonal if their (C) (30º, 45º) (D) (45º, 60º)
centres are mirror images of each other in their common
chord and distance between centres is equal to length
of common chord.
9 .62 | Circle

Q.36 The intercept made by a tangent on the x-axis is Q.4 Let PQ and RS be tangents at the extremities of the
diameter PR of a circle of radius r If PS and RQ intersect
9 10 11 12
(A) (B) (C) (D) at a point X on the circumference of the circle, then 2r
4 4 4 4 equals  (2001)
PQ + RS
(A) PQ ⋅ RS (B)
Paragraph for questions 37 to 39 2

Consider the circle S : x2 + y2 – 4x – 1 = 0 and the line L 2PQ ⋅ RS PQ2 + RS2


(C) (D)
: y = 3x – 1. If the line L cuts the circle at A and B then PQ + RS 2

Q.37 Length of the chord AB equal Q.5 Let AB be a chord of the circle x2 + y2 = r2 subtending
a right angle at the centre. Then the locus of centroid of
(A) 2 5 (B) 5 (C) 5 2 (C) 10 the triangle PAB as P moves on the circle is (2001)
(A) A parabola (B) A circle
Q.38 The angle subtended by the chord AB in the
minor arc of S is (C) An ellipse (D) A pair of straight lines

3π 5π 2π π
(A) (B) (C) (D) Q.6 If two distinct chords, drawn from the point
4 6 3 4
(p, q) on the circle x2 + y2 = px + qy (where pq ≠ 0) are
bisected by the x-axis, then (1999)
Q.39 Acute angel between the line L and the circle S is
(A) p2 = q2 (B) p2 = 8q2
π π π π
(A) (B) (C) (D)
2 3 4 6 (C) p2 <8 q2 (D) p2 > 8q2

Q.7 Consider
Previous Years’ Questions L1 : 2x + 3y + p – 3 = 0
L2 : 2x + 3y + p + 3 = 0
Q.1 Tangents drawn from the point P (1, 8) to the circle
where p is a real number and
x2 + y2 – 6x – 4y – 11 = 0 touch the circle at the point A
and B. The equation of the circumcircle of the triangle C : x2 + y2 – 6x + 10y + 30 = 0 (2008)
PAB is (2009)
Statement-I: If line L1 is a chord of circle C, then line L2
(A) x2 + y2 + 4x – 6y + 19 = 0 is not always a diameter of circle C. and
(B) x2 + y2 – 4x – 10y + 19 = 0
Statement-II: If line L1 is a diameter of circle C, then line
(C) x2 + y2 – 2x + 6y – 29 = 0 L2 is not a chord of circle C.
(D) x2 + y2 – 6x – 4y + 19 = 0
Paragraph 1: Let ABCD be a square of side length 2
unit. C2 is the circle through vertices A, B, C, D and C1 is
Q.2 Let ABCD be a quadrilateral with area 18, with side
the circle touching all the sides of square ABCD. L is the
AB parallel to the side CD and AB = 2CD. Let AD be
line through A. (2006)
perpendicular to AB and CD. If a circle is drawn inside
the quadrilateral ABCD touching all the sides, its radius
is  (2007) Q.8 If P is a point of C1 and Q is a point on C2, then
3 PA2 + PB2 + PC2 + PD2
(A) 3 (B) 2 (C) (D) 1 is equal to
2 QA2 + QB2 + QC2 + QD2
Q.3 The locus of the centre of circle which touches (A) 0.75 (B) 1.25 (C) 1 (D) 0.5
(y – 1)2 + x2 = 1 externally and also touches x axis, is
 (2005)
Q.9 A circle touches the line L and the circle C1 externally
(A) {x2 = 4y , y ≥ 0}∪{(0, y), y < 0}
such that both the circle are on the same side of the
(B) x2 = y line, then the locus of centre of the circle is
(C) y = 4x2 (A) Ellipse (B) Hyperbola
(D) y2 = 4x ∪ (0, y), y∈R (C) Parabola (D) Parts of straight line
M a them a ti c s | 9.63

Q.10 A line M through A is drawn parallel to BD. Point S Q.15 Let T1, T2 and be two tangents drawn from
moves such that its distances from the line BD and the (–2, 0) onto the circle C : x2 + y2 = 1. Determine the
vertex A are equal. If locus of S cuts M at T2 and T3 and circles touching C and having T1, T2 as their pair of
AC at T1. then area of ∆T1T2T3 is tangents. Further, find the equations of all possible
1 common tangents to these circles when taken two at a
2
(A) sq unit (B) sq unit time. (1999)
2 3
(C) 1 sq unit (D) 2 sq unit
Q.16 Two parallel chords of a circle of radius 2 are at
a distance 3 + 1 apart. If the chords subtend at the
Paragraph 2: A circle C of radius 1 is inscribed in an
π 2π
equilateral triangle PQR. The points of contact of C with center, angles of and , where k > 0 , then the
the sides PQ, QR, RP are D, E, F respectively. The line k k
value of k  is
PQ is given by the equation 3 x + y – 6 = 0 and the [Note : k  denotes the largest integer less than or
3 3 3 equal to k ]  (2010)
point D is  ,  . Further, it is given that the origin
 2 2
 
and the centre of C are on the same side of the line PQ. Q.17 The circle passing through the point ( −1,0 ) and
 (2008) touching the y − axis at ( 0,2 ) also passes through the
point (2011)
Q.11 The equation of circle C is
 3   5 
(A) (x – 2 3 )2 + (y – 1)2 = 1 (A)  − ,0  (B)  − ,2 
2  2   2 
 1
(B) (x – 2 3 ) +  y +  = 1
2
2  3 5
 (C)  − ,  (D) ( −4,0 )
(C) (x – 3 ) + (y + 1)2 = 1
2  2 2

(D) (x – 3 )2 + (y – 1)2 = 1
Paragraph 3: A tangent PT is drawn to the circle
Q.12 Point E and F are given by x2 + y 2 =
4 at the point P ( )
3,1 . A straight line

 3 3  3 1 L, perpendicular to PT is a tangent to the circle


(A)  , ,
 2 2
 
( 3,0 ) (B)  , ,
 2 2
 
( 3,0 ) ( x − 3)
2
1.
+ y2 =

 3 3  3 1 3 3  3 1 Q. 18 A common tangent of the two circles is  (2012)


(C)  ,  , ,  (D)  ,  , , 
 2 2  2 2 2 2   2 2
        (A) x = 4 (B) y = 2

Q.13 Equations of the sides QR, RP are (C) x + 3y =


3 (D) x + 2 2 y =
6

2 2
(A) y = x + 1, y = – x –1 Q.19 A possible equation of L is  (2012)
3 3
1 (A) x − 3y =
1 (B) x + 3y =
1
(B) y = x, y = 0
3 (C) x − 3y =
−1 (D) x + 3y =
5
3 3
(C) y = x + 1, y = – x–1 2
2 2 Q.20 Let S be the focus of the parabola y = 8x
and let PQ be the common chord of the circle
(D) y = 3 x, y = 0
x2 + y 2 − 2x − 4y =
0 and the given parabola. The area
of the triangle PQS is  (2012)
Q.14 Let 2x2 + y2 – 3xy = 0 be the equation of a pair of
tangents drawn from the origin O to a circle of radius 3 Q.21 The locus of the mid-point of the chord of contact
with centre in the first quadrant. If A is one of the points of tangents drawn from points lying on the straight line
of contact, find the length of OA.  (2001) 2 2
4x – 5y = 20 to the circle x + y =9 is (2012)
9 .64 | Circle

(A) 20(x2 + y2) ﹘ 36x + 45y = 0 1 1 2


(A) f  2  ≥ f (1 ) (B) f  3  ≤ f 
(B) 20(x2 + y2) + 36x ñ 45y = 0     3
(C) 36(x2 + y2) ﹘ 20x + 45y = 0
f ' (3) f ' (2)
(D) 36(x2 + y2) + 20x ñ 45y = 0 (C) f ( 2 ) ≤ 0 (D) ≥
f (3) f (2)

Q.22 Circle(s) touching x-axis at a distance 3 from the


Q.26 The circle C1 : x2 + y 2 =
3 , with centre O, intersects
origin and having an intercept of length 2 7 on y-axis 2
the parabola x = 2y at the point P in the first quadrant.
is (are) (2013)
Let the tangent to the circle C1 at P touches other two
2 2
(A) x + y − 6x + 8y + 9 =0 circles C2 and C3 at R 2 and R 3 , respectively. Suppose
2 2
(B) x + y − 6x + 7y + 9 =0 C2 and C3 have equal radii 2 3 and centres Q2 and
2 2 Q3 respectively. If Q2 and Q3 lie on the y-axis, then
(C) x + y − 6x − 8y + 9 =0
 (2016)
2 2
(D) x + y − 6x − 7y + 9 =0
(A) Q2Q3 = 12

Q.23 The common tangents to the circle x + y = 2 2 2 (B) R 2R3 = 4 6


2
and the parabola y = 8x touch the circle at the points
(C) Area of the triangle OR 2R3 is 6 2
P, Q and the parabola at the points R, S. Then the area
of the quadrilateral PQRS is  (2014) (D) Area of the triangle PQ2Q3 is 4 2
(A) 3 (B) 6 (C) 9 (D) 15
2 2
Q.27 Let RS be the diameter of the circle x + y = 1,
Q.24 A circle S passes through the point (0, 1) and where S is the point (1, 0). Let P be a variable point
is orthogonal to the circles ( x − 1 ) + y 2 = (other than R and S) on the circle and tangents to the
2
16 and
x2 + y 2 =
1 . Then (2014) circle at S and P meet at the point Q. The normal to the
circle at P intersects a line drawn through Q parallel to
(A) Radius of S is 8 (B) Radius of S is 7 RS at point E. Then the locus of E passes through the
(C) Centre of S is (-7, 1) (D) Centre of S is (-8, 1) point(s)  (2016)
1 1 
(A)  , 
Q.25 Let 3 3 
x
 n 1 1
 nn ( x + n)  x + n  .....  x + n   (B)  , 
  2  n   4 2
f ( x ) = lim   ,
 n! x2 + n2  x2 + n  ....  x2 + n  
2 2
( )
n→ ∞
    1 1 
 4   n2   (C)  , − 
   3 3
for all x > 0. Then (2016)
1 1
(D)  , − 
 4 2
M a them a ti c s | 9.65

PlancEssential Questions
JEE Main/Boards JEE Advanced/Boards

Exercise 1 Exercise 1
Q.12 Q.18 Q.21 Q.5 Q.9 Q.14
Q.23 Q.29 Q.17 Q.19 Q.21 Q.24

Exercise 2 Exercise 2
Q.3 Q.7 Q.14 Q.2 Q.4 Q.9
Q.15 Q.20 Q.13 Q.16 Q.21
Q.22 Q.25 Q.27
Previous Years’ Questions
Q.22 Q.25 Q.26
Q.1 Q.3 Q.5 Q.29 Q.32
Q.8 Q.11 Q.13
Previous Years’ Questions
Q.1 Q.3 Q.6
Q.7 Q.9 Q.13

Answer Key

JEE Main/Boards Q.12 x = 3 and y = ±3.

Exercise 1 Q.14 x2 + y2 + 2ax + 2py – (b2 + q2) =0; a2 + b2 + p2 + q2

Q.15 x2 + y2 + 18x – 2y + 32 = 0
Q.1 x2 + y2 + 3x + 12y + 2 = 0
Q.16 32 sq. units
Q.3 (1, –1), 13 , x2 + y2 – 2x + 2y – 11 = 0
Q.17 x2 + y2 – 12x – 10y + 52 = 0
Q.4 3x - 4y = 7, 4x + 3y = 0
Q.20 4 2
Q.6 13x + 9y = 77, 3x – y – 27 = 0
1
Q.21 cos–1
Q.7 2x2 + 2y2 + 16x – 8y – 41 = 0 2 2
Q.8 23x2 + 23y2 – 156x + 38y + 168 = 0 Q.22 x2 + y2 – 2x – 6y – 8 =0

Q.9 y = x Q.23 8 sq. units.

Q.10 x = 3 Q.26 (2, –1) ; 2


109 Q.28 4(x2 + y 2 ) + 2y − 29 =
0
Q.11
2
9 .66 | Circle

Q.29 x2 + y 2 + 8x − 6y + 9 =0 Q.30 x2 + y2 – x = 0 Q.31 2 a2 = b2

Exercise 2
Single Correct Choice Type

Q.1 D Q.2 B Q.3 A Q.4 B Q.5 A Q.6 D

Q.7 D Q.8 B Q.9 C Q.10 C Q.11 A Q.12 D

Q.13 C Q.14 A Q.15 B Q. 16 C Q.17 A Q.18 D

Q.19 C Q.20 D Q.21 A

Previous Years’ Questions

Q.1 D Q.2 B Q.3 C Q.4 A Q.5 C Q.6 A

Q.7 C Q.8 B Q.10 2x2 + 2y2 – 10x – 5y + 1 = 0 Q.11 (a,b) and (0,0)

Q.12 4x2+25y2=4x2y2 Q.14 A Q.15 B Q.16 D Q.17 D

Q.18 B Q.19 C

JEE Advanced/Boards

Exercise 1
Q.1 (i) x2 + y2 – 4x – 3y = 0, (ii) x2 + y2 = 24, (iii) 4x + 3y = 24 Q.2 32

 23 
Q.3 625 Q.4 4 Q.6  2, 
 3 
1 1 1
Q.7 x2 + y2 – 3x – 3y + 4 = 0 Q.8 (1, 0)&  ,  ; r = Q.9 4x2 + 4y2 + 6x + 10y – 1 = 0
2 2 2 2
 1 1
Q.10 x2 + y2 + 16x + 14y – 12 = 0 Q.11 (–4, 4);  − ,  Q.12 x2 + y2 + 4x – 6y = 0; k= 1;
 2 2

Q.13 x2 + y2 = 64 Q.14 9x – 10y + 7 =0; radical axis Q.15 x2 + y2 + 7x – 11y + 38 = 0

Q.16 x2 + y2 + 6x – 3y = 0 Q.17 12 Q.18 x2 + y2 – 12x –12y + 64 = 0

Q.19 (A) S; (B) R; (C) Q ; (D) P Q.20 10 Q.21 19

Q.22 (–4, 2), x2 + y2 – 2x – 6y – 15 = 0

Q.23 x2+ y2 – 6x + 4y = 0 OR x2 + y2 + 2x – 8y + 4 = 0 Q.24 x2 + y 2 +x – 6y + 3 = 0


M a them a ti c s | 9.67

Exercise 2
Single Correct Choice Type

Q.1 A Q.2 C Q.3 B Q.4 B Q.5 D Q.6 A


Q.7 D Q.8 A Q.9 A Q.10 B Q.11 B Q.12 A
Q.13 A Q.14 D

Multiple Correct Choice Type

Q.15 C, D Q.16 B, C, D Q.17 B, C Q.18 B, D Q.19 A, C, D Q.20 A, C, D


Q.21 A, B, D Q.22 B, C Q.23 A, C, D Q.24 B, C Q.25 A, B, D

Assertion Reaosing Type

Q.26 C Q.27 A Q.28 D Q.29 A Q.30 A Q.31 A

Comprehension Type

Paragraph 1: Q.32 B Q.33 C Q.34 C


Paragraph 2: Q.35 D Q.36 B
Paragraph 3: Q.37 D Q.38 A Q.39 C

Previous Years’ Questions

Q.1 B Q.2 B Q.3 A Q.4 A Q.5 B Q.6 D


Q.7 C Q.8 A Q.9 C Q.10 C Q.11 D Q.12 A
2 2
 4 1 5  4
Q.13 D Q.14 3(3 + 10 ) Q.15  X +  + y =   ; y = ± x +  Q.16 3
 3 3 39  5

Q.17 D Q.18 D Q.19 A Q.20 4 Q.21 A Q.22 C
Q.23 D Q.24 B C Q.25 A C D Q.26 C Q.27 A C
9 .68 | Circle

⇒ 3y + 12 = 4x + 12
Solutions
⇒ 4x − 3y =
0

JEE Main/Boards ∴ Slope of tangent at P is


−1
=
−3
4/3 4
Exercise 1 ∴ Equation of tangent is ( y + 4=
−3
) 4
( x + 3)
Sol 1: ∴ Centre lies on 2x – y – 3 = 0 ⇒ 4y + 16 =−3x − 9
∴ Let the centre be C ≡ (h, 2h –3) ⇒ 3x + 4y + 25 =
0
It also passes through A ≡ (3, –2) and B ≡ (–2, 0)
∴ AC = BC Sol 5: Equation of tangent at (1, –2) is

⇒ (h – 3)2 + (2h – 1)2 = (h + 2)2 + (2h – 3)2 x x1 + y y1 = a2

⇒ – 6h + 9 – 4h + 1 = 4h + 4 – 12h + 9 – 2h =3 x – 2y = 5

3  −3  1 5
∴h=– ∴
= C  , −6  ∴y = x–
2 2 2
 2 
∴ Equation of the circle is Equation of C2 is (x – 4)2 + (x + 3)2 = ( 5 )2

(x – h)2 + (y – k)2 = R2 Now the tangent will touch C2 If c2 = r2(1 + m2)


2
2 2 5
 3  −3  c2 =  
⇒  x +  + (y + 6)2 =  , − 3  + ( −6 + 2 )
2
2
 2   2  2
 1 5
r (1 + m ) = 5 ×  1 +  =  
2 2

⇒ x2 + 3x +
9
+ y2 + 12y + 36 =
81
+16  4 2
4 4 ∴The given line is tangent to C2
x2 + y2 + 3x + 12y + 2 = 0
Sol 6: Equation of circle is
Sol 2: We can see that (0, 0), (1, 1) & (6, –4) form a right C ≡ (x – 1)2 + (y + 4)2 = (2 10 )2
angled triangle with (0, 0) & (6, –4) as diameter
Shifting origin to (1, –4)
Equation of circle is (x – 0) (x – 6) + y(y + 4) = 0
∴ C’ ≡ X2 + Y2 = (2 10 )2 & P = (7, 1)
⇒ C = x(x – 6) + y (y + 4) = 0
∴ Y – 1 = m(X – 7)
We can see that (5, –5) satisfies this equation
∴Y = mX + (1 – 7m)
∴ 4 points are concyclic
∴c2 = a2(1 + m2)
Sol3: A ≡ (–1, 2), and B ≡ (3, –4) (1 − 7m)
2
(
= 40 1 + m2 )
Equation of the circle is (x + 1) (x – 3) + (y – 2) (y + 4) ⇒ 9m – 14m – 39 = 0
2

⇒ x + y – 2x + 2y – 11 = 0
2 2
⇒ 9m2 – 27m + 13m – 39 = 0
∴ C =( −g, − f ) =(1, − 1 ) −13
m = 3 or m =
9
Radius = g2 + f 2 − c = 1 + 1 + 11 = 13
Since slope remains same in both system
∴Equation of lines in old co-ordinates are
Sol 4: Given equation of circle is x2 + y2 = 25 P ≡ ( −3 , − 4 )
−13
0+4 4 (y + 3) = 3(x – 8) &(y + 3) = (x – 8)
∴ Slope of normal=
OP = 9
0+3 3
Or 3x − y − 27 =0 and 13x + 9y =
77
4
∴ Equation of normal is ( y + 4 )= ( x + 3)
3
M a them a ti c s | 9.69

Sol 7: Centre = (– 4, 2) Sol 12: C1 ≡ x2 + y2 = 9


Tangent is x – y = 3 Centre = (0, 0 & R1 = 3)
−4 − 2 − 3  9  C2 ≡ x2 + y2 – 12x + 27 = 0
∴Radius = = 
2  2 Centre = (6, 0 & R2 = 3)
81 ∴ The circles touch each other externally
∴C ≡ (x + 4)2 + (y – 2)2 =
2
T2 T1
3
Sol 8: Using the concept of family of circles, let the
equation of circle be

(x 2
) ( )
+ y 2 − 8x − 2y + 7 + λ x2 + y 2 − 4x + 10 y + 8 =
0
-3
3
6
T3
As (3, -3) lies on it
∴ ( 9 + 9 − 24 + 6 + 7 ) + λ ( 9 + 9 − 12 − 30 + 8 ) =0 ∴ The equation of tangents are
⇒ 7 − 16 λ =0 y = 3, x = 3 & y = –3 (from figure itself)
7
⇒λ=
16 Sol 13: Family of circles passing through two points is
∴ Equation of the circle is (x – x1) (x – x2) + (y – y1) (y –y2) + λL = 0

(x2 + y 2 − 8x − 2y + 7 + )
7 2
16
(
x + y 2 − 4x + 10y + 8 = 0 ) ∴x(x – 1) + y2 + ly = 0
∴x2 + y2 – x + ly = 0
= 023 x2 + 23y 2 − 156 x + 38y + 168 =
or 0
 1 −λ 
Centre =  , 
2 2 
Sol 9: C ≡ x2 + y2 – 4x = 0 Now since the circle touches internally [∵ (0, 0), & (1, 0)
Centre = (2, 0) lie inside the circle]
1−0 ∴r1 – r2 = distance between their centres
Slope of line perpendicular to chord = = −1
1−2
1 λ2 1 λ2
∴ Slope of chord = 1 ∴3 – + = +
4 4 4 4
⇒ y – 1 = 1 (x – 1)
 1 + λ2 
∴ y = x is the equation of chord ∴9 = 4  
 4 
 
Alternative 1 
∴λ = ±2 2 ∴Centre =  , ± 2 
Equation of a chord bisected at a given point is T = S1 2 

∴x x1 + y y1 − 2 ( x + x1 ) = x12 + y12 − 4x1 Sol 14: Let the coordinates of diameter be (h1, k1) &
Or, x + y − 2x − 2 = 1 + 1 − 4 (h2, k2)

Or, x – y = 0 ∴Equation of circle is


(x – h1) (x – h2)+(y – k1) (y – k2) = 0
Sol 10: Equation of chord of contact ⇒ x2 – y2 – (h1 + h2)x – (k1 – k2)y + (h1h2+k1k2)=0
xx1 + yy1 + g(x + x1) + f(g+g1) + c = 0
⇒ x2 + y2 – (–2a)x – (–2p)y + (– b2 – q2) = 0
⇒ 6x – 2(x + 6) = 0 ; x = 3
⇒ x2 + y2 + 2ax + 2py – (b2 + q2) = 0

Sol 11: Length of tangent from a point ∴R = a2 + p2 + b2 + q2

= x12 + y12 + 2gx1 + 2fy1 + C


Sol 15: Given equation of line is x + y = 2  …(i)
2 2
4(3) + 4(2) + 4 × 3 + 16 × 2 + 13 109 109 2 2
2 , we get
On solving (i), with x + y =
= = =
4 4 2
9 .70 | Circle

x2 + ( 2 − x ) =
2
2 As circle with center (6,5) touches it externally

⇒ x2 + 4 − 4x + x2 =2 Y

⇒ 2x2 − 4x + 2 =0 (6,5)

⇒ ( x − 1 ) = 0 ⇒ x = 1 This means the line represented


2 C2

by (i) and the circle intersects only at (1, 1) (2,2)

Similarly, on solving x + y = 2 and C1


X
x2 + y 2 + 3x + 3y − 8 =,
0 we get
∴ C1 C2 =+
r1 r2
2x2 − 4x + 4 + 3 ( 2 ) − 8 =0 , we get
⇒ 2x2 − 4x + 2 =0 ⇒ (6 − 2)2 + (5 − 2)2 = 2 + r

⇒ 2 ( x − 1) =
0
2 ⇒ r2 + 4r + 4 = 16 + 9

⇒x =1 ⇒ r2 + 4r – 21 = 0

Hence, the line intersects only at one point (1, 1) ⇒ r2 + 7r – 3r – 21 = 0

Hence, proved. ∴r = 3 ( r cannot be negative)


∴Equation of C2 is (x – 6)2 + (y – 5)2 = 9
Sol 16: Let C ≡ (h, k) be the center of the circle
∴4k = h + 7 Sol 18: Let the equation of circle be

∴ AC = BC x2 + y2 + 2gx + 2fy + c = 0

⇒ (4k – 7 + 3)2 + (k – 4)2 = (4k – 12)2 + (k – 4)2  1


Let  m,  be point on the circle.
 m
(
⇒ ( 4k − 4 ) = 4k − 122
2
) On substitution we get
⇒K=
2
If m4 + 2gm3 + 2fm + cm2 + 1 = 0
∴ C ≡ (1, 2 )
m1, m2, m3, m4 are roots of this equation
Now
A(-3,4) B(5,4) 4y=x+7 then, m1m2m3m4 = 1

Sol 19: Equation of line


C(1,2)
⇒ x cos x + y sin x – p = 0
C D
Now family of circle passing
Equation of chord AB is y – 4 = 0 through the intersection of
the circle & line is
∴Perpendicular distance of center from chord AB is
2−4 x2 + y2 – a2 + λ (x cos x + y sin
=2 x – p) = 0
1
∴ AB = 8 and BC = 2PQ = 4 ∴ Radius of circle = AM
= a2 − p2
∴Area of rectangle = 8 × 4 = 32
2 2
 λ cos x   λ sinx  2
⇒   +  + a + λp = a2 − p2
Sol 17: Radius of C1 = 2  2   2 
∴Centre = (2, 2) λ2
⇒ + lp + p2 = 0
4
(λ + 2p)2 = 0 ⇒ λ = – 2p
∴S ≡ x2 + y2 – 2px cos x
– 2py sin x + 2P2 – a2 = 0
M a them a ti c s | 9.71

Sol 20: Slope of AB = 1 Let θ be the angle subtended at the circumference


∴ Slope of L1 × slope of AB =
−1 ∴ Angle subtended at circumference
⇒ Slope of L1 =
−1 1
= (Angle subtended at centre)
2
1 1
∴ cosθ = ⇒ θ = cos–1
2 2 2 2

Sol 22: Given x2 + y 2 − 2x − 2λy − 8 =0

( )
⇒ x2 + y 2 − 2x − 8 − 2λ ( y ) =0  ….(i)
Let S ≡ x2 + y 2 − 2x − 8 =0 and  …(iii)
0 
L≡ y = …(iii)
∴ The equation is represents a family of circles passing
3 5
And mid-point of AB, M ≡  ,  through the intersection of S = 0 & L = 0.
2 2
∴ On solving (ii) and (iii), we get
∴ Equation of line L1 is
x2 − 2x − 8 =0
 5  3
y −  =−1  x −  2 ± 4 + 32
 2   2  x
⇒= = 4 or − 2
2
Or, (2y - 5) = - (2x - 3) ∴ The fixed point are A (4, 0) and B (-2, 0) from the
Or 2x + 2y – 8 = 0 diagram, the perpendicular bisector of AB is con.
Current with the tangents at P
Or, x + y – 4 = 0
∴ Length of perpendicular from (0, 0) on L1 is

0+0−4
=2 2
2

( )
2
( a)
2
∴ Length of the chord
= 2 − 2 2

( )
2
= 2 16 − 2 2

=4 2

Sol 21: Equation of circle is x2 + y2 – 4y = 0


∴ M ≡ (1, 0 )
∴ Centre = (0, 2) & radius = 2
And Equation of line MP is x = 1  …..(iv)
Perpendicular distance of center from the line x + y = 1 is
∴ On solving (iv) with x + 2y + 5 = 0
0 + 2 −1 1
= We get 1 + 2y + 5 = 0
2 2
−6
⇒ 2y + 6 = 0 ⇒ y = =−3
2
∴ P ≡ (1, − 3 )
Centre of circle (i) is C ≡ (1, λ )
If P is the point of intersection of tangents then CB is
perpendicular to BP
λ −0 0+3
∴ × =−1
1 − 4   4 −1
9 .72 | Circle

λ 3 The equation of radical axes are S1 – S2 = 0


∴ × =−1 ⇒ λ = 3
−3 3 3  5 7−9
∴ S 1 – S2 =  − 4  x – y + =0
∴ Equation of the required circle is 2  2 2

x2 + y 2 − 2x − 6y − 8 =0 ⇒5x + 5y – 5 = 0 ⇒ x + y – 1 = 0
and S1 – S3 = 0 ⇒ – 4x – y + 7 = 0
Sol 23: Length of tangent = S11 4x + y – 7 = 0

∴ QP = 42 + 52 − 42 − 10 − 11 = 2 ∴The radical centre is (2, –1)


Length of tangent = S1 = 22 + 12 − 8 + 7 =2
P
Sol 27: Let (h, k) be the point on circle x2 + y2 = a2
Q S ⇒ ∴ h2 + k 2 =
a2  …(i)
Equation of chord of contact for x2 + y 2 =is
b2
R b2 …(ii)
hx + ky =

1 As (ii) touches the circle x2 + y 2 =


c2
Area of PQRS = 2DPQS = 2 × × PS x QPPS
2 −b2
∴ c
=
= Radius of circle = 22 + 12 + 11 = 4 h2 + k 2

∴ Area of PQRS = 4 × 2 = 8 ⇒ b2 =
ac
∴ a, b and c are in G.P.
Sol 24: The equation of any curve passing through the
intersection of Sol 28: Let the required circle be
L1 ≡ a1x + b1y + c1 = 0 x2 + y2 + 2gx + 2fy + c = 0  … (i)
L2 ≡ a2x + b2y + c2 = 0 The given circles are
L3 ≡ y = 0 & L4 ≡ x = 0 is L1 L2 + λ L3 L 4 x2 + y2 + 3x – 5y + 6 = 0  … (ii)
⇒ (a1x + b1y + c1) (a2x + b2y + c2) + lxy = 0 29
and x2 + y2 – 7x + =0 … (iii)
where λ is a parameter 4
Now 1, 2 & 1, 3 are orthogonal
This curve represents a circle if coeff. of x2 = coeff. of y2
3 −5
∴ a1a2 = b1b2 ∴2g + 2f =c+6
2 2
−7 29
Sol 25: Let any point on c2 be (h, k) 3g – 5f = c + 6 & 2g × + 2f × 0 = c +
2 4
Length of tangent from any point to circle 29
⇒–7g = c +
= S1 4
−5
∴l = h2 + k 2 + 2gh + 2fk + c1 ∴10g – 5f =
4
Now since (h, k) satisfies circle 1 ∴8g – 4f = – 1
∴h + k + 2gh + 2fk = – c
2 2
Equation of circle is
∴l = c1 − c (8g + 1)
x2 + y2 + 2gx + y+c=0
2
Sol 26: The tangents to the these circle are equal in The centre lies on the line
length 3x + 4y + 1 = 0
∴The point is radical centre (8g + 1)
⇒ 3(– g) – 4 +1=0
4
M a them a ti c s | 9.73

⇒ –11g = 0 ∠ OPR
= 90° − ∠ QPR= 45°
1 29
⇒ g = 0, f = and c = – OM
4 4 ∴ In ∆OMP, sin 45° =
∴Equation of circle is OP
4x2 + 4y2 + 2y – 29 = 0 1 a
⇒ =
2 b
Sol 29: Given equation of circle is x 2 + 4x + ( y − 3) =
2
0 ⇒ b =2 a .
0+h 3+k 
Let M ≡ (h, k ) ∴B ≡  , 
 2 2  Sol 32: According to condition
h 3+k 
∴B ≡  ,  S1 S2
2 2 
Y 1
(h, k)
1
2
R 3

(0,3) θ1 + θ2 = 90º
∴tanθ1tanθ2 = 1
B r1 1
X tanθ1 = =
M Length of tangent h2 + k 2 − 1
3
tanθ2 =
As point B lies on the circle
h2 + k 2 − 3
h2 h 3+k 
2
According to condition –
∴ + 4× + − 3 =0
4 2  2  ∴ 3 = (h2 + k2 –1) (h2 + k2 – 3)

h2 k2 9 k 3 3 = (h2 + k2)2 –4 (h2 + k2) + 3


⇒ + 2h + + −2 × =0
4 4 4 2 2 ∴ h2 + k2 = 0
⇒ h2 + k 2 + 8h − 6k + 9 =
0 or h2 + k2 = 4

∴ The value of point B is Now h2 + k2 ≠ 0 as no tangent will be possible.

x 2 + y 2 + 8x − 6y + 9 =0 ∴ The locus of point is a circle

Sol 30: Let (h, k) be middle points Sol 33: Let the other end of diameter be (h, k)

Equation of chord through (h, k) is ∴ Equation of circle is

xh – (x + h) + yk = h2 – 2h + k2 … (i) (x – a) (x – h) + (y – b) (y – k ) = 0

As the chord given by equation (i) passes through (0, 0) a+h b+k 
∴ Center ≡  , 
 2 2 
∴ On substituting, x = 0 and y = 0, we get
Since the circle touches the x-axis
– h = h2 – 2h + k2
∴ | y-coordinate| = radius
∴Locus of midpoint is x2 – x + y2 = 0
2 2
b+k a+h b +k 
⇒ =   +  − (ah + bk)
Sol 31: Given, OM = a and OP = b 2  2   2 
From the diagram, 2
a+h
∠ PRQ =
90°
∴  = (ah + bk)
 2 
And PR = QR
∠ QPR =
∠ PQR =
45°
9 .74 | Circle

∴Locus of point is λ2
Sol 2: (B) S ≡ x2 + y2 + lx + =0
x2 + 2ax + a2 = 4ax + 4by 2
(x – a)2 = 4by λ
2
λ2 λ2
Radius = g2 + f 2 − c =   − = −
2 2 4
Sol 34: Let G be perpendicular from C on AB
∴Radius is not defined for any real value of l
And M be midpoint of CD
Let radius = R Sol 3: (A) For an equilateral B
E triangle inscribed in circle of
radius r, in ∆ OAB using cosine a r
rule, we get
o
120
r O
r 2 + r 2 − a2
cos120º = A
C M D F 2r 2 a/2
⇒ – r2 = 2r2 – a2
A
G O
B ⇒a= 3r
Area of equilateral triangle
3 2 3 3 3 2
∴MO2 + MD2 = OD2 (O is centre) = a = × ( 3 r)2 = r
4 4 4
R2
MO2 = R2 –
4 Radius of given circle = g2 + f 2 − c = 1
3R
MO = 3 3
4 ∴A =
4
3R
⇒ CG =
4 Sol 4: (B) Let the centre of circle be (– h, 0)
R R
AG = AO – GO = AO – CM = R – = where h > 0
2 2
3R 2 R2 Radius = 5
AC2= AG2 + GC2 = +
4 4 ∴ Equation of circle is (x + h)2 + y2 = 25
∴ AC = R It passes through the point (2, 3)
AE AB ∴(h + 2)2 = (4)2 ⇒ h = 2 or h = –6
∴ = (As ∆ AEB  ∆ ACG )
AC AG
But h > 0 ⇒ h = 2 ⇒ ( 2 + h) + 9 =
2
h 2 or − 6
25 ⇒=
AE AB ∴ Equation of the circle is x2 + y2 + 2x – 21 = 0
⇒ = ⇒ AE = 2AB
R R
∴ Intercept made on y-axis = 2 f 2 − c = 2 21
2

Sol 5: (A) S1 : x2 + y2 = 1
Exercise 2
S2 : x2 + y2 – 2x – 6y = 6
Single Correct Choice Type S3 : x2 + y2 – 4x – 12y = 9

Sol 1: (D) The centers are A = (2, 3) ; B = (–1, –2) ; C = r1 = 1; r2 = 12 + 32 + 6 =4; r3= 22 + 62 + 9 =7
(5, 8)
∴r1, r2, r3 are in A.P.
3 − ( −2) 5
∴ Slope of AB = =
2 − ( −1) 3 Sol 6: (D) S1: x2 + y2 + 2lx + 4 = 0
8−3 5 S2: x2 + y2 – 4lx + 8 = 0
and slope AC = =
5−2 3 Since both represent real circles
∴ The three points are collinear
M a them a ti c s | 9.75

∴r1 ≥ 0 & r2 ≥ 0
∴l2 – 4 ≥ 0 ∴ λ ≤ –2 or λ ≥ 2 ... (i)  1
Let  x,  be a point on the circle.
∴4l2 – 8 ≥ 0 ∴λ ≤ – 2 or λ ≥ 2  ... (ii)  x

From 1, 2 λ (– ∞, –2] U [2, ∞) ∴ x4 + 2gx3 + cx2 + 2fx + 1 = 0


1
All of these lie within the range ⇒ abcd = =1
1
abc
Sol 7: (D) s = x2 + y2 + 16x – 24y + 183 = 0 Sol 11: (A) Circumradius R = , where ∆ is the area
4∆
Centre ≡ (–8, 12) Radius = 5 of a triangle
12 × 12 × 6
Let (x1, y1) be the image of (–8, 12) w.r.t. to the line ⇒ R=
1 
4x + 7y + 13 = 0 4 ×  × 6 × height 
2 
x1 − ( −8) y1 − 12
∴ = Height = 122 − 32 = 3 15
4 7

=
{
−2 4 × ( −8 ) + 7 × 12 + 13 } ∴R=
12 × 6
=
8 15
2
4 +7 2 3 15 5

x1 + 8 y1 − 12 Sol 12: (D) Given, ac = bd


= =–2
4 7
⇒ AO × OC = OB × OD
x1 = – 16, y1 = – 2
Y
Equation of required circle is
(x + 16)2 + (y + 2)2 = 52
D
x2 + y2 + 32x + 4y + 235 = 0

Sol 8: (B) Equation of circle is


(x – 0) (x – a) + (y – 1) (y – b) = 0  …(i) B X
O
Let the circle given by eq. (i) cut the x-axis at (h, 0) A C

h(h – a) + b = 0 This is true in case of circle and two secants


h – ah + b = 0
2
∴A, B, C and D lie on a circle.
The abscissa are roots of equation x – ax + b = 0
2
Sol 13: (C)

Sol 9: (C) x = 2y – 10 & x2 + y2 = 100


⇒ 4y2 – 40y + y2 = 0
⇒ 5y(y – 8) 0
∴ y = 8 (as point lies in 1st quadrant & x = + 6)
The line perpendicular to x – 2y + 10 =0 passing
through (6, 8) is (y – 8) = –2(x – 6)
2x + y = 20
It cuts the y-axis at (0, 20)
Since the centres lie on co-ordinate axes
Sol 10: (C) Let equation of circle be The centre are (1, 0), (–1, 0), (0, 1) and (0, –1)
x + y + 2gx + 2fy + c = 0
2 2
Consider two circles with centre (1, 0) & (0, 1)
Their point of intersection will lie on the line y = x
9 .76 | Circle

Putting y = x in (x – 1)2 + y2 = 1 Alternate:


⇒ 2x2 – 2x = 0 AM AB
Since, =2 ⇒ 1
=
AB BM
⇒ x = 1 & y = 1 (ignoring x = y = 0)
Let M be (h, k)
(1, 1) is the point
h k +3
By symmetry the other 3 points are Then, B ≡  , 
2 2 
(1, –1) (–1, 1) (–1, –1). Which lies on Circle.
It is a square of side 2 units Substitute to get the required Locus.
Area = 4 sq. units
Sol 16: (C) P = (0, 5)
Sol 14: (A) The y co-ordinate = 2, centre = (h, 2) & S1 = x2 + y2 + 2x – 4 = 0
radius = 2
x S2 = x2 + y2 – y + 1 = 0
On using the condition of tangency on y = ,
2 L1 = 25 − 4 = 21
2×2 −h
we get =±2 L2 = 21
5
⇒ h=4±2 5 ∴L1 = L2
But h > 0
Sol 17: (A) Let centre of circle be (h, k)
x coordinate is 4 + 2 5 .
∴h – 2k = 4 Y

Sol 15: (B) Let the midpoint of chord be (h, k) ⇒ h = 2k + 4 P(2,5)

∴Equation of chord is T = S1 ∴Centre is (2k + 4, k) C(h,k)


⇒ xh + 2( x + h) + yk – 3(y + k) + 9
= h2 + 4h + k2 – 6k + 9
X’ X
Since (0, 3) lies on this chord
2h + 3k – 3(3+k) = h2 + 4h + k2 – 6k
Locus of midpoint is
h2 + 2h + k2 – 6k + 9 = 0
Y’
1 3 Now CP ⊥ tangent
A (h, k) B M 5−k
∴ × 2 =−1
2 − 2k − 4
∴Let M be (x , y)
(2k + 2)
 3× 0 + x 9 + y  ∴5 – k =
(h, k) =  ,  2
 4 4 
∴5 – k = k + 1
Substituting in 1 we get locus of M.
∴k = 2
2 2
x 2(x)  y + 9  6 × (y + 9)
∴  + +  − +9=0 Center is (8, 2)
4
  4  4  4
(8 − 2) + (2 − 5)
2 2
⇒ x2 + y2 + 8x – 6y + 81 – 216 + 144 = 0 Radius = 3 5
=

⇒ x2 + 8x – (y – 3)2 = 0
Sol 18: (D) Let circle be S ≡ x2+y2+2gx+2fy + c = 0
S1 ≡ x2 + y2 = 1
S2 ≡ x2 + y2 + 2x – 3 = 0
M a them a ti c s | 9.77

S3 ≡ x2 + y2 + 2y – 3 = 0 [From (i) and (ii)]


⇒ S – S1 = 0 is the equation of chord of contact & it AB2 AD2
AC2 =
passes through centre of S1 AB2 − AD2
⇒ 2gx + 2fy + c + 1 = 0 AB.AD
AC =
Satisfying (0, 0) ⇒ c = – 1, AB2 − AD2
Similarly S – S2 = 0
Sol 21: (A) x3 + y3 + 3xy – 1 = 0
⇒ (2g –2)x + 2fy + 2 = 0
⇒ (x +y)3 – 3xy(x + y) + 3xy – 1 = 0
Satisfying (-1, 0), we get 2 – 2g + 2 = 0
⇒ (x +y)3 – 3xy(x + y – 1) – 13 = 0
⇒g=2
⇒ (x +y)3 – 13 = 3xy(x + y – 1)
Similarly, S - S3 = 0
⇒ (x +y –1){(x + y)2 + (x + y) + 1} – 3xy(x + y – 1) = 0
⇒ (2gx + (2f – 2)y + 2 = 0
We get,
(Satisfying (0, –1), we get ⇒ f = 2
∴ (x +y –1){(x + y)2 + (x + y) + 1 – 3xy} = 0
∴ Centre is (–2, –2)
(x +y –1) (x2 + y2 – xy + x + y + 1) = 0
Sol 19: (C) Let tangent from origin be y = mx For the curve x2 + y2 – xy + x + y + 1 = 0
Using the condition of tangency, we get 1
ab – h2 = 1 – >0
7m + 1 4
⇒ =5
m2 + 1 and
(7m + 1)2 = 25(m2 + 1) a h g 1 −1 / 2 1 / 2
⇒ 24m + 14m – 24 = 0
2 ∆ = h b f = −1 / 2 1 1/2
g f c 1/2 1/2 1
⇒ 12m2 + 7m – 12 = 0
⇒ 12m2 + 16m – 9m – 12 = 0  1 1  1 1 1  1 1
= 1 × 1 −  +  − −  +  − − 
 4 2  2 4 2  4 2
(4m – 3) (3m + 4)
3 4 3 3
∴m = and m = − = – =0
4 3 4 4
π
The angle between tangents = ∴ It is a point
2
Sol 20: (D) Since A, D, C lies on the circle with AC as
the diameter
AD ⊥ DC B
Previous Years’ Questions
Sol 1: (D) Equation of circle passing through a point
D (x1 , y1 ) and touching the straight line L, is given by
(x − x1 )2 + (y − y1 )2 + λL = 0

A C
∴ Equation of circle passing through (0, 2) and touching
x=0
∴ ∆ ADC  ∆ ABC ⇒ (x − 0)2 + (y − 2)2 + λx = 0  …..(i)
AC AD
⇒ =  …(i) Also, it passes through (-1, 0)
BC AB
⇒ 1 + 4 − λ= 0 λ ⇒ 5
Also, BC2 = AB2 + AC2  …(ii)
∴ Eq. (i) becomes,
AD2
AC2 = (AB2 + AC2) x2 + y 2 − 4y + 4 + 5x =0
AB2
⇒ x2 + y 2 + 5x − 4y + 4 =
0,
9 .78 | Circle

For x-intercept put y = 0 Sol 4: (A) Given, circle is inscribed in square formed by
2
⇒ x + 5x + 4 = 0 the lines
(x + 1)(x + 4) =
0 x2 − 8x=
+ 12 0 and y 2 − 14y =
+ 45 0
∴ x =−1, −4 ⇒ x 6 and
= = x =2, y 5 and= y 9

Sol 2: (B) For the point of intersection of the two given Which could be plotted as
curves y

C1 : y 2 = 4x
D (2,9) C (6,9)
y=9
and C 2 : x2 + y 2 − 6x + 1 =0
y

(1, 2)
y=5
A (2,5) B (6,5)

x’ x x’ x
O
(0, 0) (3, 0)
y’

(1, -2) Where ABCD clearly forms a square

y’ ∴ Centre of inscribed circle


= Point of intersection of diagonals
We have, x2 + 4x − 6x + 1 =0
= Mid point of AC or BD
⇒ x2 − 2x + 1 =0
 2 + 6  5 + 9 
⇒ (x − 1)2 =
0 = =  ,  (4,7)
 2   2 
⇒ x= 1 (equal real roots)
⇒ y =2, −2 ⇒ Centre of inscribed circle is (4, 7)

Thus, the given curves touch each other at exactly two Sol 5: (C) The line 5x-2y+6=0 meets
point (1, 2) and (1, -2).
The y-axis at the point (0, 3) and therefore the tangent
has to pass through the point (0, 3) and required length
Sol 3: (C) Here radius of smaller circle,
= x12 + y12 + 6x1 + 6y1 − 2
AC= 12 + 32 − 6= 2 Clearly, from the figure the radius
AC– = 02 + 32 + 6(0) + 6(3) − 2
of bigger circle
= 02 + 32 + 6(0) + 6(3) −=
2 25 5
=
2 2 2 2
r = 2 + [(2 − 1) + (1 − 3) ] = 25 5
=
2
r =9 Sol 6: (A) Since, the given circles intersect orthogonally.
⇒r= 3
2 ( g1 g2 + f1 f2 ) =
G + C2

∴ 2( −1)(0) + 2( −k)( −k) = 6 + k


A
3
⇒ 2k 2 − k − 6 =0 ⇒ k =− ,2
2 2

C C1(2,1)
(1,3) Sol 7: (C) Let O is the point at centre and P is the point
2 at circumference. Therefore, angle QOR is double the
angle QPR. So it is sufficient to find the angle QOR.
B
M a them a ti c s | 9.79

y Sol 11: Let the given circles C1 and C2 have centres O1


and O2 and radii r1 and r2 respectively.
Q (3,4)
Let the variable circle C touching C1 internally, C2

4,3)
externally have a radius r and centre at O

x’ R (- O (0,0)
x
C2 C1
O
r 2
2

P r
y’ O1 O
c
Now, slope of OQ, m1 = 4/3, slope of OR, m2 = ‒3/4 r1
Here, m1m2 = −1
Threfore, ∠QOR =
π/2
Which implies that ∠QPR =
π/4
Now, OO2 =
r + r2 and OO1 =
r1 − r
⇒ OO1 + OO2 =+
r1 r2
Sol 8: (B) Given, x2 + y 2 =
4
Centre ≡ C1 ≡ (0,0) and R1 =
2 Which is greater than

Again, x2 + y 2 − 6x − 8y
= − 24 0, then C2 ≡ (3, 4) O1O2 as O1O2 < r1 + r2 (C2 lies inside C1 )

and R 2 = 7 again, C1C2= 5= R 2 − R1 ⇒ Locus of O is an ellipse with foci O1 and O2


Since, the given circles touch internally therefore, they Alternate Solution
can have just one common tangent at the point of
contact. Let equations of C1 be x2 + y2 = r12 and of C2 be
(x − a)2 + (y − b)2 =
r22
Sol 9: Since, the tangents are perpendicular. Let centre C be (h, k) and radius r, then by the given
So, locus of perpendicular tangents to circle condition

x2 + y 2 =
169 is a director circle having equation (h − a)2 + (k − b)2 =r + r2 and h2 + k 2 =r1 − r
x2 + y 2 =
338
⇒ (h − a)2 + (k − b)2 + h2 + k 2 = r1 + r2

Sol 10: The equation of circle having tangent Required locus is


2x+3y+1=0 at (1, -1)
(x − a)2 + (y − b)2 + x2 + y 2 = r1 + r2
⇒ (x − 1)2 + (y + 1)2 + λ(2x + 3y + 1) = 0
Which represents an ellipse whose foci are at (a, b) and
x2 + y 2 + 2x(λ − 1) + y(3λ + 2) + (λ + 2) =0  ….(i) (0, 0).
Which is orthogonal to the circle having end point of
diameter (0, -1) and (-2, 3) Sol 12: Equation of any tangent to circle x2 + y 2 =
r 2 is
⇒ x(x + 2) + (y + 1)(y − 3) =
0 x cos θ + y sin θ = r  ….(i)
or 2 2
x + y + 2x − 2y − 3 =0  …(ii) Suppose Eq. (i) is tangent to 4x + 25y = 2
100 2

2(2λ − 2) 2(3λ + 2) x2 y 2
∴ ⋅1 + ( −1) = λ + 2 − 3 Or + =1at(x1 , y1 )
2 2 25 4
⇒ 2λ − 2 − 3λ − 2 = λ − 1 xx yy
Then, Eq. (i) and 1 + 1 = 1 are identical
⇒ 2λ = −3 ⇒ λ = −3 / 2 25 4
y1
∴ From Eq. (i) equation of circle, x1 / 25 4 = 1
∴ =
2 2 cos θ sin θ r
2x + 2y − 10x − 5y + 1 =0 25cos θ 4 sin θ
⇒ x1
= ,=
y1
r r
9 .80 | Circle

The line (i) meet the coordinates axes in A(r sec θ,0)   π π π π 
and β(0,r cosecθ) . Let (h, k) be mid point of AB.   θ− +θ+ θ− −θ−  
1 3 3 ⋅ cos 3 3  + 2r sin θ 
= r  2sin
3  2 2  
r sec θ r cosec θ    
Then,
= h = and k   
2 2
1
r r = [r{2cos θ cos π / 3} + 2r cos θ]
Therefore,
= 2h = and 2k 3
cos θ sin θ
1
25 4 = [r ⋅ cos θ + 2r cos θ=] r cos θ
∴ x=
1 and y=1 3
2h 2k 1  π  π
x2 y 2 and q
= [r sin  θ −  + r sin  θ +  + 2r sin θ]
As (x1 , y1 ) lies on the ellipse + 1
= 3  3  3
25 4
1  π  π
1  625  1  4  = [r{sin  θ −  + sin  θ + } + 2r sin θ]
We get  +   = 1 3  3   3 
25  4h2  4  k 2 
1
= [r(2sin θ cos π / 3) + 2r sin θ]
25 1 3
⇒ + =1
4h2 k 2 =
1
[r(sin θ) + 2r sin θ]
or 2
25k + 4h = 2 2 2
4h k 3
= r sin θ
Therefore, required locus is 4x2 + 25y 2 = 4x2 y 2
Now, (p,q) = (r cos θ,r sin θ) lies on x2 + y 2
Sol 13: Let the coordinate of point P be (2r cos θ, 2r sin θ) = r 2 , which is C1
We have, OA = r, OP = 2r
Sol 14: (A) Eq. of circle touching x − a × y at (1,0 ) u
Since, ∆OAP is a right angled triangle. 2 2
( x − 1) + ( y − k ) k2
=

C2 Circle passes through ( 2,3) , then


A
C1 2 2
- ( x − 1) + (3 − k ) k2
=

P  O 1 + 9 − 6k + k 2 =k2

B ⇒ 6k =
10
10
⇒ 2k =
3
cos φ =1 / 2
Sol 15: (B) The eq. of circle touching the
⇒ φ = π/3
∴ Coordinates of A and B are
( )
a − a × u at 3,0 is
2 2
{r cos(θ − π / 3),r sin(θ − π / 3)] and (1 − 3) + ( −2, −k ) =k 2

⇒ 4 + 4 + 4k + k 2 =
k2
  π    π
r cos  θ +   , r sin  θ +  ⇒ 4k =
−8
  3    3
⇒ k =−2
If p,q is the centroid of ∆PAB , then 2 2
Circle: ( x − 3) ( −2 − k ) =k 2
1
=p [r cos(θ − π / 3) + r cos(θ + π / 3) + 2r cos θ] Point (5, −2 )
3
2 2
=
1
[r{cos(θ − π / 3) + cos(θ + π / 3)} + 2r cos θ] (5, −3) + ( −2 + 2 ) = 2+2 = 4
3
Only (5, −2 ) lies on circle.
  π π π π 
  θ− +θ+ θ− −θ−  
1 3 3 ⋅ cos 3 3  + 2r cos θ 
r  2cos
3  2 2  
  
 
M a them a ti c s | 9.81

x2 y 2
Sol 16: (D) + 1
=
10 9

to is = (± 7 ,0 )
Circle having cente as ( 0,3)
2
x2 + ( y − 3) =γ 2 passes through focus , then
2 2
Sol 19: (C) x + y − 8x − 8y − 4 =
0
(± 7 )
2 2
+ ( 0 − 3) =γ2
2 2
2
(x − 4) + (y − 4) 36
=
7 + 9 =γ
Circles touch each other exotically
⇒ γ 2 =16
2 2
⇒ x2 + ( y − 3) =
16
2 k + 6= (n − 4 ) + (k − 4 )
⇒ x2 + y 2 − 6y − 7 =0 k 2 + 36 + 12k = h2 + 16 − 3h + k 2 − 3k + 16
⇒ h2 − 3h − 9 =20k
Sol 17: (D) If circles C and T touch each other externally
⇒ x2 − 3x − 20y − 4 =
0
then
2 2 If y < 0
1 + y= (1 − 0 ) + (1 − y )
2
⇒ (1 + y ) =1 + 1 − y 2 ( )
⇒ 12 + y 2 + 2y =1 + 1 + y 2 − 2y (4,4) (h,k)

1 K
⇒y=
4
(h,-k)

2 2 2
(1,1) ( −k + 6 ) = (n − 4 ) + (k − 4 )
⇒ h2 − 8h + 4k − 4 =
0
T ⇒ x2 − 8x + 4y − 4 =
0
Locus is Parabola

Sol 18: (B) ( x − 2 ) + ( y − 3 ) =


25
2 2
JEE Advanced/Boards

Exercise 1
2 2
( x + 3) + ( y + 9 ) 64
=

2 2
C1C2 = ( 2 + 3) + (3 + 9 ) Sol 1: The equation of line through origin is y = mx
Let point on circle be (h1, mh1) and (h2, mh2)
= 25 + 144
S = x2 + y2 – 8x – 6y + 24 = 0
= 169 13
=
O = origin
r1 + r2 = 3 + 5 = 13
(i) The equation of chord of S whose mid-point is
⇒ r1 + r2 =
c1 c2 (h, k) is
hx+ ky – 4(x + h) – 3(y + k) + 24
Circles touch each other externally therefrom, three
tangents are possible = h2 + k2 – 8h – 6k + 24
9 .82 | Circle

Since it passes through origin Sol 3: Equation of common chord


∴–4h – 3k = h2 + k2 – 8h – 6k = x2 + y2 + 4x + 22y + a–(x2 + y2 – 2x + 8y - b)
∴ Locus of point is = 6x + 14 y + (a + b) = 0
x2 + y2 – 4x – 3y = 0 Now centre of second circle lies on this
(ii) OP = OA × OB ∴ 6 × 1 + 14 × (– 4) + (a + b) = 0
It is a known property that ∴ (a + b) = 50
T B Now a , b > 0
∴ AM > GM
A a+b
⇒ > ab
2
O ∴ 25 > ab
ab < 625
OA × OB = OT2=OP2
∴OP = OT = constant k Sol 4: x2 + y2 = 1

OT = S(0,0) = 24 y−4
Z=
x−7
∴The locus of P is the circle of radius 24 and centre 4
= origin In this the slope from the point (7, 4); tanq2 =
7
⇒ x2 + y2 = 24 is the locus of P
(7, 4)
2OA.OB OA. OB m
(iii) OP = =
OA + OB OM 

∴OP × OM = OA × OB
∴ A and M are harmonic conjugates of P & B
2 1

A P M B
AM AB AM
∴ = ⇒ =2
PM MB PM
∴ P is mid-point of A & M
m − tan θ2
tanθ = tan (θ1 – θ2) =
∴Locus of P: 1 + mtan θ2
x + y – 8x – 6y + 24 – (x + y – 4x – 3y) = 0
2 2 2 2
r m− 4 /7
∴4x + 3y = 24 is locus of P =
 tangent 4
1 + m×
7
Sol 2: Radius of given circle= 4 + 2 − C = 6 − C
1 m− 4 /7 7m − 4
⇒ ± = =
r = 2 r1 and r1 = 2 r2, r2 = 2 r3 8 4m 7 + 4m
1+
Sum of radii of all circles 7
3 5
r r r r ∴M = and m =
=r+ + +...............= ⇒ =2 4 12
2 2 1 1
1− 1− 2×3 6 ×5
2 2 ∴2M + 6m = + =4
∴r = 2 – 2 4 12

⇒ 6−C = 2 – 2 ⇒6–C=4+2–4 2

∴C = 4 2 = 32 ⇒ n = 32
M a them a ti c s | 9.83

Sol 5: The radical axis of 2 circles is – 6 + λ(–4) = 0


 3 −3
 2g −  x + (2f – 4)y = 0 λ=
 2 2
Centre of the given circle = (–1, 1) Equation of circle is

and radius = 1 x2 + y2 – 8 – 3x – 3y + 12 = 0

Since it is a tangent to the circle ⇒ x2 + y2 – 3x – 3y + 4 = 0


 3
(2f − 4) −  2g − 
2 Sol 8: C: x2 + y 2 + y − 1 + k ( x + y − 1 ) =0
⇒  =1
2 It is the family of circle passing through points of
 3
 2g −  + (2f − 4)
2
intersection of a circle & L.
 2 
2
Putting x = 1 – y in C1
 3  3
⇒ (2f – 4) +  2g −  + 2(2f – 4)
2
 2g −  We get y2 – 2y + 1+fy2 + y – 1 = 0
 2  2
1 1
2 ⇒ 2y2 – y = 0 ⇒ y = 0, & x = 1 or
 3 2 2
=  2g −  + (2t – 4)2
 2 1 1
∴ The point of intersection are A(1, 0) and B  , 
 3 2 2
∴ (2f – 4)  2g −  = 0
2 The minimum value of radius is when point act as

diameter
3
∴ Either f = 2 or g = 2 2
1
4 1 1 1 1 1
∴rmin =   +  = × 2 =
2 2 2 2 2 2 2
Sol 6: The line passing through points A(3, 7) and
B(6, 5) is 2x + 3y – 27 = 0 Sol 9: The equation of circle co-axial with 2 circle is S1
+ lS2=0
The family of circles passing through these points is
2x2 + 2y2 – 2x + 6y – 3 +
(x – 3) (x – 6) + (y – 7) (y – 5) + λ (2x + 3y – 27) = 0
λ(x2 + y2 + 4x + 2y + 1) = 0
⇒ x2 – 9x + 18 + y2 – 12y + 35 + λ (2x + 3y – 27) = 0
= (2 + λ)x2 + (λ + 2)y2 + (4λ – 2)x
∴Chord of contact = s1 – s2
+ (2λ +6)y + λ – 3 = 0
⇒ –5x – 6y + 50 + λ(2x + 3y – 27) = 0
2 − 4λ 1 − 2λ
⇒ L1 + λL2 = 0 xcentre = =
2(2 + λ ) λ+2
The point which passes through intersection of L1 and −(λ + 3)
L2 is the point of intersection of all λ ycentre =
λ+2
5x + 6y = 50 5
Radical axis of the two circle is s1 – s2 ≡ 5x – y + =0
2
2x + 3y = 27 Centre lies on radical axis

∴x = 2 & y =
23
∴ P=
 23 
 2,  ∴5 ×
(1 − 2λ ) + λ+3 5
+ =0
3  3  λ+2 λ+2 2
⇒ 10 – 20λ + 2λ + 6 + 5λ + 10 = 0
Sol 7: The locus of point of intersection of mutually
perpendicular tangent is the director circle ⇒ 13λ = 26∴λ = 2

∴Locus of point = x2 + y2 = 8 ∴Equation of circle is 4x2 + 4y2 + 6x + 10y – 1 = 0

The equation of family of circle touch a given circles &


at (x1, y1) is S + λ(L) where L = tangent Sol 10: s1 ≡ x2 + y2 – 4x – 6y – 12 = 0

x2 + y2 – 8 + λ (x × 2 + y × 2 – 8) =0 s2 ≡ x2 + y2 + 6x + 4y – 12 = 0

Now this passes through (1, 1) s3 ≡ x2 + y2 – 2x – 4 = 0


9 .84 | Circle

The circle passing through point of intersection of s1 (x + 2) (y – 3) = 0


and s2 is s ≡ s1 + ls2 = 0
The centre is point of intersection of these two lines
⇒ x + y – 4x – 6y – 12 + λ
2 2
c ≡ (–2, 3)
(x2 + y2 + 6x + 4y – 12) = 0
g = 2 and f = – 3
⇒(λ + 1)x2 + (λ + 1)y2 + (6λ – 4)x
s ≡ x2 + y2 + 4x – 6y = 0
+ (4λ – 6)y – 12 (λ +1) = 0
s1 = x2 + y2 – kx + 2ky – 8 = 0
Since it is orthogonal to s3
Since s & s1 are orthogonal
∴2gg1 + 2ff1 = c+c1
∴2gg1 + 2ff1 = 0 - 8
(6λ − 4) −12(λ + 1) ⇒ 2( – k) + (–3) × 2k = 0 – 8
⇒ x – 1 + 0= –4
λ +1 (λ + 1)
∴k = 1
∴4 – 6λ = – 16 (λ + 1)
Sol 13: Since the circle cuts co-ordinate axis orthogonally
10λ = – 20
∴C ≡ (0, 0)
λ=–2
∴s ≡ x2 + y2 – a2 = 0
s ≡ – x2 – y2 – 16x – 4y + 12 = 0
s ≡ x2 + y2 – 14x – 8y + 64 = 0
∴x2 + y2 + 16x + 14y – 12 = 0
Since s & s1 are orthogonal
Sol 11: Let s = x + y + 2gx + 2fy + c = 0
2 2
∴ 2 × 0 × –7 + 2 × 0 × –4 = – a2 + 64
Now (–g, –f) lies on 2x – 2y + 9 = 0 ∴ a2 = 64
⇒–2g + 2f + 9 = 0 ∴ s ≡ x2 + y2 – 64 = 0
and it is orthogonal to x2 + y2 – 4 = 0
Sol 14: Let the given circles
2g × 0 + 2f × 0 = c – 4
S1 ≡ x2 + y2 + 2g1x + 2f1y + c1 = 0
C=4
S2 ≡ x2 + y2 + 2g2x + 2f2y + c2 = 0
9
and f = g –
2 Let the circle orthogonal to the two circles be
 9
s = x2 + y2 + 2gx + 2  g −  y + 4 = 0 x2 + y2 + 2gx + 2fy + c = 0
 2
∴2gg1 + 2ff1 = c1 + c
s ≡ x + y – 9y + 4 + 2g (x + y)
2 2

and 2gg2 + 2ff2 = c + c2


∴ It passes through point of intersection of S and L
⇒ 2g(g1 – g2) + 2f(f1 – f2) = c1 – c2
Putting x = – y in s
Now the centre is (–g, –f)
2y2 – 9y + 4 = 0 ⇒ 2y2 – 8y – y + 4 = 0
1 ∴x =–g & y = –f substituting instead of g & f
1
∴y = or y = 4 & x = – , – 4
2 2 We get 2x(g1 – g2) + 2y(f1 + f2) = (c1 – c2)
 −1 1  Which is the radical axis & (straight line)
∴The points are  ,  & (–4, 4)
 2 2 The locus of centres of given s1, s2 is s1 – s2=0
Sol 12: Let the equation of circle be 4x + 5x – 6y – 4y + 7 = 0
x + y + 2gx + 2fy = 0
2 2
9x – 10y + 7 = 0
(it passes through origin)
The line pair is Sol 15: Consider a point circle at (–2, 7)

xy – 3x + 2y – 6 = 0 (x + 2)2 + (y –7)2 = 0

x(y – 3) + 2(y – 3) = 0 Now the equation a circle touching a circle at point is


s + λL
M a them a ti c s | 9.85

Where L is tangent to L 2nd smallest circle be r1 & 2nd largest circle be r2


∴s ≡ (x + 2)2 + (y – 7)2 + λ(x + y –5) = 0
⇒ x2 + y2 + (λ + 4)x + (λ − 14)y + (53 – 5λ) = 0
r1 r r2
∴s2 ≡ x2 + y2 + 4x – 6y + 9 = 0 8 15

Since s & s2 are orthogonal


∴(λ + 4) + (λ – 14) x – 3 = 53 – 5λ + 9
∴ r12 = 8r & r22 = 18 r
⇒4λ = 12 ⇒ λ = 3
r2 = r1 r2
∴ Equation of circle
∴r4 = 8r × 18 r
s ≡ x2 + y2 + 7x – 11y + 38 = 0
r= 8 × 18 = 12

Sol 16: Let the circle be x2 + y2 + 2gx + 2fy + c = 0


Sol 18: The pair of lines is
(–6, 0) lies on the circle
7x2 – 18 xy + 7y2 = 0
∴ 36 – 12g + c = 0  … (i)
Since co-eff of x = coeff of y.
The power of (i, i) is 5
angle bisectors are
⇒ 1 + 1 + 2g + 2f + c = 5
(x – y) = 0 & x + y = 0
⇒ 2g + 2 f + c = 3  … (ii)
Since the given circle lies in the 1st quadrant
S is orthogonal to
∴Our circle should also lie in the 1st quadrant
x + y – 4x – 6y – 3 = 0
2 2
∴ Its centre should lie on y = x
⇒ 2g(– 2) + 2f(– 3) = c – 3
Centre ≡ (h , h)
∴ 4g + 6f + c – 3 = 0  … (iii)
Now (x – h)2 + (y – h)2 = k2
From (ii) and (iii)
Let y = mx be equation of tangent
2g + 2c = 6 ⇒ g + c = 3  … (iv)
h − mh
=R
From i and iv g = 3
1 + m2
−3
∴c = 0 & f = ∴R2(m2 + 1) = h2(m2 – 2m + 1)
2
s ≡ x2 + y2 + 6x – 3y = 0 ∴(R2 – h2)m2 + 2h2m + R2 – h2 = 0
Comparing to pair of lines
Sol 17: Radius of largest circle = 18 2h2 −18
We get =
Radius of smallest circle = 8 R −h2 2 7

14h2 = – 18R2 + 18h2


∴4h2 = 18R2
r1 r2 r3
3
∴h = R h is in 1st quadrant
2
When 3 circle touching each other have direct common Since C touches C1
tangent
= (R1 – R) = distance between centres
The radius of the middle circle is GM of radius of other 2 2
2 circles  3R   3R 
∴(4 2 – R)2 =  4 −  +4 − 
∴ r22 = r1r3  2  2

In the given problem  3R 


⇒4 2 – R = ± 2 4 − 
Let radius of middle circle be r,  2
9 .86 | Circle

 3R  Equation of common chord is


∴4 2 – R = – 2 4 − 
 2 2(b – a)x + 2(b – a)y = b2 – a2
3 2R ∴2x + 2y = a + b
8 2 = +R
2 It passes through (a, a)
8 2 =4R 4a = a + b
b = 3a
R=2 2
b
⇒ =3
3× 2 2 a
h= =6
2 (D) C2 passes through centre of C1
Equation is (x – 6)2+(y – 6)2 = (2 2 )2 ∴a2 + a2 – 2ab – 2ab + b2 = 0
⇒ b2 – 4ab + 2a2 = 0
Sol 19: (r, p, q) (A) Centre of C1 ≡ (a, a) & radius = a for
2
C2 centre ≡ (b, b) & radius = b b b
⇒   − 4  + 2 =0
C1 & C2 cannot touch other internally a a

b 4 ± 16 − 8 4±2 2
∴ = = =2± 2
a 2 2
C1 But b > a
C2
∴b = 2 + 2
a

Sol 20: y = x + 10 & y = x – 6 are tangents


(b − a)2 + (b − a)2 = (b + a)
The centre of circle passes through
∴ 2 (b – a) = (b + a)
(10 − 6)
y=x+ =y=x+2
∴( 2 – 1)b = ( 2 + 1)a 2
1
Also radius, = ⊥ distance between lines
b 2 +1 2
∴ = =3+2 2
a 2 −1 1 c − c2 1 16
= × 1 = × =4 2
2 2
a +b 2 2 2
(B) Equation of
C1 ≡ x2 + y2 – 2ax – 2ay + a2 = 0 ∴Circle is (x – h)2 + (y – (h + 2))2 = (4 2 )2

C2 ≡ x2 + y2 – 2bx – 2by + b2 = 0 h + k = 2h + 2 = a + b a

C1 & C2 are orthogonal Since y – axis is tangent

2(–ax – b) + 2(–ax – b) = a2 + b2 ∴h = Radius

4ab = a2 + b2 ∴h = 4 2

a2 – 4ab + b2=0 and h + k = 2h + 2 = 8 2 + 2


∴a + b = 10
b 4 ± 16 − 4 4 ± 12
= =
a 2 2
Sol 21:
C2 C3
=2± 3 C1
b
But b > a ∴ =2+ 3 C
a r1 r2
(C) C1 and C2 intersect such that common chord is longest
∴ C2 bisects C1
M a them a ti c s | 9.87

Since, centres of the Circle are collinear.  (2 + 3λ ) 


2
∴(λ – 1) + 
2
 – λ – 2 = 13
2r1 + 2r2  2 
∴Radius of bigger circle = = 14
2 ∴l2 = 4 ∴ λ = ± 2
Now distance of point of intersection from centre = R –
∴ Equation of circles are
(2r1) = 14 – 2 × 4 = 6 = d
x2 + y2 + 2x – 8y + 4 = 0 or x2 + y2 – 6x + 4y = 0
Length of chord

= 2 R 2 − d2 = 2(14)2 – (6)2= 4 40 = 8 10 Sol 24: Equation of circle touching other. circle is at


point is s + λ (L) = 0
m + n + p = 1 + 8 + 10 = 19
Where L is equation of tangent at the point
Sol 22: Equation of a circle passing through two points x2 + y2 + 4x – 6y – 3 + λ (2x + 3y
(x – 1) (x – 4) + (y – 7) (y – 8) + λ(L) =0 + 2(x + 2) – 3 (y + 3) – 3) = 0
L passing through (4, 7) & (1, 8) It passes through (1, 1)
−1
is y – 8 = (x – 1) −(1 + 1 + 4 − 6 − 3) 3 −3
3 ∴λ = = =
(2 + 3 + 6 − 12 − 3) −4 4
3y + x – 25 = 0
∴ Equation of circle is
∴ (x – 1) (x – 4) + (y – 7) (y – 8) + λ(3y + x – 25) =0
4x2 + 4y2 + 16x – 24y – 12 – 3 (4x – 8) = 0
(5, 6) satisfies this equation
4x2 + 4y2 + 4x – 24y + 12 = 0
(4 + 2)
λ=+ =3
+2 x2 + y2 + x – 6y + 3 = 0
Equation of circle is x2 + y2 – 2x – 6y – 15 = 0
Let the points of intersection of tangent be (h , k) Exercise 2
chord of contact is
Single Correct Choice Type
hx + ky – (x + h) – 3(y + k) – 15 = h2 + k2 – 2h – 6h – 15
(h – 1)x + (k – 3)y + h + 3k – h2 – k2 = 0 Sol 1: (A) Since BAC = 90º

(h − 1) 5 locus of A is the circle with (3, 0), (–3, 0) as diameter


Now, =  … (i)
h + 3k − h2 − k 2 17 Let A = (h, – k)
k −3 1 (h –3) ( h + 3) + k2 = 0
and =
h + 3k − h2 − k 2 17 Now, centroid
h−1 h+3−3 k +0 +0
= 5 ⇒ h –1 = 5( k –3) C(x, y) =  , 
k −3  3 3 
h = 5(K – 3) + 1 Substituting h, k in terms of (x, y)
Substituting in 1 we get k = 2 (3x – 3) (3x + 3) + (3y)2 = 0
∴h = – 4 x2 + y2 = 1 is the equation of centroid
∴Point is (–4, 2)
Sol 2: (C) | y | = x + 1 & (x –1)2 + y2 = 4
Sol 23: The equation of circle which touches a given Substituting value of | y |
line at a point is
(x –1)2 + (x + 1)2 = 4
(x–1)2 + (y – 1)2 + λ(2x – 3y +1) = 0
x2 = 1
∴x2 – y2 + 2(λ – 1)x – (2 + 3λ)y + λ + 2 = 0
x=±1
R= 13 For x = –1; y = 0
9 .88 | Circle

For x = +1; | y | = 2 ∴ y ± 2 3 − 2m
Now y = mx is tangent to the circle ∴ = 13
∴ Three possible solutions are possible 1 + m2
4m2 – 12m + 9 = 13m2 + 13 ⇒ 9m2 + 12m + 4 = 0
Alternate method
9m2 + 6m + 6m + 4 = 0
Plotting the graph of |y| = x + 1 and (x – 1)2 + y2 = 4
(3m + 2)2 = 0
2
m=–
3
∴Equation of line is 3y + 2x = 0
-1 1

Sol 6: (A) The circumcenter of triangle A,B,C is (0, 0)


Let c ≡ (h , k)
We can directly see that three possible intersection are 1 +h 1 +k 
And centroid (c1) is  , 
possible  3 3 
Let the orthocentre be (x, y)
Sol 3: (B) Line 1 passes through (3, 1) and The centroid divides O and C in ratio 2 : 1
Line 2 passes through (1, 3)
1 +h 1 +k  x y
Lines L1 and L2 are ⊥ ∴ locus of point of intersection is
∴ ,  =  , 
 3 3  3 3
a circle with (3, 1) & (1,3) as ends of diameter
∴h = (x – 1) and k = (y – 1)
Locus of points is (x – 3) (x – 1) + (y – 1) (y – 3) = 0
(x –1)2 + (y – 1)2 = 1
∴x2 + y2 – 4x – 4y + 6 = 0
∴x2 + y2 – 2x – 2y + 1 = 0

Sol 4: (B) Plotting the point on a graph


Sol 7: (D) Centre of circle is (– 8, –6)
y2 A B Equation of line is y = 2x + 5
∴ Q is the foot of perpendicular of (– 8, –6) on 2x – y
+5=0
y1 D C
x − ( −8) y − ( −6) −( −5)
∴ = =
x1 x2 2 −1 5

It is not necessary that ∴x = – 6 & y = – 7

|x2 – x1| = |y2 – y1| ∴Q ≡ (–6, –7)

With (x2, y1) & (x1, y2) as ends of diameter ∠ABC = 90º
Sol 8: (A) Centre of C1 = (2, 0) R1 = 4 & R2 = 4
and ∠ADC = 90º
Centre of C2 = (– 2, 0)
∴ ABCD are concyclic

Sol 5: (D) Let A = (0, 6), B = (5, 5) & C = (–1, 1)


−1
Slope of AB = & mAC = 5 (-2, 0) (2, 0)
5
∴ AB ⊥ AC
Circumcentre is midpoint of BC
∴The other 2 points of rhombus lie on y axis put in
O = (2, 3) x = 0 we get
1
And radius = 62 + 4 2 = 13 Y=±2 3
2
M a them a ti c s | 9.89

∴ Length of 1st diagonal is (2 – (–2) = 4 and length of


Length intercepted = 2 R 2 − ( ⊥ dis tance)2
2nd diagonal = 4 3
2
1 1 O−O+C 2 C2
Area of rhombus = ab = ×16 3 = 8 3 sq. units ∴ l1 = 2 22 −   =2 2 −
2 2  2  2

(5 − 7 + C)2
Sol 9: (A) From (3, 4) chords are drawn to l2 = 2 (3)2 −
2
x2 + y2 – 4x = 0 l1 = l2
Let mid points of chord be (b, h) C2 (C − 2)2
∴4 – =9–
∴h2 + k2 – 4h = xh + yk –2(h + x) 2 2
Now (3, 4) pass through these chords ∴C2 – 4C + 4 – C2 = 10

∴h2 + k2 – 4h = 3h + 4k –2(h + 3) 3
C=–
2
∴Locus of mid-point is x2 + y2 – 5x – 4y + 6 = 0
3
Line is y = x –
2
Sol 10: (B) Let p = (x, y)
2x – 2y – 3 = 0
 20 cos θ + 15 20 sin θ + 15 
(x, y) =  , 
 5 5  Sol 14: (D) Equation of circle is x2 + y2 = r2
x −1 y −1 Let P ≡ (a, b)
cos θ = & sinθ =
4 4
Let the midpoint of a point (h, k) on circle &
(x – 1)2 + (y –1)2 = 16 P(a, b) be M(x, y)
This is a circle. a+h b+k 
(x, y) =  , 
 2 2 
Sol 11: (B) (3, 4) & (–1, –2) are ends of diameter h = 2x – a ; k = 2y – b
(x – 3)(x + 1) + (y – 4)(y + 2) = 0
(2x –a)2 + (2y –b)2 = r2 is locus of M
x + y – 2x – 2y – 11 = 0
2 2
2 2 2
 a  b r
x −  +y −  =  
Sol 12: (A) Shortest distance from line to circle  2  2 2
= ⊥ distance – radius
Multiple Correct Choice Type
Centre of circle ≡ (3, –4) & radius = 5
Sol 15: (C, D) Let h, k be the point of intersection
9 − 16 − 25 32
∴ ⊥ distance = =
25 5  k   k 
∴ Slope of lines is   and  
h−1 h+1
32 7
∴shortest distance = –5= For point (1, 0) and (–1, 0)
5 5
tan θ − tan θ1
And tan (θ – θ1) =
Sol 13: (A) Slope of the line is 1 1 + tan θ tan θ1
∴ The angle between lines is either 45º or 135º
∴y = x + c
θ – θ1 = 45º or 135º
The two circle are
s1 ≡ x2 + y2 = 4 k k

c1 = (0, 0) & R = 2 ± 1 = h−1 h+1
k2
1+ 2
s2 ≡ x2 + y2 – 10x – 14y + 65 = 0 h −1
c2 = (5, 7) & R = 3 2k
±1=
h + k2 − 1
2
9 .90 | Circle

∴h2 + k2 – 2k – 1 = 0 C ≡ (0, 1) R = 2 S : (x + 1)(x –5) + (y –1) (y – 5) = 0

∴h2 + k2 + 2k – 1 = 0 C ≡ (0, –1) R = 2 ∴x2 + y2 – 4x – 6y = 0


Center c = (2, 3) ; r = 13
Sol 16: (B, C, D) s1 ≡ x + y + 2x + 4y + 1 = 0
2 2 3
The line joining origin to center is y = x
2
s2 ≡ x2 + y2 – 4x + 3 = 0 ∴3x – 2y = 0
s3 ≡ x + y + 6y + 5 = 0
2 2
The points will lie on the line ⊥ to 3x – 2y = 0 & passing
Radical axes of s1 and s2 is through (2, 3) at a distance of r from (2, 3)

6x + 4y – 2 = 0 −2 −2
L:y–3= (x –2) tanθ =
3 3
3x + 2y - 1 = 0
2x + 3y – 13 = 0
Radical axes of s3 and s2 is
Let points be (h, k)
6y + 4x + 2 = 0
When θ is in 2nd quadrant
3y + 2x + 1 = 0
sinθ > 0 & cosθ < 0
5x + 3y = 0
h = a + r cosθ ; k = a + r sinq
x = 1 y = –1
−3
(1, -1) is the radical centre ∴h = 2 + 13 ×
13
It is a known property that circle which is orthogonal to 2
3 circle has its center equal to radical center & radius k=3+ 13 ×
13
= length of tangent from radical center to any circles.
∴P1 = (–1, 5)
Radices = 1+1+ 2− 4 +1 = 1
When q lies in 4th quadrant
Equation of orthogonal circle is (x –1) + (y +1) = 1
2 2
sinθ < 0 & cosθ > 0
This circle touches both x & y axis.
3
h=2+ × 13
Its x & y-intercept are 1 13
 −2 
k=3+  × 13 
Sol 17: (B, C) c1 ≡ x2 + y2 – 4x + 6y + 8 = 0
 13 
c2 ≡ x2 + y2 – 10x – 6y + 14 = 0 ∴P2 = (5, 1)
Centre of c1 ≡ (2, –3)
Sol 19: (A, C, D) s1 : x2 + y2 + 2x + 4y – 20 = 0
Centre of c2 ≡ (5, 3)
s2 ≡ x2 + y2 + 6x – 8y + 10 = 0
r1 = 4+9−8 = 5
c1 = (–1, –2) & c2 = (–3, 4)
r2 = 25 + 9 − 14 = 2 5
r1 = 12 + 22 + 20 = 5
c1c2 = r1 + r2
r2 = 32 + 42 − 10 = 15
c1c2 = (5 − 3)2 + (6)2 = 3 5
c1c2 = 22 + 62 = 40 = 2 10
∴ c1 & c2 touch each other
c1c2 = r1+r2
∴ Radical axis is the common tangent and the mid-
point of c1c2 doesn’t lie on radical axis as their radius and c1c2 > | r1 – r2 |
are not the same. ∴ The two circles intersect each other at 2 points
2gg1 + 2ff1 = 2 × 3 + 4x – 4 = – 10 = c + c1
Sol 18: (B, D) A = (–1, 1); B = (0, 6); C = (5, 5)
AB ⊥ BC
∴The circle passing through ABC will have AC as a
diameter
M a them a ti c s | 9.91

The 2 circle are orthogonal 2 circle are possible


When all 3 lines are parallel no circles are possible
r1 r2
C1 C2
Sol 21: (A, B, D) c1 = (x + 7)2 + (y – 2)2 = 25
∴r1 = 5
c2 is director circle of c1
∴ r2 = 5 2

Length of common tangents And c3 director circle of c2


∴ r3 = 5 2 × 2 = 10
= (c1c2 )2 − (r2 − r1 )2 = 40 − (5 − 15)2
Area enclosed by c3 = pr2 = 100 p
= 10 15 = 5(12/5)4 Area enclosed of c2 = π ×( 2 r)2 = 2pr2
The equation of common chord is s1 – s2 = 2 times area enclosed by c1
⇒ 4x – 12y + 30 = 0
⇒ 2x – 6y + 15 = 0 Sol 22: (B, C) S1 ≡ x2 + y2 – 2x – 4y + 1 = 0 r1 = 2
−2 + 12 + 15 25 G ≡ (1, 2 ) , r1 =
2
Perpendicular from c1 on this ⇒ =
40 40
S2 ≡ x2 + y2 + 4x + 4y – 1 = 0
Length of common chord = 2 r 2 − a2 C2 ≡ ( −2, − 2 ) , r2 =3
2
 25  625 C 1C 2 = 32 + 42 = 5
=2 25 −   =2 25 −
 40  40 The two circle touch each other externally and common
tangent is S2 – S1 = 0
75 10 3 3
=2 = =5 6x + 8y – 2 = 0
8 2 2 2
3x + 4y – 1 = 0
Sol 20: (A, C, D) Consider 2 lines not parallel to one
another and when the third line passes through Sol 23: (A, C, D) S1 ≡ x2 + y2 – 6x – 6y + 9 = 0
intersection of both lines, no circle is possible. S2 ≡ x2 + y2 + 6x + 6y + 9 = 0
When the third line doesnot pass through point of C1 = (–g, –t) = (3, 3)
intersection of the lines & is not parallel to either of
them 4 circle are possible. r1 = 32 + 32 − 9 = 3

and C2 = (–3, –3)

C1 r2 = 32 + 32 − 9 = 3

C 1C 2 = 62 + 62 = 6 2
C3 C4
r1 + r2 = 6
C2
They do not intersect with each other
Since their radius are same
When the 3 line is parallel to one of the line then
rd
∴ External direct common tangents are parallel

r

C1
M

Also, the point of intersection of transverse common


tangents is midpoint of C1 and C2 (same radii)
9 .92 | Circle

M = (0, 0) Every line L is normal to circle


r 3 1 Statement-I is true & statement-2 is false
sinθ = = =
MC1 32 + 32 2
Sol 27: (A) Length of tangent from (13, 6)
θ = 45º
Angle between tangents = 2θ = 90º = 132 + 62 − 13 × 6 + 8 × 6 − 75 = 10

Sol 24: (B, C) S1 ≡ x2 + y2 + px + py – 7 = 0


S2 ≡ x2 + y2 – 10x + 2py + 1 = 0
C  P
S1 & S2 are orthogonal
∴2gg1 + 2ff1 = c + c1
⇒ p(–5) + p . p = – 6
32 + ( −4 ) + 75
2
∴ Radius of circle
= = 10
⇒ p2 – 5p + 6 = 0
∴tanθ = 1
⇒ P =2 or p = 3
∴θ = 45º
Sol 25: (A, B, D) (A) Two circles having the same center. Angle between tangents = 2θ = 2 × 45 = 90º
Have infinitely many common normal.
Director circle of a circle S1 is such that the angle
(B) Radical axis is always perpendicular to the line between the tangents drawn from any point on director
joining center but it does not necessarily bisect the line circle to S1 is 90º
joining the centres. It bisects only when r1 = r2
(C) Let the centres of the two circles be C1 & C2. Sol 28: (D) (1, 5) lies outside the circle
Consider a point O, on radical axis centres which lies on as 1 + 25 – 2 – 7 = 17 > 0
the line C1C2
Now OC12 = r12 + OT12
C1

OC22 = r22 + OT22


Since length of tangent is same
∴ OC12 < OC22 if r1 < r2 C2
⇒ OC1 < OC2 ∴Two circles shown C1, C2 are possible
∴ Statement-I is false
O
C1 r1 C1 Sol 29: (A) Since x + y – 2 = 0 is ⊥ bisector of C1C2
r2
T1 Radius of both the circles is same
T2
Since length of common chord = 2 2

(D) Consider two circles having same centre these ABCD is a square since diagonals are equal & ⊥ to
circles donot have a radical axis each other B
r r
Assertion Reasoning Type

L L
A C
 1  2  (0, 0) 2 2 (2, 2)
Sol 26: (C) L : k(x − y − 4) + 7x + y + 20 = 0 r r
L are the lines passing through intersection of L1 & L2
D
Point of intersection is (– 2, – 6)
Which is center of circle c
M a them a ti c s | 9.93

When their centres are mirror image of each other AH2 – HB2 = AP × AQ – [AB × AP
1
then the common chord bisects C1C2 and × length + AB × AQ – AB2 – APAQ]
2
1 AB2 = AP × AQ – [AB(AP + AQ) – AB2 – APAQ]
of common chord = c1c2
2 2AP × AQ
∴ AB =
AP + AQ

∴ Statement-I is true
C 1 C2
2 Statement-II: AK2 = AB × AO & AK2 = AP × AQ
C1 C2
C1 C2 (AP + AQ)
2 ∴AB × = AP × AQ
2
tanθ = 1 2AP × AQ
∴AB =
(AP + AQ)
θ = 45º
The circles are orthogonal
Comprehension Type
When the centres are mirror image & length of chord
= distance between centres then the two circles are
Paragraph 1: (32-34)
orthogonal. The inverse is not true
∴ Statement-II is wrong Sol 32: (B) A : {(x, y) : y ≥ 1 }
B : {(x, y) : x2 + y2 – 4x – 2y – 4 = 0}
Sol 30: (A) Let AB = diameter
C : {(x, y) : x + y = 2

A
B
C
P
A B 2

The circle with AB as diameter is


There is only one point P of intersection of region A,
(x – x1) (x – x2) + (y – y1).(y – y2) = 0 B, C
when C is obtuse, then C lies inside the circle
D(x3, y3) < 0 Sol 33: (C) B : x2 + y2 – 4x – 2y – 4 = 0

(Power of a point inside a circle < 0) ⇒ 2x2 + 2y2 – 8x – 4y – 8 = 0


⇒ (x – 5)2 + (x + 1)2 + (y – 1)2 + (y – 1)2 – 36= 0
Sol 31: ∴f(x) = 36
H
Sol 34: (C) S is director circle of B
∴B : (x – 2)2 + (y – 1)2 = 9
Q A
O B P s : (x – 2)2 + (y – 1)2 = 18
Arc of B = 9p
K
Arc of s = 18p
Since KPHQ are concyclic Area of S-Area of B = 9π
∴ PB × BQ = HB × BK = (HB)2
∴ (AB – AP) (AQ – AB) = (HB)2
Also AH2 = AP × AQ (from property of tangents)
9 .94 | Circle

Paragraph 2: (35-36) 1 10
= × =1
2 5/2
Sol 35: (D) Let m be slope of tangents
∴ (y – 2) = m( x – 4) are equation of tangent θ = 45º
π 3π
s = x2 + y2 = 4 Angle at minor arc = π – =
4 4
For tangents c2 = a2 (1 + m2)
∴(2 – 4m)2 = 4(1 + m2) Sol 39: (C)
A
12 m2 – 16m = 0 1
/4 
4 m(3m – 4) = 0 90
O
4 4
m = 0 or m = ⇒ tanθ =
3 3
θ ∈ (45º, 60º) Ans.(D) B

π π π
Sol 36: (B) the tangents are q1 = – =
2 4 4
y = 2 & 4x – 3y – 10 = 0 π
10 5 & θ + q1 =
∴ Intercepts made on x axis by 2nd tangent = = 2
4 2 π
θ=
4
Paragraph 3: (37-39)

Sol 37: (D) s : x2 + y2 – 4x – 1 = 0 Previous Years’ Questions


L : y = 3x – 1
Sol 1: (B) For required circle, P(1, 8) and O(3,2) will be
Centre of circle = (2, 0)
the end point of its diameter.
Radius = 5
Length of chord AB A

= 2 r 2 − (perpendicular distance from centre )


2
(1,8) P O (3,2)

Perpendicular distance from centre B

6 −1 5
= = ∴ (x − 1)(x − 3) + (y − 8)(y − 2) =
0
10 2
⇒ x2 + y 2 − 4x − 10y + 19 =
0
5 5
AB = 2 5− = 2 = 10
2 2
1
Sol 2: (B) 18
= (3α )(2r) ⇒ α=
r 6
2
Sol 38: (A)
y

 2
 D C (, 2r)
(0, 2r)
B

(r, r)

Angle subtends at minor arc = 180 – angle at major arc x’ x


1  AB (0, 0) A B (2, 0)
tanθ =
2 ± distance y’
M a them a ti c s | 9.95

2r Sol 3: (A) Let the locus of centre of circle be (h, k)


Line, y =− (x − 2α ) is tangent to circle
α touching
(x − r)2 + (y − r)2 = r2 (y − 1)2 + x2 =
1 and x-axis shown as
2α 3r and
= = αr 6
Clearly, from figure,
r=2 y
x
r x r
B A

 (0, 1) (h, k)
r l kl
2r O A
O 1
90 lkl
- x’ x
r O B

y’
C 2 x-r D
2x
Distance between O and A is always 1+|k|,
x
r x r ie, (h − 0)2 + (k − 1)2 =1+ | k |,
B A
⇒ h2 + k 2 − 2k + 1
=1 + k 2 + 2 | k |
 r
2r ⇒ h2 =+
2 | k | 2k
O
90 ⇒ x2 =
2 | y | +2 y
-
r  y, y ≥ 0
where | y |=  y, y ≥ 0
where | y |= − y, y < 0
− y, y < 0
C 2 x-r D 
2
∴ x2 =2y + 2y, y ≥ 0
2x ∴ x =2y + 2y, y ≥ 0
and x22 = 2y + 2y, y < 0
and x = 2y + 2y, y < 0
Alternate solution 2
⇒ = x2 4y when y ≥ 0
⇒ = x 4y when y ≥ 0
1 and x22 0 when y < 0
=
(x + 2x) × 2r =
18 and x
= 0 when y < 0
2 ∴ {(x,
= y) : x22 4y, when y ≥ 0} ∪ {(0, y) : y < 0}
xr = 6 ...(i) ∴ {(x,
= y) : x 4y, when y ≥ 0} ∪ {(0, y) : y < 0}

x −r
In ∆AOB, tan θ =
r Sol 4: (A) From figure it is clear that ∆PRQ and ∆RSP
and in ∆DOC are similar.
2x − r
tan(90o − θ) =
r
x −r r P
r r
R
∴ = 90- 
r 2x − r 
⇒ x(2x − 3r) =
0
3r
⇒ x= ....(ii) X
2
-


90

From Eqs. (i) and (ii) we get


Q S
r=2
9 .96 | Circle

PR PQ There are given two distinct chords which are bisected


∴ =
RS RP at x-axis then, there will be two distinct values of h
⇒ PR 2 =
PQ ⋅ RS satisfying Eq. (i).
So, discriminant of this quadratic equation must be > 0.
⇒ PR =
PQ.RS
⇒ 2r = PQ ⋅ RS ⇒ D >0
⇒ ( −3p) − 4 ⋅ 2(p2 + q2 ) > 0
2

Sol 5: (B) Choosing OA as x-axis, A=(r, 0), B=(0, r) and ⇒ 9p2 − 8p2 − 8q 2 > 0
any point P on the circle is (r cos θ,r sin θ) . If (x, y) is the ⇒ p2 − 8q2 > 0
centroid of ∆PAB , then
y
⇒ p2 > 8q2

Sol 7: Equation of given circle C is

(x − 3)2 + (y + 5)2 =9 + 25 − 30
A (p,q)
ie, (x − 3)2 + (y + 5)2 =
22
C (h,0)
x’ x
O Centre = (3, -5)
B
y’
If L1 is diameter, then 2(3) + 3( −5) + p − 3 = 0 ⇒ p = 12

3x r cos θ + r + 0
= ∴ L1 is 2x + 3y + 9 =0
and 3y
= r sin θ + 0 + r L2 is 2x + 3y + 15 =
0
∴ (3x − r)2 + (3y − r)2 =
r2 Distance of centre of circle from L2 equals

Hence, locus of P is a circle. 2(3) + 3( −5) + 15 6


= < 2 (radius of circle)
22 + 33 13
Sol 6: (D) From equation of circle it is clear that circle
passes through origin. Let AB is chord of the circle. ∴ L2 is a chord of circle C.
y Statement-II, false.

Sol 8: (A) Let the, equation of circles are


C1 : (x − 1)2 + (y − 1)2 =
(1)2

A (p,q)
and C2 : (x − 1)2 + (y − 1)2 =
( 2)2

C (h,0)
x’ x C2
O Q
B
D (0,2) C (2,2)
y’ C1 P

A ≡ (p,q) ⋅ C is mid point and coordinate of C is (h, 0)


2 2
(1,1) x + y - 2x - 2y =0

Then coordinates of B are (-p + 2h, -q) and B lies on the


circle x2 + y 2 = px + qy. we have
A (0,0) B (2,0)
2 2
( −p + 2h) + ( −q) = p( −p + 2h) + q( −q)
x2 + y2 - 2x - 2y =0
⇒ p2 + 4h2 − 4ph + q2 =−p2 + 2ph − q2
⇒ 2p2 + 2q2 − 6ph + 4h2 =
0
∴ Coordinates of P(1 + cos θ,1 + sin θ)
⇒ 2h2 − 3ph + p2 + q2 =
0 ....(i)
M a them a ti c s | 9.97

and Q(1 + 2 cos θ,1 + 2 sin θ) (Rejecting ‘2’ because origin and centre of C are on the
same side of PQ).
∴ PA2 + PB2 + PC2 + PD2
The point ( 3,1) satisfies Eq. (i).
= {(1 + cos θ)2 + 1 + sin θ)2 } + {(cos θ − 1)2 + (1 + sin θ)2 }
+ {(cos θ − 1)2 + (sin θ − 1)2 } ∴ Equation of circle C is (x − 3)2 + (y − 1)2 =
1.
+ {(1 + cos θ)2 + (sin θ − 1)2 }
Sol 12: (A) Slope of line joining centre of circle to point
= 12 D is
Similarly, QA2 + QB2 + QC2 + QD2 =
16 3
−1
2 1
∑ PA2
12 tan θ
= =
∴ = 0.75
= 3 2 3
∑ QA 16 2 − 3
2
It makes an angle 30o with x-axis.
Sol 9: (C) Let C be the centre of the required circle.
Now, draw a line parallel to L at a distance of r1
(radius of C1 ) from it.
Now, CC1 = AC
⇒ C lies on a parabola.

∴ Point E and F will make angle 150o and -90o with


Sol 10: (C)
x-axis.
Since, AG = 2
∴ E and F are given by
1
∴ AT1 =T1G = x− 3 y −1
2 = = 1
o
cos150 sin150o
As A is the focus, T1 is the vertex and BD is the directrix x− 3 y −1
of parabola. and = = 1
cos( −90 ) sin( −90o )
o

Also, T2 T3 is latus reetum.  3 3


∴ = E  ,  and
= F ( 3,0)
 2 2
 
D C

Sol 13: (D) Clearly, points E and F satisfy the equations


G given in option (d).
T1
T2
B
Sol 14:
A
2x2 + y 2 − 3xy =
0 (given)
T3 2 2
⇒ 2x − 2xy − xy + y = 0
1
∴ T2 T3 =
4⋅ ⇒ 2x(x − y) − y(x − y) =
0
2
⇒ (2x − y)(x − y) = 0
1 1 4
∴ Area of ∆T1 T2 T3 = × × =1 sq unit
2 2 2 ⇒ y = 2x, y = x are the equations of straight lines
passing through origin.
Sol 11: (D) Let centre of circle C be (h, k) Now, let the angle between the lines be 2θ and the
line y = x
3h + k − 6
Then, =1 Makes angle of 45o with x-axis.
3+1
Therefore, tan(45o + 2θ) =2 (slope of the line y =2x)
⇒ 3h + k − 6 =+2
⇒ 3h + k =4 ....(i)
9 .98 | Circle

E D Again, in ∆OCA
3 3
30 tan θ= ,OA
=
OA tan θ
(3,1) 3
=
( −3 + 10 )
3(3 + 10 )
F
∴ =
( −3 + 10 )(3 + 10 )
3(3 + 10 )
tan 45o + tan2θ = = 3(3 + 10 )
⇒ 2
= (10 − 9)
1 − tan 45o × tan2θ
Sol 15:
y y
y = 2x
y=x T1
3 B1
C
3
A 30 A1 L
x’ x
S c1 M O N C2

A2

45 B2
x’
O T2

y’ y’

1 + tan2θ
⇒ = 2 l
1 − tan2θ
C1 C2
(1 + tan2θ) − (1 − tan2θ) 2 −1 1
⇒ = =
(1 + tan2θ) + (1 − tan2θ) (2 + 1) 3
2 tan2θ 1
⇒ =
2 3
1 From figure it is clear that, triangle OLS is a right triangle
⇒ tan2θ = with right angle at L.
3
2 tan θ 1 Also, OL = 1 and OS = 2
⇒ =
1 − tan θ 3
2
1
∴ sin(∠LSO)= ⇒ ∠LSO= 30o
2
⇒ (2 tan θ) ⋅ 3 = 1 − tan2 θ
Since, SA
= SA2 , ∆SA1 A2 is an equilateral triangle.
⇒ tan2 θ + 6 tan θ − 1 = 0 1

The circle with centre at C1 is a circle inscribed in the


−6 ± 36 + 4 × 1 × 1
⇒ tan θ = ∆SA1A2. Therefore, centre C1 is centroid of ∆SA1A2. This,
2
C1 divides SM in the ratio 2:1. Therefore, coordinates of
C1 are (-4/3,0) and its radius C1 M=1/3
−6 ± 40
⇒ tan θ = ∴ Its equation is (x + 4 / 3)2 + y 2 = (1 / 3)2  …(i)
2
⇒ tan θ = −3 ± 10 The other circle touches the equilateral triangle SB1B2
 π ∆
⇒ tan θ = −3 + 10  0 < θ <  Externally. Its radius r is given by r = ,
 4 s−a
1 3
where B=
1B2 a. But
= ∆ (a)(SN)
= a
2 2
3 a
and s − a= a − a=
2 2
Thus, r =3
M a them a ti c s | 9.99

⇒ Coordinates of C2 are (4,0) 2π π


∠POQ = =
∴ Equation of circle with centre at C2 is 3 k
π π
(x − 4)2 + y 2 =
32  …(ii) ∠ROS = =
3 k
Equations of common tangents to circle (i) and circle ⇒k =3
C are
1 Sol 17: (D)
x= −1 and y = ± (x + 2) [T1 and T2 ]
3
Equation of common tangents to circle (ii) and circle C
are
1
x= −1 and y = ± (x + 2) [T1 and T2 ]
3
Two tangents common to (i) and (ii) are T1 and T2 at O.
To find the remaining two transverse tangents to (i) and
(ii), we find a point I which divides the joint of C1C2 in
the ratio= r1 : r2 1=
/3:3 1:9 2 2
( x − h) + ( y − 2 ) h2
=
Therefore, coordinates of I are (-4/5,0)
Equation of any line through I is y = m (x+4/5). It will
Passes through ( −1,0 ) , then
2 2
touch (i) if ( −1, −h) + ( 0 − 2 ) =h2
2
 −4 4 
m + −0 (1 + h) − h2 = −4
 3 5 1
= ⇒ (1 + h − h)(1 + h + h) =−4
1+m 2 3

⇒ −
8m 1
= 1 + m2 ⇒ (1 )( 2h + 1 ) =−4
15 3
h = −5 / 2
⇒ 64m2 =
25(1 + m2 )
Circle is
⇒ 39m2 =
25
2 2
5  5 2 5
⇒ m= ±  x +  + ( y − 2) = 
39  2 2

Therefore, these tangents are Only ( −4,0 ) satisfies the eq. of circle.

5  4 D is the Answer.
y=
± x + 
39  5 
2 2
Sol 18: (D) Any tangent to circle x + y =4 and
2 2
Sol 16: Let equation of Circle be x + y =
4 and parallel 2

chords =are x 1 and − 13


( x − 3) 1 , then
+ y2 =

3x1 + 0 × y1 − 4
=1
x12 + y12

3x1 − 4
=1
y
⇒ 3x1 − 4 =
2

⇒ x1 =
2,2 / 3
2 4 2 
P ≡ (1,13) , Q ≡ (1, −13) ⇒ ( x1 , y1 ) ≡ ( 2,0 ) &  , 
3 3 
 
( ) (
R ≡ − 3,1 ,S ≡ − 3, −1 )
9 .100 | Circle

Tangents If (h,k) mid-point, the eq. of chord of contact T=S1


2x 4 2
2 . x + 0 = 4 ⇒ x = 2 and + 4
=
3 3
⇒ x+2 2 = 6

Sol 19: (A) The tangent to circle x2 + y 2 =


4 at ( 3,1) 4
PT ≡ 3x + y =4
Eq. of L is x − 3y =
λ
Circle ( x − 3) + y 2 =
2
1 is touching L, then
xh + yk = h2 + k 2  … (ii)
(i) & (ii) are identical, then
3− 3×0 − λ
=1 h k h2 + k 2
1+ 3 = =
5f 4f − 20 45
3 − λ =2 9h
t=
λ =1,5 h + k2
2

45k
Tangents x − 3y =
1 4t − 20 =
h + k2
2

Sol 20: Let P be (2t2, 4t) lies on circle 9h × 4 45k


⇒ − 20 =
2 2
4t 4 + 16t2 − 4t2 − 16t =
0 h +k h + k2
2

⇒ t 4 + 4t2 − t2 − 4t =
0 ⇒ 36h − 20 n2 + k 2 = (
45k )
(
⇒ t ( t − 1) t + t + 4 =
2
0 ) ( )
⇒ 20 h2 + k 2 − 36h + 45k =
0
⇒t=0,1
( )
⇒ 20 x2 + y 2 − 36x + 45y =
0

Sol 22: (C) Let circle touching x-axis be


(x − α) + (y − k )
2 2
=k 2  … (i)
Also for y-axis intercepts

(0 − α ) = ( y − k ) = k2
2 2

⇒ ( y − k ) = k 2 − α2
2

P ≡ ( 2, 4 ) Q ≡ ( 0,0 ) S ≡ ( 2,0 )
⇒ y = k ± k 2 − α2
1
∆= × 2 × 4 = 4 sq units
2 = 2 k 2 −=
Intercept α2 2 7

 4t − 20  ⇒ k 2 = 7 + α2
Sol 21: (A) Let point P be  t, 
 5  From (i) α =3
⇒ k 2 = 7 + 9 = 16
 4t − 20 
Eq. of chord of contact xt + y  =9 ⇒ k =±4
 5 
(5t ) x + y ( 4t − 20 ) =
45  … (i) Circle: ( x − 3 ) + ( y − 4 ) =
2
16
2

( x − 3) + ( y + 4 )
2 2
16
=
M a them a ti cs | 9.101

Sol 23: (D) Let tangent to parabola y2=8x Passes through (2, 8), then
Be ty=x+2t2 8=2m-2m-m3 ⇒ m=-2

It is also tangent to circle, then


Normal
2t2
= 2
y=-2x=12
1 + t2

( 4 − 2) + ( 4 − 8 )
2 2
(
⇒ 4t 4 =2 1 + t2 ) SP = = 4 + 16 = 2 5 units

⇒ 2t 4 − t2 − 1 =0 Let SQ : QP = 1 : λ

( )(
⇒ 2t2 + 1 t2 − 1 =
0 ) 1
.
λ
S ( 2,8 ) Q (h,k ) P ( 4, 4 )
⇒ t =±1

⇒ S ≡ ( 2, 4 ) & R ≡ ( 2, −4 )  4 + 2λ 8λ + y 
Q (h,k ) ≡  ,  lies on
 1+ λ 1+ λ 
⇒ P ≡ ( −1,1) & Q ( −1,1)
Circle, then
1
Area=
2
( 2 + B ) × 3= 15 sq units  4 + 2λ
2
  8λ + 4 
2

 − 2 +  − 8 =
4
 1+ λ   1+ λ 
2 2
Sol 24: (B, C) Let circle be x + y + 2yx + 2y + C =
0 2 2
 2   −4 
Applying condition for orthogenality ⇒  + 4
 =
 1+ λ   1+ λ 
2gx − 1 + 2f × 0 = C + ( −15)
20
⇒ 4
=
15 and 2g × 0 + 2f × 0 = C − 1
⇒ 2g + c = (1 + λ )
2

⇒C=
1
⇒g=
7 ⇒ 1+ λ = 5
⇒=
λ 5 −1
Also,
1 + 2f + C =
0
⇒ f =−1
SQ
=
QP
( 5 −1 )
Centre ≡ ( −g, −f ) ≡ ( −7,1)
x – intercept of normal at P is 6 slope of tangent at Q
Radius
= 2
g +f −C
= 2
49 + 1 −=
1 7 1
is
2
Hence, B and C are the correct options

Sol 26: (C) For point of intersection


Sol 25: (A, C, D) (x-2)2+(y−8)2=4
2y+y2=3
Shortest distance is measured along common normal
⇒ y 2 + 2y − 3 =0
The equation of normal to parabola
y=mx-2am-am3 ⇒ y=mx-2m-m3 ⇒ (y + 3)(y − 1) =
0
9 .102 | Circle

⇒ y = 1, -3 Sol 27: (A, C) Let point P be ( cos θ,sin θ ), The tangent


and normal are
⇒= ( 2,1)
The eq. of tangent at 1, 2 ( ) x cos θ + y sin θ =1 x sin θ − y cos θ = 0

3
2x + y = (i)  1 − cos θ 
⇒ θ ≡ 1, 
 sin θ 
Eqs. of circle C2 and C3
 y cos θ 
C2 ≡ x2+(y-y2)2 =12 ⇒ E ≡ 1, , y  ≡ (h,k ) (let )
 sin θ 
C2 ≡ x2+(y-y3)2 =12
 1 − Cosθ  cos θ
⇒ h=
 .
If line (i) touches circle, then  Sinθ  sin θ
2 ×0 + y −3  1 − Cosθ 
=2 3 k= 
2 +1  Sinθ 

⇒ y −3 =
6

⇒ y −3 =±6 h
1−
⇒y=−3,9
K= h + K2
2

K
⇒ y2 =
−3 and y 3 =
9 1−
h2 + K 2
⇒ Centres Q 2 ≡ ( 0, −3)
Q 3 ≡ ( 0,9 )
⇒ Q 2Q 3 =
12

For point of contact R2 and R3 ⇒ K2 + h = h2 + K 2 ⇒ y 2 + x = x2 + y 2

( )
R 2 ≡ 2 2, −1 and R 3 ≡ −2 2,7 ( )
(4 2)
2
+ (8) =
2
R 2R3= 32 + 64= 96= 16 × 6= 4 6

0(0,0), R2( 2 ,-1), R3(-2 2 ,7)

0 0 1
1
Area of ∆ OR2R3= 2 2 −1 1
2
−2 2 7 0

=
1
2
( 1
7×2 2 −2 2 = ×6×2 2
2
)
= 6 2 sq. units
Now
Area of ∆ PQ2Q3

2 1 1
1
= 0 −3 1
2
0 9 1

1
= 2 ( −3 −=
9 )  6 2 sq units
2 

You might also like